You are on page 1of 59

ROBBINS AND COTRAN REVIEW OF PATHOLOGY

FINALS TOPICS

FINALS

Female Genital Tract Breast Endocrine System Central Nervous System Peripheral Nerves and Skeletal Muscles

FEMALE GENITAL TRACT

QUESTION RATIONALE

1. A 31-year-old, sexually active woman has had a mucopurulent vaginal B. Chlamydia trachomatis
discharge for 1 week. On pelvic examination, the cervix appears reddened The inflammatory cells in the cervical discharge with redness (erythema),
around the os, but no erosions or mass lesions are present. A Pap smear and the biopsy findings indicate that the patient has cervicitis. Chlamydia
shows numerous neutrophils, but no dysplastic cells. A cervical biopsy trachomatis is the most common cause of cervicitis in sexually active
specimen shows marked follicular cervicitis. Which of the following women.
infectious agents is most likely to produce these findings? Candidiasis, gonorrhea, and trichomoniasis also are common.
A. Candida albicans Candidiasis often produces a scant, white, curd-like vaginal discharge;
B. Chlamydia trachomatis Gonorrhea may have an associated urethritis; and Trichomonas may
C. Gardnerella vaginalis produce a profuse homogeneous, frothy, and adherent yellow or green
D. Herpes simplex virus vaginal discharge.
E. Human papillomavirus Gardnerella is found in bacterial vaginosis, a common condition caused
F. Neisseria gonorrhoeae by overgrowth of bacteria. Gardnerella infection produces a moderate,
G. Trichomonas vaginalis homogeneous, low viscosity, adherent vaginal discharge that is white or
gray and has a characteristic fishy odor; clue cells are seen on a wet
mount.
Herpetic infections are more likely to manifest as clear vesicles on the
skin in the perineal region.
Infection with human papillomavirus is associated with condylomata,
dysplasias, and carcinoma.

2. A 31-year-old woman has had vulvar pruritus along with a thick, whitish, A. Candida albicans
odorless, globular vaginal discharge for the past week. On pelvic The presence of the budding cells with pseudohyphae indicates a fungal
examination, the cervix appears erythematous, but there are no erosions infection with Candida. Candidal (monilial) vaginitis is common; this
or masses. A Pap smear shows budding cells and pseudohyphae. No organism is present in about 5% to 10% of women. Recurrent episodes of
dysplastic cells are present. Which of the following infectious agents is vaginal candidiasis may be associated with non-albicans species.
most likely to produce these findings? The inflammation tends to be superficial, and there is typically no invasion
A. Candida albicans of underlying tissues.
B. Chlamydia trachomatis
C. Neisseria gonorrhoeae Ureaplasma is a bacterial agent, as is Chlamydia, and both can produce
D. Trichomonas vaginalis cervicitis.
E. Ureaplasma urealyticum Neisseria gonorrhoeae, a gram-negative diplococcus, is the causative
agent of gonorrhea.
Infection with Trichomonas vaginalis can produce a purulent vaginal
discharge, but the organisms are protozoa and do not produce hyphae.

3. A 17-year-old sexually active girl has had pelvic pain for 1 week. A pelvic C. Ectopic Pregnancy
examination shows mild erythema of the ectocervix and pain on palpation Gonorrheal infections can lead to salpingitis and pelvic inflammatory
of right adnexa. A Pap smear shows many neutrophils, but no disease with scarring of the fallopian tube. This predisposes to ectopic
dysplastic cells. A cervical culture grows Neisseria gonorrhoeae. If the pregnancy, because the fertilized ovum has difficulty traversing the tube.
infection is not adequately treated, she will be at increased risk for which
of the following complications? Gonorrhea and other genital tract infections do not cause dysfunctional
A. Cervical carcinoma bleeding. Gonorrhea does not carry the risk of dysplasias or carcinomas
B. Dysfunctional uterine bleeding that human papillomavirus infection does. Gonorrhea and other infections
C. Ectopic Pregnancy do not contribute to endometrial hyperplasia. The cause of
D. Endometrial hyperplasia endometriosis is not known with certainty, but infection does not seem to
E. Endometriosis play a role in this process. Placenta previa results from low-lying
F. Placenta previa implantation of the placenta and is not related to sexually transmitted
diseases.

4. A 25-year-old woman has experienced discomfort during sexual A. Bartholin Gland


intercourse for the past month. On physical examination, there are no Bartholin glands may become obstructed, inflamed, and cystic because of
lesions of the external genitalia. Pelvic examination shows a focal area of abscess formation, which then produces focal pain.
swelling on the left posterolateral inner labium that is very tender on
palpation. A 3-cm cystic lesion filled with purulent exudate is excised. In A Gartner duct cyst may form in the lateral vaginal wall from the remnant
which of the following structures is this lesion most likely to develop? of a wolffian (mesonephric) duct; the cyst is filled with fluid and is usually
A. Bartholin gland not inflamed.
B. Gartner duct Hair follicles are not present at the inner labia.
C. Hair follicle The Bartholin gland lies just inferior to the fascia of the urogenital
D. Urogenital diaphragm diaphragm and just anterior to the vestibular bulb, which is not glandular
E. Vestibular bulb and does not become cystic.

❗USE AT YOUR OWN RISK ❗ page 1 of 59


ROBBINS REVIEW

5. A 53-year-old postmenopausal woman is concerned about pale areas C. Lichen sclerosus et atrophicus
on her labia that have been slowly enlarging for the past year. The areas Lichen sclerosus et atrophicus is most common in postmenopausal
cause discomfort and become easily irritated. Physical examination shows women. Although this lesion is not premalignant, there is a 1% to 5% risk
pale gray to parchment- like areas of skin that involve most of the labia that women with this condition will later develop a squamous cell
majora, labia minora, and introitus. The introitus is narrowed. A biopsy carcinoma. In contrast, lichen simplex chronicus appears grossly as
specimen is taken and microscopically shows thinning of the squamous leukoplakia from squamous hyperplasia and is not associated with
epithelium, a dense band of upper dermal hyaline collagen, and scattered malignancy.
upper dermal mononuclear inflammatory cells. What is the most likely
diagnosis? Extramammary Paget disease is rare; it produces reddish areas of
A. Extramammary Paget disease scaling and is caused by the presence of adenocarcinoma-like cells at the
B. Human papillomavirus infection dermal-epidermal junction.
C. Lichen sclerosus et atrophicus Human papillomavirus infection is associated with condylomata
D. Pelvic inflammatory disease acuminata and with squamous epithelial dysplasias.
E. Vulvar intraepithelial neoplasia Pelvic inflammatory disease results from infection of internal genital
organs with organisms such as Neisseria gonorrhoeae and Chlamydia
trachomatis.
Vulvar intraepithelial neoplasia is marked by dysplastic squamous
epithelial changes.

6. A 40-year-old woman has noted pruritic patches on her vulva for the past D. Squamous cell hyperplasia
4 months. On physical examination there are multiple 1.5- to 3-cm white, Squamous cell hyperplasia, formerly called lichen simplex chronicus, is
scaly plaques on the vulva. A biopsy of one lesion is taken, and on most often seen in women aged 30 to 50. It is not premalignant, but it may
microscopic examination, it shows epidermal thickening with coexist with lichen sclerosus, and leukoplakia suggests the possibility of a
hyperkeratosis and intense dermal inflammation. Mitoses are seen in squamous cell carcinoma to be distinguished on biopsy.
keratinocytes, but they exhibit no atypia. What is the most likely diagnosis?
A. Condyloma acuminatum Human papillomavirus infection is associated with condylomata
B. Contact dermatitis acuminata and with squamous epithelial dysplasias that show
C. Psoriasis keratinocyte vacuolization and minimal inflammation.
D. Squamous cell hyperplasia Contact dermatitis produces red patches and vesicles, with intense
E. Vulvar intraepithelial neoplasia round cell infiltrates, and tends to diminish when the offending antigen
(such as a skin cream) is not used.
Psoriatic lesions have extensive scaling, and microscopically show focal
thinning of the epidermis with marked parakeratosis.
Vulvar intraepithelial neoplasia is marked by dysplastic squamous
epithelial changes

7. A 36-year-old sexually active D. Human papillomavirus


woman has noticed that warty The epithelium shows typical features of infection with human
vulvar lesions have been papillomavirus (HPV)—specifically, prominent perinuclear vacuolization
increasing in size and number (koilocytosis) and angulation of nuclei. These lesions, called condylomata
over the past 5 years. On physical acuminata, may occur anywhere on the anogenital surface, as single
examination, there are multiple lesions or, more commonly, as multiple lesions. They are not
0.5- to 2-cm, red-pink, flattened precancerous. Condylomata are associated with HPV infection, often
lesions with rough surfaces types 6 and 11.
present on the vulva and
perineum. One of the larger Candidal infections produce vaginitis or cervicitis with exudate and
lesions is excised; its microscopic erythema.
appearance is shown in the figure. Which of the following infectious agents Chlamydial infections may produce urethritis, cervicitis, and pelvic
is most likely to produce these lesions? inflammatory disease.
A. Candida albicans Haemophilus ducreyi is the agent that produces the soft chancre of
B. Chlamydia trachomatis chancroid.
C. Haemophilus ducreyi Treponema pallidum is the infectious agent of syphilis, characterized by
D. Human papillomavirus the gross appearance of a hard chancre.
E. Treponema pallidum

8. A 57-year-old woman recently noticed a pale area of discoloration on the E. Vulvar intraepithelial neoplasia
labia. Pelvic examination shows the presence of a 0.7-cm flat, white area Presence of dysplastic cells occupying half of the thickness of the
on the right labia majora. A biopsy specimen is obtained and on epithelium suggests vulvar intraepithelial neoplasia (VIN). The incidence
microscopic examination shows dysplastic cells that occupy half the of these lesions has been increasing, probably because of more cases of
thickness of the squamous epithelium, with minimal underlying chronic human papillomavirus (HPV) infections. Some VIN lesions may progress
inflammation. In situ hybridization shows human papillomavirus type 16 to invasive cancers.
DNA in the epithelial cells. What is the most likely diagnosis?
A. Chronic vulvitis Chronic inflammation alone does not produce dysplasia
B. Condyloma acuminatum A condyloma is usually a raised, nodular lesion. It also is caused by HPV,
C. Lichen sclerosus et atrophicus principally HPV-6 and HPV-11.
D. Squamous hyperplasia Lichen sclerosus is a vulvar dystrophy characterized by thinning of the
E. Vulvar intraepithelial neoplasia squamous epithelium and sclerosis of the dermis.
Similar to VIN, squamous hyperplasia, another form of vulvar dystrophy,
can appear as an area of leukoplakia, but no dysplastic changes are
present.

9. A 52-year-old woman has noted an increasing size of a red, pruritic B. Extramammary Paget disease
lesion on her left labium over the past 7 months. On physical examination, Extramammary Paget disease is a rare condition that is usually not
this rough, scaly lesion is 0.4 × 0.9 cm. The perineum appears normal; associated with an underlying malignancy, in contrast to Paget disease of
there is no lymphadenopathy, and there are no rectal lesions. A Pap smear the breast. In many cases, the extramammary Paget cells remain in the

❗USE AT YOUR OWN RISK❗ page 2 of 59


ROBBINS REVIEW

shows no abnormal findings. The lesion is excised and on microscopic epithelium, often for years, creating an annoying itchy red lesion.
examination shows large atypical cells lying singly or in small clusters However, in a fourth of cases there may be an underlying neoplasm, so
within the epidermis. These cells have abundant cytoplasm that stains with that local invasion and even metastases are possible.
periodic acid–Schiff (PAS). What is the most likely diagnosis?
A. Condylomata acuminata A condyloma is the result of human papillomavirus (HPV) infection and
B. Extramammary Paget disease leads to koilocytotic atypia, but the cells of a condyloma are not malignant.
C. Lichen sclerosus et atrophicus Lichen sclerosis is a white patch of epithelial thinning with dermal fibrosis
D. Lichen simplex chronicus and chronic inflammation that can be extensive enough to constrict the
E. Vulvar intraepithelial neoplasia vaginal orifice; it may have an autoimmune basis, and there is an
increased risk for future development of a squamous carcinoma.
Lichen simplex chronicus is an area of epithelial hyperplasia that has
no atypia and no association with malignancy.
Vulvar intraepithelial neoplasia has neoplastic cells extending the full
thickness of the epithelium; it is related to HPV infection.

10. An 18-year-old sexually B. Diethylstilbestrol (DES) exposure


active woman has had The microscopic appearance is that of a malignant tumor containing cells
dyspareunia followed by vaginal with a clear cytoplasm. Vaginal clear cell carcinomas are associated with
bleeding for the past month. On exposure of the patient’s mother to diethylstilbestrol (DES) during
pelvic examination, a red, pregnancy. These tumors are generally first diagnosed in the late teenage
friable, 2.5-cm nodular mass is years.
seen on the anterior wall of the
upper third of the vagina. The Congenital adrenal hyperplasia can produce masculinization in girls,
microscopic appearance of a manifesting in early childhood.
biopsy specimen is shown in the Infection with human papillomavirus is associated with squamous
figure. Which of the following epithelial dysplasias and malignancies, not with clear cell
conditions is likely to have adenocarcinomas.
contributed most to the origin of this neoplasm? Polycystic ovary disease can lead to hormonal imbalances from excess
A. Congenital adrenal hyperplasia androgen production, but vaginal neoplasms do not arise in this
B. Diethylstilbestrol (DES) exposure setting.
C. Human papillomavirus infection Trichomonal infections do not give rise to neoplasia.
D. Polycystic ovary syndrome (PCOS)
E. Trichomonas vaginitis

11. A 4-year-old girl is brought to the physician by her parents, who noticed D. Sarcoma botryoides
blood stained underwear and “something” protruding from her external Embryonal rhabdomyosarcoma is an uncommon vaginal tumor that can
genitalia. On physical examination, there are polypoid, grape-like masses be found in girls younger than 5 years. Because it forms polypoid, grape-
projecting from the vagina. Histologic examination of a biopsy specimen like masses, it is sometimes called sarcoma botryoides. Histologically, it
from the lesion shows small, round tumor cells, some of is a small round blue cell tumor that shows skeletal muscle differentiation
which have eosinophilic straplike cytoplasm. Immunohistochemical in the presence of muscle-specific proteins such as desmin.
staining shows desmin, vimentin, and myogenin in these cells. What is the
most likely diagnosis? Neuroblastomas are childhood tumors and are also small blue cell
A. Clear cell carcinoma tumors, but they occur in the adrenal glands or extra-adrenal sympathetic
B. Infiltrating squamous cell carcinoma C chain.
C. Neuroblastoma Clear cell carcinomas of the vagina may be related to in utero exposure
D. Sarcoma botryoides to maternal diethylstilbestrol (DES), but have an onset in the second or
E. Vulvar intraepithelial neoplasia third decades of life.
Invasive squamous cell carcinomas are rare in very young patients,
and they show histologic evidence of squamous epithelial differentiation,
and are related to human papillomavirus (HPV) infection.
Vulvar intraepithelial neoplasia is a carcinoma in situ of the vulvar skin,
squamous in origin, and related to HPV infection.

12. A healthy 30-year-old woman comes to the physician for a routine B. Multiple sexual partners
health maintenance examination. No abnormalities are found on physical Cervical intraepithelial neoplasia (CIN) I represents minimal (mild)
examination. A screening Pap smear shows cells consistent with a low- dysplasia (low-grade squamous intraepithelial lesion, or LSIL) and is a
grade squamous intraepithelial lesion (LSIL). Subsequent cervical biopsy potentially reversible process. Dysplasias are preneoplastic and may
specimens confirm the presence of cervical intraepithelial neoplasia (CIN) progress to carcinomas if not treated. Risk factors for cervical dysplasias
I. Which of the following risk factors is most likely related to her Pap smear and carcinoma include early age at first intercourse, multiple sexual
findings? partners, and a male partner with multiple previous sexual partners. These
A. Diethylstilbestrol (DES) exposure factors all increase the potential for infection with human papillomavirus.
B. Multiple sexual partners
C. Oral contraceptive use Diethylstilbestrol (DES) exposure is a factor in the development of clear
D. Prior treatment for a malignancy cell carcinomas of the vagina and cervix.
E. Vitamin B12 (cobalamin) deficiency Use of oral contraceptives, which contain very low amounts of
hormonally active compounds, does not cause cervical dysplasia or
carcinoma.
Treatment of cancers does not typically result in dysplasias, although the
atypical changes in epithelial cells from radiation and/or chemotherapy
may be challenging to distinguish from cancer.
A vitamin B12 deficiency may produce some megaloblastic epithelial
changes, but not dysplasia.

❗USE AT YOUR OWN RISK❗ page 3 of 59


ROBBINS REVIEW

13. A 33-year-old woman comes to E. Vaccination for human papillomavirus


her nurse practitioner for a routine The figure shows a high-grade squamous intraepithelial lesion (HSIL)
health maintenance examination. On termed cervical intraepithelial neoplasia (CIN) III because the dysplasia
physical examination, there are no involves the full thickness of the cervical epithelium. Such lesions arise
abnormal findings. A Pap smear more frequently in women who have had first intercourse at an early age,
shows abnormalities; colposcopy and have multiple sexual partners, or have a male partner with multiple sexual
a biopsy are performed. The figure partners. These factors are believed to increase the risk of infection with
shows the microscopic appearance of human papillomavirus (HPV), particularly types 16 and 18, which have
the biopsy specimen. Which of the high risk for dysplasias and carcinomas of the cervix. Because of the
following is the best strategy to causal relationship with HPV infection, the use of HPV vaccines has been
prevent the development of this lesion? shown to prevent disease progression.
A. Avoidance of tobacco products
B. Consumption of a diet rich in vegetables Cervical squamous neoplasia has not been shown to be associated with
C. Maintenance of an ideal body weight smoking, diet, body weight, or hormonal influences.
D. Use of oral contraceptives
E. Vaccination for human papillomavirus

14. A 42-year-old woman has a Pap smear as part of a routine health E. Risk for invasive carcinoma
maintenance examination. There are no remarkable findings on physical This patient’s cervical intraepithelial neoplasia (CIN) II is a high-grade
examination. The Pap smear shows cells consistent with a high grade squamous intraepithelial lesion (HSIL) that may progress to invasive
squamous intraepithelial lesion (HSIL) with human papillomavirus type 18. carcinoma in several years if not treated, particularly because she has a
Cervical biopsy specimens are obtained, and microscopic examination high-risk subtype of HPV.
confirms the presence of extensive moderate dysplasia (CIN II) along with
intense chronic inflammation with squamous metaplasia in the Infection with HPV often drives this process, but the presence of HPV
endocervical canal. What is the most likely explanation for proceeding with alone does not determine therapy. HPV infection cannot be eradicated
cervical conization for this patient? with antibiotics, but patients may clear the virus.
A. Her reproductive years are over Chronic cervicitis with squamous metaplasia is not a malignant lesion
B. HPV infection cannot be treated and does not determine therapy in this case.
C. Perimenopausal state The conization can preserve fertility in women who are of childbearing
D. Presence of chronic cervicitis age.
E. Risk for invasive carcinoma

15. A 28-year-old sexually active woman comes to her physician’s E. Viral inactivation of the RB1 gene product
assistant for a routine health maintenance examination. There are no Dysplasias of the cervix should not be ignored because they naturally
abnormal findings on physical examination. She has been taking oral progress to more severe dysplasias and to invasive carcinomas. Although
contraceptives for the past 10 years. A Pap smear shows a high-grade not all cases progress, the physician should not take this chance.
squamous epithelial lesion (HSIL), also termed moderate dysplasia, or Dysplasias are strongly related to human papillomavirus (HPV) infections,
cervical intraepithelial neoplasia (CIN) II. What is the most likely molecular and HPV DNA can be found in up to 90% of cases. Viral E6 and E7
pathogenesis for this finding? proteins bind to Rb to up-regulate cyclin E. In about 10% to 15% of cases,
A. Estrogenic stimulation of cell proliferation there is no evidence of HPV, and other factors may play a role in the
B. Inheritance of a tumor suppressor gene mutation development of dysplasia. With such HPV infection, the Pap smear may
C. Recurrent gonococcal cervicitis show changes of cervical intraepithelial carcinoma (CIN) I (low-grade
D. Up-regulation of anti apoptosis genes squamous intraepithelial lesion, or LSIL). Oral contraceptives with low-
E. Viral inactivation of the RB1 gene product dose estrogens and progestins do not increase the risk of dysplasia
significantly. Though the RB1 gene is involved, this is not an inherited
problem, and retinoblastomas are not seen with HPV infection. Cervicitis
usually is due to bacterial or fungal organisms and is not a significant risk
for dysplasia or carcinoma. Anti-apoptosis genes such as BCL2 do not
play a role in cervical carcinogenesis.

16. A 34-year-old woman has a routine Pap smear for the first time. The C. No further therapy
results indicate that dysplastic cells are present, consistent with a high- Microinvasive squamous cell carcinomas of the cervix are stage I lesions
grade squamous intraepithelial lesion (HSIL), also called cervical that have a survival rate similar to that of in situ lesions. Such minimal
intraepithelial neoplasia (CIN) III. She is referred to a gynecologist, who invasiveness does not warrant more aggressive therapies. The likelihood
performs colposcopy and takes multiple cervical biopsy specimens that all of metastasis or recurrence is minimal.
show CIN III. Conization of the cervix shows a focus of microinvasion at
the squamocolumnar junction. Based on these findings, what is the next
most likely step in treating this patient?
A. Bone scan for metastatic lesions
B. Course of radiation therapy
C. No further therapy
D. Pelvic exenteration
E. Vaginal hysterectomy

17. A 45-year-old woman has had a F. Squamous cell carcinoma


small amount of vaginal bleeding and a The cervical lesion shown in the figure is large and ulcerative and projects
brownish, foul-smelling discharge for the into the vagina. It is most likely an invasive squamous cell carcinoma that
past month. On pelvic examination, there has infiltrated the subepithelial region.
is a 3-cm lesion on the ectocervix, shown
in the figure. Microscopic examination of Dysplastic changes confined to the epithelium represent cervical
the lesion is most likely to show which of intraepithelial neoplasia and do not form mass lesions.
the following? Glandular invasive lesions indicate an adenocarcinoma, which is much
A. Adenocarcinoma less common than squamous cell carcinoma of the cervix.
B. Cervical intraepithelial neoplasia Chronic cervicitis has erythema, but no mass effect.
C. Chronic cervicitis Clear cell carcinomas are uncommon and most likely found arising in the

❗USE AT YOUR OWN RISK❗ page 4 of 59


ROBBINS REVIEW

D. Clear cell carcinoma vagina.


E. Extramammary Paget disease Extramammary Paget disease usually arises on the vulva, producing an
F. Squamous cell carcinoma eczematous lesion, not a mass, because the neoplastic cells are confined
to the epithelium and to adjacent skin adnexa.

18. A 43-year-old woman has had postcoital bleeding for 6 months. She F. Squamous cell carcinoma
experienced menarche at age 11 years and has had 12 sexual partners This woman has several risk factors for the development of cervical
during her life. She continues to have regular menstrual cycles without squamous cell carcinoma, including multiple sexual partners, documented
abnormal intermenstrual bleeding. Pelvic examination shows a focal, infection of the cervix with high-risk human papillomavirus (HPV) type 16,
slightly raised area of erythema on the cervix at the 5 o’clock position. A and diagnosis of a high-grade squamous intraepithelial lesion (HSIL). The
Pap smear shows a high grade squamous intraepithelial lesion (HSIL), also remaining choices are not related to HPV infection.
termed severe cervical intraepithelial neoplasia (CIN III). Analysis of cells
from the cervix shows the presence of human papillomavirus type 16. Clear cell carcinomas of the cervix are uncommon; some are associated
Which of the following malignancies is she at greatest risk of developing if with maternal use of diethylstilbestrol (DES) in pregnancy.
the lesion is not treated? An immature teratoma arises in the ovary.
A. Clear cell carcinoma A Krukenberg tumor is a form of metastasis to the ovary.
B. Immature teratoma Leiomyosarcomas are rare and typically arise in the myometrium,
C. Krukenberg tumor although they can occur in the cervix.
D. Leiomyosarcoma Sarcoma botryoides is a vaginal lesion that typically occurs in young
E. Sarcoma botryoides girls.
F. Squamous cell carcinoma

19. A 13-year-old girl began menstruation 1 year ago. She now has A. Anovulatory cycles
abnormal uterine bleeding, with menstrual periods that are 2 to 7 days long Anovulatory cycles are a common cause of dysfunctional uterine bleeding
and 2 to 6 weeks apart. The amount of bleeding varies from minimal in young women who are beginning menstruation and in women
spotting to a very heavy flow. On physical examination, there are no approaching menopause. There is prolonged estrogenic stimulation that
remarkable findings. A pelvic ultrasound scan shows no abnormalities. is not followed by secretion of progesterone.
Which of the following is most likely to produce these findings?
A. Anovulatory cycles An ectopic pregnancy has acute findings and does not have a prolonged
B. Ectopic pregnancy course.
C. Endometrial carcinoma Endometrial carcinomas are rare in patients this age.
D. Endometrial polyp Polyps are more common in older women.
E. Uterine leiomyomata Submucosal leiomyomas are a cause of less variable bleeding and are
more likely to be seen in older women.

20. A 41-year-old G5, P5 woman has noticed lower abdominal pain with B. Endometrial neutrophilic infiltrates
fever for the past 2 days. She delivered a normal term infant 1 week ago. Acute endometritis in this case is the result of retained products of
On examination, she has a temperature of 37.4° C. There is a foul-smelling conception after delivery. Endometritis may also follow premature rupture
vaginal discharge. Which of the following pathologic findings is she most of membranes with ascending infection to the uterine cavity. There is often
likely to have? polymicrobial infection with organisms found in the vagina. Some cases of
A. Cervical intraepithelial neoplasia chronic endometritis may be associated with Neisseria and Chlamydia
B. Endometrial neutrophilic infiltrates infections and produce lymphoplasmacytic infiltrates within the
C. Myometrial smooth muscle neoplasm endometrium.
D. Ovarian endometrioma
E. Tubal granulomatous inflammation Cervical dysplasias are confined to the epithelium and are usually
F. Vaginal trichomoniasis asymptomatic so that detection is by Pap smear.
A myometrial neoplasm is unlikely to produce acute inflammation.
An ovarian endometrioma is a mass lesion resulting from continued
hemorrhage into a focus of endometriosis; but this mass lesion is not
associated with pregnancy, and endometriosis may be a cause for
infertility.
Mycobacterium tuberculosis infection may spread to the female genital
tract, most often the fallopian tube, but acute signs are unlikely to be
present, and inflammation of the tube can be a cause for infertility.
Vaginitis may produce acute inflammation with discharge, but
trichomonal infections typically are associated with a watery gray-to-
green discharge.

21. A 35-year-old woman presents with infertility. She has had A. Aromatase inhibitors
dysmenorrhea, dyspareunia, and pelvic pain on defecation for 4 years. In 30% to 40% of cases, endometriosis presents with infertility, menstrual
Laparoscopic examination reveals red-blue nodules on the surface of the irregularities, and pelvic pain. The presence of endometrial tissue in the
uterus and extensive adhesions between ovaries and the fallopian tubes. nodules confirms this diagnosis. The glands in the nodules are
Histologic examination of a biopsy from one of the nodules shows hyperplastic but show no evidence of malignancy; in addition, all the genes
hyperplastic endometrial glands and hemorrhage in the stroma. Molecular implicated in endometrial cancer are normal. Hypomethylation of the two
analysis of the biopsy material reveals hypomethylation of the promoter genes, NR5A1 (steroidogenic factor 1) and ESR2 (estrogen receptor beta)
regions of the genes that encode steroidogenic factor 1 and estrogen is found in endometriosis. These lead to overproduction of
receptor beta. There are no mutations in the PTEN, KRAS, and MLH1 prostaglandins and estrogens.
genes. Which of the following is an appropriate treatment modality in this
case? Aromatase inhibitors are used to suppress estrogen production.
A. Aromatase inhibitors Lesions of endometriosis are not neoplastic and chemotherapy or major
B. Chemotherapy surgery with organ removal is not indicated. Endometriosis is not
C. Estrogen infectious, so antibiotics are not indicated.
D. Antitubercular therapy

❗USE AT YOUR OWN RISK❗ page 5 of 59


ROBBINS REVIEW

E. Total abdominal hysterectomy

22. A 36-year-old woman has had menorrhagia and pelvic pain for six A. Adenomyosis
months. She had a normal, uncomplicated pregnancy 10 years ago but has In adenomyosis, endometrial glands extend from the endometrium down
failed to conceive since then. She has been sexually active with one partner into the myometrium. The process may be superficial, but occasionally it
for the past 20 years and has had no dyspareunia. On pelvic examination is extensive, and the uterus becomes enlarged two to four times its normal
she has a symmetrically enlarged uterus, with no apparent nodularity or size because of a reactive thickening of the myometrium.
palpable mass. A serum pregnancy test result is negative. What is the most Chronic endometritis does not extend to the myometrium and does not
likely diagnosis? increase uterine size.
A. Adenomyosis Endometrial hyperplasias do not increase the size of the uterus because
B. Chronic endometritis the process is limited to the endometrium.
C. Endometrial hyperplasia In endometriosis, endometrial glands and stroma are found outside the
D. Endometriosis uterus in such sites as peritoneum, ovaries, and ligaments.
E. Leiomyoma A leiomyoma is a myometrial tumor mass that, if large, produces an
asymmetric uterine mass.

23. A 32-year-old woman has cyclic abdominal pain that coincides with her C. Endometriotic cyst
menses. Attempts to become pregnant have failed over the past 5 years. Endometriosis is a condition in which functional endometrial glands are
There are no abnormal findings on physical examination. Laparoscopic found outside the uterus. Common sites include ovaries, uterine
examination shows numerous hemorrhagic 0.2- to 0.5-cm lesions over the ligaments, rectovaginal septum, and pelvic peritoneum. These
peritoneal surfaces of the uterus and ovaries. Which of the following endometrial glands can respond to ovarian hormones so that cyclic
ovarian lesions is most likely to be associated with her findings? abdominal pain coincides with menstruation. Recurrent hemorrhages may
A. Fibroma incite scarring and the formation of fibrous adhesions in the pelvic region.
B. Brenner tumor This may cause distortion of the ovaries and fallopian tubes and may lead
C. Endometriotic cyst to infertility. One common variation is formation of an endometrioma, or
D. Krukenberg tumor chocolate cyst, which represents a focus of endometriosis that becomes
E. Metastatic choriocarcinoma an expanding cystic lesion as its center becomes filled with chocolate-
F. Mucinous cystadenocarcinoma brown sludge from the recurrent hemorrhage. The remaining choices are
not associated with endometriosis, although endometrioid tumors may
form in foci of endometriosis.

24. A 49-year-old perimenopausal D. Repeated failure of ovulation


woman has had menometrorrhagia Endometrial hyperplasia with numerous crowded glands as shown in the
for the past 3 months. On physical figure results from excessive estrogenic stimulation. This lesion often
examination, there are no occurs with failure of ovulation about the time of menopause. Hyperplasias
remarkable findings. The do not develop from endometritis. Estrogen-secreting ovarian tumors also
microscopic appearance of an may produce endometrial hyperplasia, but teratomas are not known for
endometrial biopsy specimen is this phenomenon. A secretory pattern of the endometrium is seen in
shown in the figure. The patient pregnancy, not the proliferative pattern shown in the figure. Oral
undergoes a dilation and curettage, contraceptives contain small doses of estrogenic compounds that do not
and the bleeding stops, with no lead to hyperplasia.
further problems. What condition is most likely to produce these findings?
A. Chronic endometritis
B. Ovarian mature cystic teratoma
C. Pregnancy with missed abortion
D. Repeated failure of ovulation
E. Use of oral contraceptives

25. A 52-year-old perimenopausal woman has had vaginal bleeding for a B. Endometrial glands resembling stratum basalis
week. She has no medical problems and takes no medications. She has an endometrial polyp, seen most often in perimenopausal and
Hysteroscopy is performed and there is a single, 2-cm, smooth, soft mass postmenopausal women. The lesion can lead to abnormal bleeding, but
protruding into the endometrial cavity. Biopsies are taken. What is rarely gives rise to a malignancy.
microscopic examination of this lesion most likely to show?
A. Endocervical glands with squamous metaplasia Endocervical glands with squamous metaplasia are seen most often
B. Endometrial glands resembling stratum basalis with chronic cervicitis.
C. Papillae with marked cellular atypia Papillae with marked cellular atypia are seen with the serous type of
D. Smooth muscle cells in bundles endometrial carcinoma.
E. Tubular glands lined by clear cells with glycogen Smooth muscle cells in bundles characterize a leiomyoma, which may
be submucosal.
Tubular glands lined by clear cells with glycogen are seen with the
rare clear cell carcinoma

26. A 42-year-old woman has had menometrorrhagia for the past 2 months. C. Granulosa-theca cell tumor
She has no history of prior irregular menstrual bleeding, and she has not The mass is probably producing estrogen, which has led to endometrial
yet reached menopause. On physical examination, there are no vaginal or hyperplasia. Estrogen-producing tumors of the ovary are typically sex cord
cervical lesions, and the uterus appears normal in size, but there is a right tumors, such as a granulosa-theca cell tumor or a thecoma-fibroma, the
adnexal mass. An abdominal ultrasound scan shows the presence of a 7- former more often being functional.
cm solid right adnexal mass. Endometrial biopsy shows
hyperplastic endometrium, but no cellular atypia. What is the most likely Teratomas can contain various histologic elements, but not estrogen
lesion that underlies her menstrual abnormalities? producing tissues.
A. Corpus luteum cyst Endometriosis can give rise to an adnexal mass called an
B. Endometrioma endometrioma, which enlarges over time. Endometrial glands are
C. Granulosa-theca cell tumor hormonally sensitive, but they do not produce hormones.
D. Mature cystic teratoma Corpus luteum cysts are common, but they are unlikely to produce

❗USE AT YOUR OWN RISK❗ page 6 of 59


ROBBINS REVIEW

E. Metastasis estrogens.
F. Polycystic ovarian syndrome Metastases to the ovary do not cause increased estrogen production.
Polycystic ovarian syndrome would involve both ovaries.

27. A 62-year-old childless woman noticed a blood-tinged vaginal A. Adenocarcinoma


discharge twice during the past month. Her last menstrual period was 10 Postmenopausal vaginal bleeding is a red flag for endometrial carcinoma.
years ago. Bimanual pelvic examination shows that the uterus is normal in Such carcinomas often arise in the setting of endometrial hyperplasia.
size, with no palpable adnexal masses. There are no cervical erosions or Increased estrogenic stimulation is thought to drive this process, and risk
masses. Her body mass index is 33. Her medical history indicates that for factors include obesity, type 2 diabetes mellitus, hypertension, and
the past 30 years she has had hypertension and type 2 diabetes mellitus. infertility.
An endometrial biopsy specimen is most likely to show which of the Choriocarcinomas are gestational in origin.
following? A submucosal leiomyosarcoma could produce vaginal bleeding, but the
A. Adenocarcinoma uterus would be enlarged because leiomyosarcomas tend to be large
B. Choriocarcinoma masses.
C. Leiomyosarcoma Malignant müllerian mixed tumors are much less common than
D. Malignant müllerian mixed tumor endometrial carcinomas, but they could produce similar findings.
E. Squamous cell carcinoma Malignant müllerian mixed tumors are typically uterine neoplasms that
have glandular and stromal elements; the malignant stromal component
can be heterologous and may resemble mesenchymal cells that are not
ordinarily found in the myometrium, such as cartilage.
Squamous carcinomas of the endometrium are rare, and more likely to
arise in the cervix.

28. A study of patients with postmenopausal uterine bleeding reveals that B. Endometrioid carcinoma
some of them have malignant neoplasms that arise from prior atypical Most endometrial cancers have the endometrioid pattern and are
hyperplastic lesions. The peak incidence is between 55 and 65 years of classified as type I endometrial carcinomas. They arise in the setting of
age in women who have obesity, hypertension, and/or diabetes mellitus. unopposed estrogen stimulation and may also have PTEN mutations as
Molecular analysis reveals mutations of the PTEN tumor suppressor gene well as microsatellite instability.
in most of them. Their malignancies tend to remain localized for years
before spreading to local lymphatics. Which of the following neoplasms is In contrast, type II endometrial carcinomas occur at an older age in the
most likely to have these characteristics? background of atrophic endometrium; they usually have a serous
A. Clear cell carcinoma carcinoma pattern, but may also exhibit clear cell and müllerian mixed
B. Endometrioid carcinoma patterns, and TP53 mutations are common.
C. Leiomyosarcoma Leiomyosarcomas and stromal sarcomas are far less common than
D. Müllerian mixed tumor endometrial carcinomas, and they have no known risk factors.
E. Serous carcinoma
F. Stromal sarcoma

29. A 62-year-old obese, nulliparous woman has an episode of vaginal C. Endometrial hyperplasia
bleeding, which produces only 5 mL of blood. On pelvic examination, there Endometrial carcinomas can be associated with estrogenic stimulation
is no enlargement of the uterus, and the cervix appears normal. A Pap from anovulatory cycles, nulliparity, obesity, and exogenous estrogens (in
smear shows cells consistent with adenocarcinoma. Which of the following higher amounts than found in birth control pills). These risks may initially
preexisting conditions is most likely to have contributed to the development give rise to endometrial hyperplasia that can progress to endometrial
of this malignancy? carcinoma if the estrogenic stimulation continues. Atypical endometrial
A. Adenomyosis hyperplasias progress to endometrial cancer in about 25% of cases.
B. Chronic endometritis Adenomyosis increases the size of the uterus and is not a risk for
C. Endometrial hyperplasia endometrial carcinoma.
D. Human papillomavirus infection Chronic endometritis and human papillomavirus infection
E. Use of oral contraceptives (associated with squamous epithelial dysplasias and neoplasia) do not
cause cancer.

30. A 40-year-old nulliparous woman has had menorrhagia for the past 6 C. Increased aerobic glycolysis
months. On physical examination, her blood pressure is 154/93 mm Hg, She has obesity, diabetes mellitus, and nulliparity— factors that contribute
there are no cervical lesions or adnexal masses, and the uterus is normal to development of endometrial hyperplasias and carcinomas caused by
in size. She is 155 cm (5 feet 1 inch) tall and weighs 74.5 kg (body mass hyperestrinism. She has complex endometrial hyperplasia with atypicality
index 38). A Pap smear shows atypical glandular cells of uncertain of cells, which is a precursor for type I endometrial carcinoma. These
significance. Hemoglobin A1c concentration is 9.8%. Endometrial biopsy lesions often have loss of PTEN tumor suppressor genes. In many if not
shows complex hyperplasia with atypia; molecular analysis detects loss of all cancers, there is activation of aerobic glycolysis (i.e., glycolysis even in
PTEN gene heterozygosity and enhanced AKT phosphorylation. Which of the presence of enough oxygen)—the so-called Warburg effect. This is
the following metabolic pathways is most likely to be activated in this linked to loss of PTEN and offers a growth advantage to tumor cells. When
tumor? aerobic glycolysis is stimulated there is a reciprocal decrease in
A. Decreased glucose uptake oxidative phosphorylation. Tumors are metabolically active, so glucose
B. Decreased prostaglandin synthesis uptake and glycogen utilization is enhanced and not reduced. This
C. Increased aerobic glycolysis uptake is the basis for positron emission tomography (PET) scans, where
D. Increased glycogen storage positron-emitting fludeoxyglucose F 18 is preferentially taken up into foci
E. Increased oxidative phosphorylation of malignancy. In many cancers the COX-2 enzyme is upregulated (e.g.,
colon cancer), and this leads to increased prostaglandins, but this is not
related to PTEN loss.

31. A healthy 59-year-old woman has had a feeling of pelvic heaviness for C. Leiomyomas
the past 11 months. There is no history of abnormal bleeding, and her last The masses shown are well circumscribed, suggesting the presence of
menstrual period was 8 years ago. Her physician palpates an enlarged multiple benign tumors. Leiomyomas (fibroids) can be present in one third
nodular uterus on bimanual pelvic examination. A Pap smear shows no to one half of all women. They tend to enlarge during the reproductive
abnormalities. Pelvic CT scan shows multiple solid uterine masses; there years, and then stop growing or involute after menopause. Although
is no evidence of necrosis or hemorrhage. A total abdominal hysterectomy leiomyomas are often asymptomatic, leiomyomas that are submucosal

❗USE AT YOUR OWN RISK❗ page 7 of 59


ROBBINS REVIEW

is performed. Based on the gross may produce menometrorrhagia and chronic blood loss, leading to iron
appearance of the mass shown in deficiency anemia. About 10% of complete moles are complicated by
the figure, what is the most likely invasive mole, which is unlikely to produce a large, circumscribed mass.
diagnosis? A leiomyosarcoma arises de novo, not from a leiomyoma, and is usually
A. Adenomyosis a larger, more irregular mass composed of more pleomorphic spindle cells
B. Endometriosis with many mitoses. Decreased ovarian function after menopause
C. Leiomyomas accelerates bone loss, which may be severe enough to be termed
D. Metastases osteoporosis, but this process is not related to female genital tract
E. Tuberculosis neoplasia. Preeclampsia with hypertension and proteinuria is associated
with abnormal decidual vascularization and placental ischemia.

32. A 53-year-old woman whose last menstrual period was 3 years ago D. Smooth muscle cells
notes vaginal bleeding for a week. On physical examination, her uterus is Leiomyosarcomas arising in the uterine corpus account for about 5% of
markedly enlarged, but there are no adnexal masses. CT imaging reveals all GYN malignancies, and is most often present in postmenopausal
an irregular 8-cm mass in the body of the uterus. A total abdominal women. The cellular atypia, coagulative necrosis, and numerous mitoses
hysterectomy is performed, and microscopic examination of the soft, distinguish this neoplasm from the much more common leiomyoma (which
hemorrhagic mass shows spindle cells with atypia and numerous mitoses. does not give rise to leiomyosarcoma), both derived from smooth muscle.
There is coagulative necrosis of tumor cells. Which of the following is the Anaplastic cytotrophoblasts are seen with choriocarcinomas. Cross
most likely cell of origin for this mass? striations are seen with rhabdomyosarcomas. Adenocarcinomas arise
A. Cytotrophoblastic cells from glandular epithelium. Germ cells give rise to ovarian tumors such as
B. Endometrial glandular cells teratoma and dysgerminoma. Squamous carcinomas are much more
C. Germ cells common but arise in the cervical portion of the uterus.
D. Smooth muscle cells
E. Squamous epithelial cells

33. A 69-year-old woman has passed blood per vagina for a month. On A. Endometrial biopsy
pelvic examination no abnormal findings are noted. Which of the following Causes for postmenopausal uterine bleeding include endometrial atrophy,
diagnostic procedures should be performed next? carcinoma, hyperplasia, and polyps. An early potentially curable
A. Endometrial biopsy endometrial carcinoma should not be missed.
B. Magnetic resonance imaging Even if the MRI is normal, a biopsy is still indicated. Infections are
C. Microbiologic culture uncommon at this age and unlikely to cause bleeding. A Pap smear is
D. Pap smear insensitive for detection of endometrial lesions. She is postmenopausal
E. Pregnancy test and neither pregnancy nor gestational trophoblastic disease is probable.

34. A 28-year-old woman has had fever, pelvic pain, and a feeling of pelvic A. Chlamydia trachomatis
heaviness for the past week. Pelvic examination shows a palpable painful Sexually transmitted diseases are the most common cause of
left adnexal mass. Laparoscopy shows an indistinct left fallopian tube that inflammation of the fallopian tube. When the incidence of gonorrhea
is part of a 5-cm circumscribed, red tan mass involving the left adnexal caused by Neisseria gonorrhoeae decreases in a population, the
region. Which of the following infectious agents is most likely to produce proportion of cases of salpingitis caused by Chlamydia and Mycoplasma
these findings? increases. The fallopian tube can become distended and adherent to the
A. Chlamydia trachomatis ovary and may form a tubo-ovarian abscess. These are features of pelvic
B. Haemophilus ducreyi inflammatory disease.
C. Herpes simplex virus Haemophilus ducreyi causes chancroid, which can produce
D. Mycobacterium tuberculosis erythematous papules of the external genitalia or vagina, but grossly
E. Treponema pallidum visible lesions may not be present in women.
Herpes simplex virus most often involves the external genitalia, but it
may produce vaginal or cervical lesions; it is unlikely to advance farther.
Mycobacterium tuberculosis is an uncommon cause of salpingitis.
Treponema pallidum infection causes syphilis, which does not produce
florid inflammation with mass effect, just a chancre.

35. A 19-year-old woman has the sudden onset of abdominal pain. On C. Follicle cyst
physical examination, there is pelvic pain on palpation. Her stool is Follicle cysts and lutein cysts of the ovary are so common that they are
negative for occult blood. The serum and urine pregnancy tests are virtually normal findings and incidentalomas in diagnostic studies. Though
negative. Transvaginal ultrasound shows no intrauterine gestational sac, most of them are less than 2 cm and asymptomatic, occasionally they can
and uterus and adnexa are normal in size. Culdocentesis yields a small be larger (4 to 5 cm) and even enlarge a little more in response to mid
amount of blood-tinged fluid. Which of the following has most likely led to cycle hormones, occasionally rupturing to produce pain and bleeding.
these findings? The negative pregnancy test helps to eliminate intrauterine or ectopic
A. Ectopic pregnancy pregnancy.
B. Endometriosis Endometriosis tends to produce more chronic pain, and though there is
C. Follicle cyst hemorrhage in the lesions, it tends to be contained within the lesions. The
D. Invasive mole pregnancy test would be positive with an invasive mole, with uterine
E. Pelvic inflammatory disease enlargement from the mass of grapelike villi.
Pelvic inflammatory disease tends to produce chronic pain, and there is
unlikely to be bleeding.

36. A 21-year-old woman experienced menarche at age 14 years and had D. Polycystic ovarian syndrome
regular menstrual cycles for the next 3 years. For the past year, she has Polycystic ovarian syndrome (PCOS) is a disorder of unknown origin that
is typically associated with oligomenorrhea, obesity, and hirsutism. The
MR image shows an enlarged ovary with multiple round cysts of increased
signal intensity. It is thought to be caused by abnormal regulation of
androgen synthesis.

Teratomas are mass lesions that can be bilateral, but usually are not

❗USE AT YOUR OWN RISK❗ page 8 of 59


ROBBINS REVIEW

had oligomenorrhea and has developed symmetric, and aside from struma ovarii not known for hormonal
hirsutism. She has noticed a abnormalities.
10-kg weight gain in the past 4 months. On Krukenberg tumors represent metastatic disease involving the ovaries,
pelvic examination, there are no vaginal or usually from a primary site in the gastrointestinal tract, and are rare among
cervical lesions, the uterus is normal in patients of this age.
size, and the adnexa are prominent. A Cystadenocarcinoma can be bilateral; however, androgen production by
pelvic ultrasound scan shows that each ovarian tumors is except by the Sertoli-Leydig cell tumors.
ovary is twice normal size, whereas the Abscesses are usually unilateral and do not account for the hormonal
uterus is normal in size. Magnetic changes seen in this patient.
resonance imaging is shown in the figure.
Which of the following conditions is most
likely to be present in this woman?
A. Immature teratomas
B. Krukenberg tumors
C. Ovarian cystadenocarcinomas
D. Polycystic ovarian syndrome
E. Tubo-ovarian abscesses

37. A 35-year-old woman has had increasing abdominal enlargement for B. Cystadenocarcinoma
the past 6 months. She states that she feels like she is pregnant, but results Cystadenocarcinomas are common ovarian tumors that are often bilateral.
of a pregnancy test are negative. On physical examination, there is The serous type occurs more frequently than the mucinous type and is
abdominal distention with a fluid wave. A pelvic ultrasound scan shows typically unilocular, whereas mucinous tumors are multilocular. Serous
bilateral cystic ovarian masses, 10 cm on the right and 7 cm on the left. cystadenocarcinomas account for more than half of
The masses are surgically removed. On gross examination, the excised ovarian cancers. As the name indicates, they are cystic in appearance.
masses are unilocular cysts filled with clear fluid, and papillary projections They may be benign, borderline, or malignant. Benign tumors have a
extend into the central lumen of the cyst. Microscopic examination shows smooth cyst wall with small or absent papillary projections. Borderline
that the papillae are covered with atypical cuboidal cells that invade tumors have increasing amounts of papillary projections.
underlying stroma. Psammoma bodies are present. What is the most likely
diagnosis? Endometrioid tumors resemble endometrial carcinomas and may arise
A. Endometrioid tumor in foci of endometriosis.
B. Cystadenocarcinoma Dysgerminomas are solid tumors of germ cell origin.
C. Dysgerminoma Granulosa cell tumors can be solid and cystic and may produce
D. Granulosa cell tumor estrogens.
E. Mature cystic teratoma Mature cystic teratomas typically contain abundant hair and gooey
F. Sertoli-Leydig cell tumor sebaceous fluid within the cystic cavity; surrounding tissues are formed
from various germ layers.
Sertoli-Leydig cell tumors are rare, yellow-brown, solid masses; they
may secrete androgens or estrogens.

38. A 40-year-old woman has noticed D. Mucinous cystadenocarcinoma


progressive enlargement of the abdomen Mucinous tumors of the ovary are of epithelial origin, are less common
over the past 5 months, although her diet has than serous tumors, and tend to be multiloculated. The appearance of
not changed, and she has been exercising ascites suggests metastases, which is most common with surface
more. Physicalexamination shows no epithelial neoplasms of the ovary.
palpable masses, but a fluid wave is present.
Paracentesis yields 500 mL of slightly cloudy Choriocarcinomas rarely reach this size because they metastasize early;
fluid. Cytologic examination of the fluid shows they are typically hemorrhagic.
malignant cells. An abdominal ultrasound Granulosa cell tumors and dysgerminomas tend to be solid masses.
scan shows a 15-cm multilobular mass that Teratomas are germ cell tumors differentiating into three germ layers;
involves the right adnexal region. The uterus malignant transformation is rare, and is usually an element of squamous
is normal in size. The mass is surgically removed; the figure shows the cell carcinoma from the ectodermal component.
gross features of a section of the excised mass. What is the most likely
diagnosis?
A. Choriocarcinoma
B. Dysgerminoma
C. Granulosa cell tumor
D. Mucinous cystadenocarcinoma
E. Teratoma w/ malignant transformation

39. A 56-year-old woman has had weight loss accompanied by abdominal A. BRCA1
enlargement for the past 5 months. There is a family history of breast and Some familial cases of ovarian carcinoma (usually serous
ovarian carcinoma. On physical examination, there are no lesions of the cystadenocarcinoma) are associated with the homozygous loss of the
cervix, and the uterus is normal in size, but there is a left adnexal mass. An BRCA1 gene. This tumor-suppressor gene also plays a role in the
abdominal ultrasound scan shows a 10-cm cystic mass in the left adnexal development of familial breast cancers. Familial syndromes account for
region, with scattered 1-cm peritoneal nodules, and ascites. Cytologic less than 5% of all ovarian cancers, however.
studies of peritoneal fluid show malignant cells. Which of the following
mutated genes is most likely a factor in the development of this neoplasm? The ERBB2 gene may be overexpressed in ovarian cancers; however,
A. BRCA1 mutations of this gene do not give rise to familial tumors, and it is best
B. ERBB2 (HER2) known for an association with breast carcinomas.
C. MYC Mutations of the RAS and MYC oncogenes occur sporadically in many
D. KRAS types of cancer.
E. RB1 The RB1 gene, a tumor suppressor, can be involved in familial
malignancies, including retinoblastoma and osteosarcoma.

❗USE AT YOUR OWN RISK❗ page 9 of 59


ROBBINS REVIEW

40. A 42-year-old woman has noted D. Peritoneal implants


dull lower abdominal pain for the past This is a borderline serous tumor of the ovary, and the figure shows a
year. She reports no abnormal complex papillary projection into the cyst lumen. This is the most common
bleeding. On physical examination serous ovarian tumor, and though most act in a benign fashion even when
there is a large left adnexal mass. The peritoneal implants are present, some tend to recur, particularly when
pregnancy test is negative. KRAS or BRAF mutations are present, and the implants are invasive.
Transvaginal ultrasound shows a right Distant metastases are unlikely. Serous tumors do
adnexal 10-cm cystic mass filled with not have hormonal effects, either estrogenic to drive endometrial
fluid. The mass is removed and has hyperplasia, or androgenic to drive masculinization. Ovarian carcinomas
the microscopic appearance shown in do not transform to sarcomas, and sarcomas at this site are rare.
the figure. Which of the following is most likely to be associated with this
lesion?
A. Brain metastases
B. Endometrial hyperplasia
C. Masculinization
D. Peritoneal implants
E. Sarcomatous transformation

41. A 33-year-old woman has had A. Mature squamous epithelium


dull, constant abdominal pain for 6 A cystic tumor with a mass of hair in the lumen is the typical appearance
months. On physical examination, the of a mature cystic teratoma. This tumor also is known as a dermoid cyst
only finding is a right adnexal mass. because it is cystic and filled with hair and sebum derived from well-
CT scan of the pelvis shows a 7-cm differentiated ectodermal structures. Teratomas with mature tissue
circumscribed cystic mass on the elements are benign tumors of germ cell origin, and they can contain
right ovary, and it contains irregular various ectodermally , endodermally , and mesodermally derived tissues.
calcifications. The right fallopian tube
and ovary are surgically excised. The Papillary structures with psammoma bodies would characterize a
gross appearance of the ovary, which cystadenocarcinoma.
has been opened, is shown in the Primitive neuroepithelium in a more solid and less cystic mass would be
figure. Microscopic examination of this lesion is most likely to show which consistent with an immature teratoma.
of the following? Sarcomas of the ovary are uncommon; a rhabdomyosarcoma element
A. Mature squamous epithelium could be part of a uterine malignant mixed mullerian tumor. A
B. Papillary structures with psammoma bodies choriocarcinoma with trophoblastic cells is usually gestational in origin
C. Primitive neuroepithelium and has a hemorrhagic appearance.
D. Rhabdomyoblasts in a cellular
E. Stroma Sheets of trophoblasts and syncytial cells

42. A 23-year-old woman has had pelvic discomfort for 4 months. On pelvic D. Immature teratoma
examination, there is a large, nontender, right adnexal mass. An abdominal Immature teratomas are not cystic like mature teratomas. Tissues derived
CT scan shows the 11-cm mass to be solid and circumscribed. On surgical from multiple germ cell layers are present, as in all teratomas, but at least
removal, the mass is solid and white, with small areas of necrosis. one immature tissue element is present. Often that immature element is
Microscopically, it contains mostly primitive mesenchymal cells along with neuroectodermal tissue. The less differentiated and more numerous the
some cartilage, muscle, and foci of neuroepithelial differentiation. What is neuroepithelial elements, the higher the grade and the worse the
the most likely diagnosis? prognosis. Adjuvant chemotherapy and radiotherapy yield a high
A. Brenner tumor response rate.
B. Dysgerminoma Brenner tumors of the ovary are uncommon solid tumors that contain
C. Granulosa cell tumor epithelial nests resembling transitional cells of the urinary tract; most are
D. Immature teratoma benign.
E. Leiomyosarcoma Dysgerminomas are the female equivalent of male testicular seminomas.
F. Malignant müllerian mixed tumor Granulosa cell tumors have cells that resemble those in ovarian follicles
and may secrete estrogens.
Leiomyosarcomas are solid tumors of smooth muscle origin that are
found most often in the myometrium.
Malignant müllerian mixed tumors are typically uterine neoplasms that
have glandular and stromal elements; the malignant stromal component
can be heterologous and may resemble mesenchymal cells not ordinarily
found in the myometrium, such as cartilage.

43.A 52-year-old woman has had dull pain in the lower abdomen for the C. Endometrial hyperplasia
past 6 months and minimal vaginal bleeding on three occasions. Her last Fibromas and thecomas are sex cord–stromal tumors that may be
menstrual period was 2 years ago. Pelvic examination shows a right hormonally active and secrete estrogens that can lead to endometrial
adnexal mass, and the uterus appears normal in size. An abdominal hyperplasia or even carcinoma. Fibromas can be associated with Meigs
ultrasound scan shows an 8-cm solid mass, a small amount of ascites, and syndrome (ovarian tumor with ascites and right pleural effusion). Most of
a right pleural effusion. A total abdominal hysterectomy is performed, and these tumors also are benign and do not metastasize.
the mass is determined to be an ovarian fibrothecoma. Which of the In most cases, chronic salpingitis is related to sexually transmitted
following additional lesions is most likely to be found in the excised infections, such as gonorrhea.
specimen? A condyloma acuminatum is related to infection with human
A. Bilateral chronic salpingitis papillomavirus and is more likely to occur in younger, sexually active
B. Cervical condylomata acuminata women on external genitalia and perineum.
C. Endometrial hyperplasia A partial mole is an uncommon form of gestational trophoblastic disease
D. Metastases to the uterine serosa with a triploid karyotype and occurs only in reproductive-age women.
E. Partial mole of the uterus

44. A clinical study of women diagnosed with ovarian neoplasms reveals E. Sertoli-Leydig cell tumor
that 1 in 200 develop masculinizing signs and symptoms, including The Sertoli cell group of ovarian neoplasms mimics testicular

❗USE AT YOUR OWN RISK❗ page 10 of 59


ROBBINS REVIEW

hirsutism, acne, breast atrophy, and amenorrhea. These women are found differentiation and may produce androgens. These neoplasms tend to be
to have well-circumscribed, lobulated, firm, yellow mass lesions averaging better differentiated and act in a more benign fashion.
5 cm. Microscopically they have plump pink cells that show positive Brenner tumors are uncommon solid masses, usually act in a benign
immunohistochemical staining for inhibin. Which of the following manner, and may be associated with endometrial hyperplasia, though
neoplasms are most likely to have these features? they may not directly produce estrogenic hormones unless there are
A. Brenner tumor thecalike cells present.
B. Dysgerminoma Dysgerminomas and endometrioid carcinomas tend not to produce
C. Endometrioid carcinoma hormonal effects.
D. Granulosa-theca cell tumor Granulosa-theca cell tumors are known for association with estrogenic
E. Sertoli-Leydig cell tumor effects.

45. A 17-year-old girl missed a menstrual period, and her pregnancy test F. Fetal trisomy 16
is positive. A month later, she notes suprapubic pain and passing blood Spontaneous abortion (miscarriage) may occur in at least a third of
clots from her vagina. She passes a small amount of tissue 3 days later. pregnancies, and most occur in the first trimester. Fetal problems are the
Pathologic examination of this tissue shows products of conception. Which most likely cause for early losses, whereas maternal problems account for
of the following is the most likely cause for her pregnancy loss? most late fetal losses. Half of early abortuses have a chromosomal
A. Bifid uterus abnormality, many of which are incompatible with prolonged survival, such
B. Group B streptococcus infection as trisomy 16. If there is recurrent early pregnancy loss, a parental
C. Polycystic ovarian syndrome germline chromosomal anomaly may be suspected.
D. Preeclampsia
E. Smoking cigarettes Infections, uterine anomalies, masses such as leiomyomas, and
F. Fetal trisomy 16 toxemia are more likely to cause fetal loss later in pregnancy.
Polycystic ovarian syndrome is more likely to be a cause for infertility.
Maternal smoking is most likely to affect fetal weight, and less likely to
cause early fetal loss.

46. A 36-year-old woman has had an uneventful pregnancy for the past 37 A. Group B streptococcus
weeks. Over the past 12 hours, she has developed lower abdominal pain. Placental infections are most likely to ascend from the vagina, and they
On examination, there is suprapubic tenderness. Her temperature is 37.4° are not usually hematogenous. Preterm premature rupture of membranes
C. Pelvic examination reveals a purulent cervical discharge. The infant is may predispose to ascending infection, or it may be caused by
delivered 12 hours later. Which of the following organisms is most likely prostaglandins released from acute inflammatory cells in the infection as
responsible for her premature labor? suggested by the purulent exudate. Premature labor with delivery is likely
A. Group B streptococcus to occur over the next 24 hours.
B. Herpes simplex virus
C. Rubella virus Of the TORCH infections, the one most likely in this case is the O,
D. Toxoplasma gondii including bacteria, such as group B streptococcus, whereas Listeria
E. Treponema pallidum monocytogenes may produce more chronic inflammation.

47. A 22-year-old woman experiences sudden onset of severe lower B. Detection of human chorionic gonadotropin in serum
abdominal pain. Physical examination shows no masses, but there is Conditions predisposing to ectopic pregnancy include chronic salpingitis
severe tenderness in the right lower quadrant. A pelvic examination shows (which may be caused by gonorrhea, but a culture would be positive only
no lesions of the cervix or vagina. Bowel sounds are detected. An with acute infection), intrauterine tumors, and endometriosis. In about half
abdominal ultrasound scan shows a 4-cm focal enlargement of the of cases, there is no identifiable cause. Gestational trophoblastic disease
proximal right fallopian tube. A dilation and curettage procedure shows only associated with a triploid karyotype with partial mole developing outside
decidua from the endometrial cavity. Which of the following laboratory the uterus is rare. Candida produces cervicitis and vaginitis and is rarely
findings is most likely to be reported for this patient? invasive or extensive in immunocompetent patients. Syphilis is not likely
A. Cervical culture positive for Neisseria gonorrhoeae to produce a tubal mass with acute symptoms (a gumma is a rare finding).
B. Detection of human chorionic gonadotropin in serum
C. 69,XXY karyotype on decidual tissue cells
D. Pap smear showing pseudohyphae of Candida
E. Positive result of serologic testing for syphilis

48. A 36-year-old primigravida develops peripheral edema late in the D. Multiple infarcts
second trimester. On physical examination, her blood pressure is 155/95 Toxemia of pregnancy in this case is best classified as preeclampsia,
mm Hg. Urinalysis shows 2+ proteinuria, but no blood, glucose, or ketones. because she has hypertension, proteinuria, and edema, but no seizures.
At 36 weeks, she gives birth to a normal viable but low-birth-weight infant. The placenta tends to be small because of reduced maternal blood flow
Her blood pressure returns to normal, and she no longer has proteinuria. and uteroplacental insufficiency; infarcts and retroplacental hemorrhages
Which of the following pathologic findings is most likely to be found on can occur. Microscopically, the decidual arterioles may show acute
examination of the placenta? atherosis and fibrinoid necrosis.
A. Chorioamnionitis
B. Chronic villitis Chorioamnionitis is most often due to ascending bacterial infections and
C. Hydropic villi leads to, or follows, premature rupture of membranes.
D. Multiple infarcts A chronic villitis is characteristic of a congenital infection such as
E. Partial mole cytomegalovirus.
Placental hydrops often accompanies fetal hydrops in conditions such
as infections and fetal anemias.
In a partial mole, a fetus is present, but it is malformed and rarely live-
born.

49. A 35-year-old primigravid woman at 30 weeks' gestation develops E. Placental ischemia


worsening headaches along with a 3-kg weight gain over 1 week. This Classic features of eclampsia are defined by hypertension,edema, and
morning she had a generalized seizure. On physical examination, she is proteinuria, typically with onset in the third trimester. The addition of
afebrile, but her blood pressure is 190/115 mm Hg (it was 120/80 mm Hg seizures defines eclampsia. Primigravid women are at greater risk. There
at a prenatal visit 1 month ago). She has peripheral edema involving her is no evidence in this case that primary renal disease could cause her
head and all extremities. Fetal heart tones of 140/min and fetal movement hypertension, and the onset was sudden. Although the precise cause of

❗USE AT YOUR OWN RISK❗ page 11 of 59


ROBBINS REVIEW

are present. Laboratory studies show hemoglobin, 12.5 g/dL; hematocrit, preeclampsia/eclampsia is unknown, placental ischemia is believed to be
37.6%; MCV, 92 μm3; platelet count, 199,000/mm3; serum creatinine, 1 the underlying mechanism. This is associated with shallow placentation
mg/dL; potassium, 4.2 mmol/L; and glucose, 101 mg/dL. Urinalysis shows and incomplete conversion of decidual vessels into high volume channels
2+ proteinuria, but no hematuria, RBCs, WBCs, or casts. Which of the required to perfuse the placenta adequately.
following is the most likely underlying factor in the causation of her Untreated patients may go on to disseminated intravascular
disease? coagulation. Cushing syndrome with adrenal cortical hyperplasia could
A. Adrenal cortical hyperplasia lead to hypertension with sodium retention, but she does not have
B. Disseminated intravascular coagulation hypokalemia or hyperglycemia.
C. Gestational trophoblastic disease Gestational trophoblastic disease predisposes patients to
D. Ovarian neoplasm producing estrogen preeclampsia, but hydatidiform mole is excluded by the presence of a
E. Placental ischemia fetus, and a partial mole would be unlikely to persist into the third trimester.
F. Uncontrolled gestational diabetes Functional ovarian tumors, most commonly estrogen secreting, such as
a granulosa cell tumor or thecoma, do not produce hypertension and
proteinuria.
Gestational diabetes may increase the risk for fetal loss, but in this case
the glucose is normal.

50. A 21-year-old G2, P1 woman is in E. Human chorionic gonadotropin


the early second trimester. She has The figure shows a hydatidiform mole, or complete mole, with enlarged,
noted a small amount of vaginal grapelike villi that form the tumor mass in the endometrial cavity. These
bleeding for the past week and has trophoblastic tumors secrete large amounts of human chorionic
had marked nausea and vomiting for 3 gonadotropin (hCG). Molar pregnancies result from abnormal fertilization,
weeks. On physical examination, the with only paternal chromosomes present. Neural tube defects can be
uterus measures large for dates. An distinguished from other fetal defects (e.g., abdominal wall defects) by use
ultrasound examination shows of the acetylcholinesterase test on amniotic fluid obtained by
intrauterine contents with a amniocentesis.
“snowstorm appearance,” and no If acetylcholinesterase and maternal serum Alpha-fetoprotein are
fetus is identified. The gross appearance of tissue obtained by dilation and elevated, a neural tube defect is likely. If the acetylcholinesterase is not
curettage is shown in the figure. Which of the following substances is most detectable, another fetal defect is suggested.
likely to be greatly increased in her serum? Alpha- Fetoprotein is a marker for some germ cell tumors that contain
A. Acetylcholinesterase yolk sac elements.
B. Alpha-Fetoprotein Estrogens can be elaborated by various ovarian stromal tumors,
C. Estradiol including thecomas and granulosa cell tumors. More ominously, a
D. Human chorionic gonadotropin decrease in maternal serum estriol suggests incipient abortion.
E. Human placental lactogen Human placental lactogen is produced in small quantities in the
developing placenta, and serum levels typically are not measured.

51. A 23-year-old woman, G3, P2, has a spontaneous abortion at 15 E. 69,XXY


weeks’ gestation. The male fetus is small for gestational age and is Partial hydatidiform mole develops from triploidy (69 chromosomes). In
malformed, with syndactyly of the third and fourth digits of each hand. The contrast to a complete mole with only paternal chromosomes, in which no
placenta also is small, and shows 0.5-cm grapelike villi scattered among fetus is present, a partial mole has a fetus because maternal
morphologically normal villi. Chromosomal analysis of placental tissue is chromosomes are present. Survival of the triploid fetus to term is rare. A
most likely to show which of the following karyotypes? partial mole may contain some grapelike villi, or none. The fetus is usually
A. 45,X malformed, often with 3,4 syndactyly.
B. 46,XX A 46,XX karyotype could be present in a complete mole or a normal
C. 47,XXY female fetus.
D. 47,XY,+18 A fetus with Turner syndrome (monosomy X) has a 45,X karyotype. Most
E. 69,XXY female fetuses with loss of an X chromosome undergo spontaneous
abortion, but some survive.
Klinefelter syndrome has a 47,XXY karyotype, and male infants are live-
born, with no placental problems.
A 47,XY,+18 karyotype of trisomy 18 is associated with multiple congenital
malformations, but not with a partial mole.

52. A 23-year-old woman suddenly E. Syncytiotrophoblast


notices a bloody, brownish vaginal Choriocarcinomas are aggressive, malignant trophoblastic tumors. Some
discharge. The next day she has of these tumors can arise without evidence of pregnancy. Metastases in
shortness of breath. On physical the vaginal wall and lungs and a hemorrhagic appearance are
examination, a 3-cm, red-brown characteristic. The large pleomorphic and hyperchromatic
mass is seen on the lateral wall of syncytiotrophoblastic cells produce human chorionic gonadotropin.
the vagina. A chest radiograph Treatment with agents such as etoposide, methotrexate, actinomycin D,
shows numerous 2- to 5-cm nodules cyclophosphamide, and vincristine can often lead to remission and cure.
in both lungs. Laboratory studies
show that her serum human Amnionic cells do not give rise to neoplasms.
chorionic gonadotropin levels are markedly elevated. A biopsy specimen Rhabdomyoblasts are present in embryonal rhabdomyosarcomas of the
of the vaginal mass is obtained and shown in the figure. Chromosome vagina of young girls.
analysis of these cells shows a 46,XX karyotype. Which of the following Serous epithelium does not give rise to gestational trophoblastic
cells is most likely present in this mass lesion? disease.
A. Amnionic Smooth muscle cells give rise to leiomyomas, and rarely
B. Rhabdomyoblast leiomyosarcomas, typically arising in the uterus.
C. Serous epithelial
D. Smooth muscle
E. Syncytiotrophoblast

❗USE AT YOUR OWN RISK❗ page 12 of 59


ROBBINS REVIEW

53. A 26-year-old woman delivered a normal neonate a month ago D. Human placental lactogen
following an uncomplicated pregnancy. She now has vaginal bleeding. This is a placental site trophoblastic tumor (PSTT), the rarest of all forms
Hysteroscopy shows a nodule in the uterine fundus. Laboratory studies of gestational trophoblastic disease. The intermediate trophoblastic cells
show hCG level of 200 mIU/mL. She is given chemotherapy but the lesion do not produce large amounts of hCG, but do produce human placental
does not regress. Hysterectomy is performed. Microscopic examination of lactogen (hPL). Most of these lesions are treated surgically and are
the nodule shows intermediate trophoblast cells. Immunostaining for which controlled, but some recur and respond minimally to chemotherapy and
of the following proteins is most likely to yield positive results in this nodule? radiation.
A. Alpha-Fetoprotein Alpha-Fetoprotein is produced by some testicular neoplasms and by
B. Chromogranin hepatocellular carcinomas.
C. Desmin Chromogranin is a marker for neuroendocrine tumors.
D. Human placental lactogen Desmin is likely to be seen in tumors of mesenchymal origin.
E. Neuron-specific enolase Neuron-specific enolase can be seen with tumors of neural and
neuroendocrine differentiation.

BREAST

QUESTION RATIONALE

1. A 21-year-old woman delivered a D. Prolactin


normal term infant a week ago and is This is accessory breast tissue with lactational change. Prolactin secretion
now nursing the infant. She now from the adenohypophysis increases in postpartum women to support milk
notes a lump in her right axilla that production in breast lobules.
has increased in size over the past
week. On physical examination Oxytocin released from the posterior pituitary stimulates myoepithelial
there is a rubbery, mobile, 1.5-cm cells to contract during nursing. The remaining hormones listed do not
mass beneath the skin at the right have a direct effect upon breast tissue. The presence of the breast tissue
anterior axillary line. The mass is in the axilla represents accessory breast tissue, and can explain the origins
excised and the microscopic of breast cancer in women following simple mastectomy.
appearance is shown in the figure.
Which of the following hormones most likely produced the greatest effect
upon this tissue?
A. Cortisol
B. Growth hormone
C. Oxytocin
D. Prolactin
E. Testosterone

2. A 24-year-old woman is breastfeeding 3 weeks after giving birth to a D. Staphylococcus aureus


normal term infant. She notices fissures in the skin around her left nipple. Staphylococcal acute mastitis typically produces localized abscesses,
Over the next 3 days, a 5-cm region near the nipple becomes whereas streptococcal infections tend to spread throughout the breast,
erythematous and tender. Purulent exudate from a small abscess drains because streptococci often produce streptolysins. Acute mastitis can be
through a fissure. Which of the following organisms is most likely to be associated with the first few months of breastfeeding.
cultured from the exudate?
A. Candida albicans Candida may cause some local skin irritation, but is likely to become
B. Lactobacillus acidophilus invasive only in immunosuppressed patients.
C. Listeria monocytogenes Lactobacillus acidophilus is the organism used to produce fermented
D. Staphylococcus aureus nonhuman milk.
E. Viridans streptococci Listeriosis can be spread by contaminated food, including milk products,
not by human milk.

3. A 30-year-old woman sustained a traumatic blow to her right breast. B. Fat necrosis
Initially, there was a 3-cm contusion beneath the skin that resolved within Fat necrosis is typically caused by trauma to the breast. The damaged,
3 weeks, but she then felt a firm, painless lump that persisted below the necrotic fat is phagocytosed by macrophages, which become lipid laden.
site of the bruise 1 month later. What is the most likely diagnosis for this The lesion resolves as a collagenous scar within weeks to months. The
lump? firm scar can mammographically and grossly resemble a carcinoma.
A. Abscess
B. Fat necrosis An abscess may form a palpable but painful mass lesion, and often from
C. Fibroadenoma Staphylococcus aureus infection when localized.
D. Inflammatory carcinoma A fibroadenoma is a neoplasm, and tumors are not induced by trauma.
E. Sclerosing adenosis Inflammatory carcinoma refers to dermal lymphatic invasion by an
underlying breast carcinoma, giving a rough red-to orange appearance to
the skin.
Sclerosing adenosis is a feature of fibrocystic changes, a common cause
of nontraumatic breast lumps.

4. A study of mammographic findings on women of reproductive years is A. Apocrine metaplasia


performed. The study identifies mammograms showing 1- to 5-cm cysts Non-proliferative cysts are quite common in the breast. When they are
with focal microcalcifications and surrounding densities. Subsequent fine- fluid-filled, they are unlikely to contain proliferative elements. The cells
needle aspiration yielded turbid fluid with few cells. Which of the following lining these cysts may be flattened cuboidal to atrophic, but often have
microscopic changes is most likely to be present in these lesions? abundant pink cyto- plasm resembling apocrine change.
A. Apocrine metaplasia Microcalcifications may be seen in both benign and malignant breast
B. Ductal carcinoma in situ lesions, but in the case of cysts they represent calcified secretions. If
C. Fat necrosis excised, the intact cysts may have a blue to brown color.

❗USE AT YOUR OWN RISK❗ page 13 of 59


ROBBINS REVIEW

D. Papillomatosis Ductal carcinomas are likely to be solid lesions.


E. Sclerosing adenosis Fat necrosis may contain many macrophages, but also connective tissue,
producing a firm lesion.
Papillomatosis is a proliferative feature in fibrocystic changes that tends
to form a solid lesion.
Sclerosing adenosis produces a firm, fibrous lesion.

5. A 27-year-old woman feels a lump in her right breast. She has normal B. Fibrocystic changes
menstrual cycles, she is G3, P3, and her last child was born 5 years ago. Nonproliferative (fibrocystic) changes account for the largest category of
On examination a 2-cm, irregular, firm area is palpated beneath the lateral breast lumps. These lesions are probably related to cyclic breast changes
edge of the areola. This lumpy area is not painful and is movable. There that occur during the men- strual cycle. In about 30% of cases of breast
are no lesions of the overlying skin and no axillary lymphadenopathy. A lumps, no specific pathologic diagnosis can be made. Fibrocystic changes
biopsy specimen shows microscopic evidence of an increased number of include ductal proliferation, ductal dilation (sometimes with apocrine
dilated ducts surrounded by fibrous connective tissue. Fluid-filled ducts metaplasia), and fibrosis.
with apocrine metaplasia also are present. What is the most likely A fibroadenoma is a discrete mass formed by a proliferation of fibrous
diagnosis? stroma with compressed ductules.
A. Fibroadenoma Carcinomas have proliferations of atypical neoplastic cells that fill ducts
B. Fibrocystic changes and can invade stroma.
C. Infiltrating ductal carcinoma Inspissated duct secretions may produce duct ectasia with a surrounding
D. Mammary duct ectasia lymphoplasmacytic infiltrate.
E. Traumatic fat necrosis Trauma with subsequent fat necrosis may produce a localized, firm
lesion that mimics carcinoma, but microscopically shows macrophages
and neutrophils sur- rounding necrotic adipocytes, and healing leaves a
fibrous scar.

6. A 47-year-old woman has a routine health examination. There are no B. Continued screening for breast cancer
remarkable findings except for a barely palpable mass in the right breast. Fibrocystic changes without epithelial hyperplasia do not suggest a
A mammogram shows an irregular, 1.5- cm area of density with scattered significantly increased risk of breast cancer. Moderate to florid hyperplasia
microcalcifications in the upper outer quadrant. A biopsy specimen from increases the risk twofold, and atypical ductal or lobular hyperplasias
this area is obtained and microscopically shows ductal hyperplasia. Which increase the risk fivefold. The risk in this patient is not great enough to
of the following is the most appropriate option for follow-up of this patient? suggest radical or simple mastectomy at this time, but follow-up is needed.
A. Cessation of smoking cigarettes Breast cancers are not associated with tobacco use.
B. Continued screening for breast cancer
C. Performing a simple mastectomy The BRCA1 gene accounts for a small percentage of breast cancers,
D. Testing for the BRCA1 oncogene primarily in families in which cancer onset occurs at a young age, and
E. Prescribing broad-spectrum antibiotic therapy genetic testing of all persons at risk for breast cancer is not warranted.
These proliferative changes are not the result of infection.

7. A 34-year-old woman has noticed a bloody discharge from the nipple of C. Intraductal papilloma
her left breast for the past 3 days. On physical examination, the skin of Intraductal papillomas are usually solitary and smaller than 1 cm. They are
the breasts appears normal, and no masses are palpable. There is no located in large lactiferous sinuses or large ducts, and have a tendency to
axillary lymphadenopathy. She has regular menstrual cycles and is using bleed, though they are benign.
oral contraceptives. Excisional biopsy is most likely to show which of the Abscesses complicating mastitis organize with a fibrous wall.
following lesions in her left breast? Fibroadenomas contain ducts with stroma and are not highly vascular;
A. Acute mastitis these lesions are not located in ducts.
B. Fibroadenoma Phyllodes tumors also arise from intralobular stroma and can be
C. Intraductal papilloma malignant, but they do not invade ducts to cause bleeding.
D. Phyllodes tumor Sclerosing adenosis, a lesion occurring with fibrocystic changes, has
E. Sclerosing adenosis abundant collagen, not vascularity.

8. A 57-year-old man has developed bilateral breast enlargement over the E. Micronodular cirrhosis
past 2 years. On physical examination, the enlargement is symmetric and Micronodular cirrhosis is most often a consequence of chronic alcoholism
is not painful to palpation. There are no masses. He is not obese and is and impairs hepatic estrogen metabolism, which can lead to bilateral
not taking any medications. Which of the following underlying conditions gynecomastia.
best accounts for his findings? ACTH-secreting pituitary adenomas cause truncal obesity because of
A. ACTH-secreting pituitary adenoma Cushing syndrome.
B. Choriocarcinoma of the testis Choriocarcinomas of the testis produce human chorionic gonadotropin
C. Chronic glomerulonephritis and may cause some breast enlargement. Choriocarcinomas are highly
D. Diabetes mellitus malignant neoplasms that would not remain indolent for 2 years.
E. Micronodular cirrhosis Chronic renal failure is unlikely to have this consequence.
F. Rheumatoid arthritis Diabetes mellitus slightly increases the risk for breast cancer in women.
Rheumatoid nodules can appear in various locations along with
rheumatoid arthritis, but they rarely occur in the breast and are unlikely
to be bilateral.

9. A 58-year-old woman sees her naturopathic health care provider for a A. Atypical cytologic changes
routine health examination. There are no remarkable findings on physical Atypical lobular hyperplasia and atypical ductal hyperplasia increase the
examination. A screening mammogram shows a 0.5-cm irregular area of risk of breast cancer fivefold; the risk affects both breasts and is higher in
increased density with scattered microcalcifications in the upper outer premenopausal women or women who have a family history of breast
quadrant of the left breast. Excisional biopsy shows atypical lobular cancer.
hyperplasia. She has been on postmenopausal estrogenprogesterone
therapy for the past 10 years. She has smoked 1 pack of cigarettes per Smoking and exogenous estrogen therapy are not well-established risk
day for the past 35 years. Which of the following is the most significant risk factors for breast cancer.
factor for the development of lobular carcinoma in patients with such The BRCA1 mutation accounts for about 10% to 20% of familial breast
lesions? carcinomas and only a few percent of all breast cancers.

❗USE AT YOUR OWN RISK❗ page 14 of 59


ROBBINS REVIEW

A. Atypical cytologic changes


B. History of smoking
C. Hormone replacement therapy
D. Postmenopausal age
E. Underlying BRCA1 gene mutation

10. A 25-year-old Jewish woman A. BRCA1


sees her physician after finding a The biopsy specimen shows an invasive breast cancer. Given the young
lump in her right breast. On physical age of the patient and the strong family history of breast cancer, it is
examination, a 2-cm, firm, non reasonable to assume that she has inherited an altered gene that
movable mass is palpated in the predisposes to breast cancer. There are two known breast cancer
upper outer quadrant. No overlying susceptibility genes: BRCA1 and BRCA2. Both are cancer suppressor
skin lesions and no axillary lymph- genes. Specific mutations of BRCA1 are common in some ethnic groups,
adenopathy are present. The figure such as Ashkenazi Jews.
shows an excisional biopsy
specimen. The family history Estrogen receptors are expressed in 50% to 75% of breast cancers.
indicates that the patient’s mother, Their presence bodes well for therapy with hormone receptor antagonists.
maternal aunt, and maternal grandmother have had similar lesions. Her There is no known relationship between the structure of the estrogen
18-year-old sister has asked a physician to determine whether she is receptor gene and susceptibility to breast cancer. Likewise, presence of
genetically at risk of developing a similar disease. A mutated gene progesterone receptors in the cancer cells indicates potential response
encoding for which of the following is most likely to be found in her sister? to hormonal therapy, not risk for breast cancer.
A. BRCA1 HER2/neu is a growth factor receptor gene that is amplified in certain
B. Estrogen receptor (ER) breast cancers and is a marker of poor prognosis, not susceptibility. There
C. HER2/neu is alteration of TP53 in many cancers, typically acquired and not familial,
D. TP53 including breast carcinomas, but it does not have predictive value for risk.
E. Progesterone receptor (PR) Inheritance of RB1 mutations increases the risk for retinoblastoma and
F. RB1 osteosarcomas, but not breast carcinomas.

11. A clinical study is performed on postmenopausal women living in Paris, C. First-degree relative with breast cancer
France, who are between the ages of 45 and 70 years. All have been The relative risk of breast cancer increases with various factors, but family
diagnosed with infiltrating ductal carcinoma positive for estrogen receptor history is one of the strongest. A history of bilateral breast disease and
(ER) and progesterone receptor (PR), but negative for HER2 expression, earlier age of onset of cancer increase the risk. The earlier age of onset
which has been confirmed by biopsy and microscopic examination of increases the risk of identifying a BRCA1 or BRCA2 mutation.
tissue. None has the BRCA1 or BRCA2 mutation. Which of the following
is most likely to indicate the highest relative risk of developing the A longer reproductive life, with early menarche (<11 years old) and late
carcinomas seen in this group of women? menopause (>55 years old), and nulliparity increase the risk of breast
A. Age at menarche older than 16 years cancer, probably because of increased estrogen exposure. “Soft” risk
B. Age at menopause younger than 45 years factors include exogenous estrogens and obesity. Mastitis does not affect
C. First-degree relative with breast cancer the risk of breast cancer.
D. Multiparity
E. Prior diagnosis of mastitis

12. A 54-year-old woman feels a E. Trastuzumab


lump in her left breast. On The expression of HER2/neu, an epidermal growth factor receptor,
examination there is a firm, suggests that biotherapy with trastuzumab may have some effectiveness.
irregular mass in the lower outer Drug names with the suffix -mab are monoclonal antibodies that target a
quadrant. A mammogram shows a specific biochemical component of cells. This form of biotherapy is useful
2-cm density with focal because normal breast cells do not express HER2/neu. Doxorubicin is a
microcalcifications. Excisional standard chemotherapeutic agent that is part of various multiagent
biopsy shows intraductal and protocols.
invasive carcinoma.
Immunohistochemical staining is Hydroxyurea is a cycle-acting agent that is not useful in breast cancer.
negative for estrogen receptors Letrozole is an aromatase inhibitor that is useful for treating ER-positive
(ER). FISH analysis (green = HER2; red = chromosome 17 centromere) breast cancers.
shows the findings in the figure. When combined with doxorubicin, which Raloxifene is a selective estrogen receptor modulator (SERM) that
of the following drugs is most likely to be useful in treating this patient? reduces risk for breast cancer and reduces osteoporosis.
A. Hydroxyurea Tamoxifen is an anti-estrogenic compound that has effectiveness in the
B. Letrozole treatment of breast cancers positive for ER/PR.
C. Raloxifene
D. Tamoxifen
E. Trastuzumab

13. A 66-year-old nulliparous woman received hormone replacement C. Very low likelihood of recurrence
therapy for 7 years following menopause at age 53 years. Her BMI is 33. This luminal A form comprises over half of all invasive breast cancers, and
She now undergoes screening mammography, and an irregular mass is it tends to be low grade, and lack BRCA1, BRCA2, TP53, and CHEK2
identified in the right breast. An excisional biopsy yields a 1.5-cm mass familial gene mutations. It is often responsive to anti-estrogen hormonal
that microscopically has invasive cells that are positive for estrogen therapy, although surgery alone can be curative. Even with metastases,
receptor but negative for HER2, with low proliferation markers and mutated the course is prolonged. This patient has multiple risks for breast cancer,
PIK3CA gene. Following surgical removal of the mass, which of the including nulliparity, obesity, and hormone replacement therapy. Breast
following clinical courses will most likely occur over the next year? cancer at her age is less likely to be familial. Trastuzumab is useful for
A. Detection of cancer in the left breast HER2-positive breast cancers.
B. Need for chemotherapy
C. Very low likelihood of recurrence
D. Need for treatment with trastuzumab
E. Occurrence of widespread metastases

❗USE AT YOUR OWN RISK❗ page 15 of 59


ROBBINS REVIEW

14. A 63-year-old woman has a B. DCIS


screening mammogram that An intraductal carcinoma, or ductal carcinoma in situ (DCIS), may not
shows an irregular density with produce a palpable mass. The figure shows ducts that contain large,
microcalcifications. On physical atypical cells in a cribriform pattern. If grossly soft, white material is
examination, there are no lesions extruded from small ducts when pressure is applied, then there is necrosis
of the overlying skin, and there is of the neoplastic cells in the ducts (that leads to dystrophic calcification),
no axillary lymphadenopathy. An and the term comedocarcinoma is applicable. Intraductal carcinomas
excisional biopsy specimen represent about one fourth of all breast cancers. If not excised, such
shows no mass on sectioning. lesions become invasive. Intraductal carcinoma has several other
Microscopic examination shows histologic patterns, including non comedo DCIS and Paget disease of the
the findings in the figure. What is nipple, in which extension of the malignant cells to the skin of the nipple
the most likely diagnosis? and areola produces the appearance of a seborrheic dermatitis.
A. Colloid carcinoma
B. Ductal carcinoma in situ Colloid carcinomas occur about as frequently as medullary carcinomas,
C. Infiltrating ductal carcinoma but they are often positive for estrogen receptor and progesterone
D. Infiltrating lobular carcinoma receptor, and the prognosis is better than average.
E. Medullary carcinoma Infiltrating ductal carcinomas tend to produce irregular, firm, mass
F. Papillary carcinoma lesions because they are more invasive.
Infiltrating lobular carcinomas can have a diffuse pattern without significant
mass effect.
Medullary carcinomas tend to be large masses; microscopically, they
have nests of large cells with a surrounding lymphoid infiltrate.
True papillary carcinomas are rare, although a papillary component may
be present in other types of breast carcinoma.

15. A 48-year-old woman has E. Paget disease of the breast


noticed a red, scaly area of skin on Paget cells are large cells that have clear, mucinous cytoplasm and
her left breast that has grown infiltrate the skin overlying the breast. They are malignant and extend to
slightly larger over the past 4 the skin from an underlying breast carcinoma, which may be occult, so that
months. On physical examination, Paget disease may be the first sign of malignancy.
there is a 1-cm area of eczematous
skin adjacent to the areola. The Apocrine metaplasia affects the cells lining the cystically dilated ducts in
figure shows the microscopic fibrocystic change.
appearance of the skin biopsy The macrophages in fat necrosis do not infiltrate the skin and do not have
specimen. What is the most likely the atypical nuclei seen in the figure.
diagnosis? Inflammatory carcinoma does not refer to a specific histologic type of
A. Apocrine metaplasia breast cancer; rather, it describes the involvement of dermal lymphatics by
B. Fat necrosis infiltrating carcinoma, and there may be thickening and a reddish-orange
C. Inflammatory carcinoma appearance to the skin.
D. Lobular carcinoma in situ In lobular carcinoma in situ, terminal ducts or acini within the breast are
E. Paget disease of the breast filled with neoplastic cells.

16. A 54-year-old woman noticed a B. LCIS


lump in her right breast. On Among primary malignancies of the breast, lobular carcinoma in situ
examination, she has an ill defined, (LCIS) is most likely to be bilateral. LCIS may precede invasive lesions by
1-cm mass in the upper outer several years.
quadrant. The mass is cystic on
ultrasound. An excision is done, and Lobular carcinoma may be mixed with ductal carcinoma, and it may be
microscopically the mass shows difficult to distinguish them histologically. The other neoplasms listed are
predominantly fibrocystic changes, less likely to be bilateral and more likely to produce a single mass effect.
but the lesion shown in the figure
also is present. Fine-needle
aspirates of both breasts reveal
additional foci of similar cells. Which of the following breast lesions is most
likely to produce these findings?
A. Infiltrating ductal carcinoma
B. Lobular carcinoma in situ
C. Malignant phyllodes tumor
D. Medullary carcinoma
E. Mucinous (colloid) carcinoma

17. A 49-year-old woman felt a A. Axillary lymphadenopathy


lump in her left breast 1 week ago. This irregular, infiltrative mass is an infiltrating (invasive) ductal carcinoma,
On examination, a firm, irregular the most common form of breast cancer. Breast carcinomas are most likely
mass is palpable in the upper outer to metastasize to regional lymph nodes. By the time a breast cancer
quadrant of her left breast. There becomes palpable, lymph node metastases are present in
are no overlying skin lesions. The more than 50% of patients. A bloody discharge from the nipple most often
gross appearance of the excisional results from an intraductal papilloma. Pain in the chest wall could be bone
biopsy specimen is shown in the metastases, but less likely local invasion, and there is a margin of adipose
figure. Which of the follow- ing tissue around the mass in the specimen shown. Breast cancers are
additional findings is she most associated in rare cases with ectopic corticotropin secretion or Cushing
likely to have on physical examination? syndrome. Lobular carcinomas are more often bilateral, but they are less
A. Axillary lymphadenopathy common than infiltrating ductal carcinomas.
B. Bloody discharge from the nipple
C. Chest wall tenderness

❗USE AT YOUR OWN RISK❗ page 16 of 59


ROBBINS REVIEW

D. Cushingoid facies
E. Mass in the opposite breast

18. A 57-year-old woman has felt A. Lobular Carcinoma


a lump in her left breast for 4 In this lobular carcinoma, note the pleomorphic cells infiltrating single file
months. She has had new onset through the stroma. The metastatic profile of this cancer includes the
headaches associated with carcinomatous meningitis suggested by her leptomeningeal enhancement,
nausea for the past month. Her as well as intra-abdominal metastases. E-cadherin is an adhesion
physician palpates a firm but molecule that serves as a tumor suppressor, and its loss characterizes
irregular 2-cm mass in her left another infiltrating carcinoma—signet ring carcinoma of the stomach.
breast. CT imaging of her brain
shows leptomeningeal Medullary carcinomas are solid masses of cells with little desmoplasia,
enhancement. A lumpectomy with but prominent lymphoid infiltrates.
axillary node sampling is Metaplastic carcinomas are rare in humans and have a component
performed. Immunohistochemical staining of these cells shows absence resembling another tissue, such as squamous carcinomas.
of E-cadherin and HER2, but presence of es- trogen receptor (ER) and In the setting of a malignant breast mass, any brain lesion is a suspected
progesterone receptor (PR). An H and E stained section is shown in the metastatic lesion; although glioblastoma is capable of extracranial
figure. No nodal metastases are present. Which of the following is the most metastases, this is rare, and there should be a bulky cerebral mass
likely diagnosis? present.
A. Lobular carcinoma Phyllodes tumors can be malignant, with a stromal component, but these
B. Medullary carcinoma are typically bulky masses, and there is a microscopic leaflike pattern of
C. Metaplastic carcinoma cystic spaces lined by epithelium.
D. Metastatic glioblastoma
E. Phyllodes tumor

19. A 39-year-old woman has noticed an enlarging mass in her left breast D. Medullary Carcinoma
for the past 2 years. The physician palpates a 4-cm firm mass. Following Medullary carcinomas account for about 1% to 5% of all breast
biopsy, a simple mastectomy is performed with axillary lymph node carcinomas. They tend to occur in women at younger ages than do most
sampling. On gross sectioning, the mass has a soft, tan, fleshy surface. other breast cancers. Despite poor prognostic indicators (such as absence
Histologically, the mass is composed of large cells with vesicular nuclei of HER2, ER, and PR), medullary carcinomas have a better prognosis than
and prominent nucleoli. There is a marked lymphocytic infiltrate within the most other breast cancers. Perhaps the infiltrating lymphocytes are a
tumor, and the tumor has a discrete, non-infiltrative border. No axillary helpful immune response.
node metastases are present. The tumor cells are triple negative, for
HER2, estrogen receptor (ER), and progesterone receptor (PR). What is Colloid carcinomas occur about as frequently as medullary carcinomas,
the most likely diagnosis? but they are often positive for ER, and the prognosis is better than average.
A. Colloid carcinoma Infiltrating ductal and infiltrating lobular carcinomas tend not to
B. Infiltrating ductal carcinoma produce large, localized lesions because they are more invasive, and they
C. Infiltrating lobular carcinoma lack a distinct lymphoid infiltrate.
D. Medullary carcinoma True papillary carcinomas are quite rare, although other types of breast
E. Papillary carcinoma carcinoma may have a papillary component.
F. Phyllodes tumor The phyllodes tumor is typically large, but it has stromal and glandular
components.

20. An epidemiologic study is conducted with male subjects who have A. Age older than 70 years
been diagnosed with breast carcinoma. Their demographic data, medical Male breast cancers are rare, and they occur primarily among the elderly.
histories, family histories, and laboratory data are examined to identify Additional risk factors include first degree relatives with breast cancer,
factors that increase the risk of cancer. Which of the following factors is decreased testicular function, exposure to exogenous estrogens, infertility,
most likely to be associated with the greatest number of male breast obesity, prior benign breast disease, exposure to ionizing radiation, and
carcinomas? residency in Western countries. Of cases in men, 4% to 14% are attributed
A. Age older than 70 years to germline BRCA2 mutations, less frequently for BRCA1, and ATM
B. Asian ancestry mutations in less than 1%.
C. ATM gene mutation
D. Chronic alcoholism Gynecomastia does not seem to be a risk factor.
E. Gynecomastia

21. A study of women with breast carcinoma is done to determine the D. Higher response to therapy
presence and amount of estrogen receptor (ER) and progesterone The estrogen receptor and progesterone receptor (ER and PR) status
receptor (PR) in the carcinoma cells. Large amounts of ER and PR are helps predict whether chemotherapy with antiestrogen compounds such
found in the carcinoma cells of some patients. These receptors are not as tamoxifen would be effective; however, the correlation is not perfect.
present in the cells of other patients. The patients with positivity for ER and ER and PR do not affect immunogenicity and are not targets for
PR are likely to exhibit which of the following traits? immunotherapy. In contrast, immunotherapy targeted to the
A. Greater immunogenicity overexpressed HER2/neu gene is being used. The overall prognosis may
B. Greater likelihood of metastases be predicted from several factors, including histologic type, histologic
C. Greater risk of familial breast cancer grade, presence of metastases, degree of aneuploidy, and tumor stage. A
D. Higher response to therapy family history and the presence of specific mutations such as BRCA1 or
E. Higher tumor stage BRCA2 correlate with familial risk of breast cancer.
F. Higher tumor grade

22. A 26-year-old woman has felt a breast lump for the past month and is B. Medullary carcinoma
worried because she has a family history of early onset and bilateral breast Patients with the BRCA1 gene mutation have a high incidence of
cancers. On physical examination, there is a firm, 2-cm mass in the upper carcinomas with medullary features that are poorly differentiated and triple
outer quadrant of her left breast. A biopsy is done, and the specimen negative (do not express the HER2/neu protein, and are negative for
microscopically shows carcinoma. Genetic analysis shows that she is a estrogen and progesterone receptors).
carrier of the BRCA1 gene mutation, as are her mother and sister. Which
of the following histologic types of breast carcinoma has the highest

❗USE AT YOUR OWN RISK❗ page 17 of 59


ROBBINS REVIEW

incidence in families such as hers?


A. Lobular carcinoma
B. Medullary carcinoma
C. Metaplastic carcinoma
D. Papillary carcinoma
E. Tubular carcinoma

23. A 79-year-old, previously healthy woman feels a lump in her right G. Presence of lymph node metastases
breast. The physician palpates a 2-cm firm mass in the upper outer Many factors affect the course of breast cancer. The involvement of axillary
quadrant. Nontender right axillary lymphadenopathy is present. A lymph nodes is the most important prognostic factor listed. If there is no
lumpectomy with axillary lymph node dissection is performed. Microscopic spread to axillary nodes, the 10-year survival rate is almost 80%. It
examination shows that the mass is an infiltrating ductal carcinoma. Two decreases to 35% to 40% with 1 to 3 positive nodes, and to 15% with more
of 10 axillary nodes contain metastases. Flow cytometry on the carcinoma than 10 positive nodes.
cells shows a small aneuploid peak and high S Phase. Increasing age is a risk for breast cancer, but age alone does not indicate
Immunohistochemical tests show that the tumor cells are positive for a prognosis, and treatment of cancers in the elderly can be successful.
estrogen and progesterone receptor (ER/PR), negative for HER2/neu An increased DNA content with aneuploidy and a high S-phase suggests
expression, and positive for cathepsin D expression. What is the most a worse prognosis, but staging is still a more important determinant of
important prognostic factor for this patient? prognosis.
A. Age at diagnosis Estrogen receptor positivity suggests a better response to hormonal
B. DNA content in the carcinoma manipulation of the tumor, whereas expression of HER2/ neu suggests
C. Estrogen receptor positivity responsiveness to biotherapy with the monoclonal antibody trastuzumab.
D. Expression of stromal proteases in the carcinoma Some histologic types of breast cancer have a better prognosis than
E. Histologic subtype of carcinoma others, but staging is a more important factor than histologic type.
F. Lack of HER2/neu expression in the carcinoma The expression of stromal proteases, such as cathepsin D, predicts
G. Presence of lymph node metastases metastases, but in this case “the horse is out of the barn,” and metastasis
has occurred.

24. A study of gene expression profiling involving breast biopsies showing A. Chemotherapy
invasive carcinoma of no specific type (NST) is performed. A subset of This is the basal-like subset of NST breast cancers that is triple negative
these cases, comprising about 15% of all cases, has the following for the usual immunohistochemical markers. Hence, lack of ER positivity
characteristics: estrogen receptor (ER) and progesterone receptor (PR) predicts that antihormonal therapy with tamoxifen will not be of benefit, and
negative, HER2/neu nega- tive, basal keratin positive, flow cytometry lack of HER2/neu indicates that trastuzumab will be ineffective. The basal-
showing aneuploidy and high proliferation rate, and association with like cancers are highly aggressive and tend to metastasize early, so
BRCA1 mutations. Which of the following therapies is most likely to be containment with surgery or radiation is unlikely. However, some of them
effective in women with this subset of NST breast cancer? are cured by chemotherapy. This emphasizes the importance of gene
A. Chemotherapy expression profiling, so that treatment is individualized to each cancer
B. Radiation patient for the best chance of success.
C. Surgery alone
D. Tamoxifen
E. Trastuzumab

25. A 51-year-old woman has noticed an area of swelling with tenderness D. Infiltrating ductal carcinoma
in her right breast that has worsened over the past 2 months. On physical The gross appearance of the skin is consistent with invasion of dermal
examination, the 7-cm area of erythematous skin is tender with a rough, lymphatics by carcinoma—the so-called inflammatory carcinoma, which is
firm surface resembling an orange peel. There is swelling of the right not a histologic type of breast cancer, but a descriptive phrase based upon
breast, nipple retraction, and right axillary nontender lymphadenopathy. the gross appearance (peau d’orange) resembling an inflammatory
Excisional biopsy of skin and breast is most likely to show which of the process. Nipple retraction and non-tender axillary lymphadenopathy also
following lesions? suggest invasive ductal carcinoma.
A. Acute mastitis
B. Atypical epithelial hyperplasia Atypical ductal hyperplasia may increase the risk of carcinoma, but it is
C. Fat necrosis not capable of invasion and does not produce visible surface skin changes.
D. Infiltrating ductal carcinoma Acute mastitis may produce pain and swelling, but it is more likely to
E. Sclerosing adenosis occur in association with breastfeeding, and as an inflammatory process
would be more likely to produce painful lymphadenopathy.
Fat necrosis on palpation can mimic that of carcinoma, but the skin is not
involved.
Sclerosing adenosis is a feature of benign fibrocystic changes producing
breast lumps, but it has no skin involvement.

❗USE AT YOUR OWN RISK❗ page 18 of 59


ROBBINS REVIEW

26. A 26-year-old woman has noticed a lump in B. Fibroadenoma


her right breast for the past year. A 2-cm, firm, Grossly and radiographically, this patient has a discrete mass that in a
circumscribed, movable mass is palpated in the woman her age is most likely a fibroadenoma.
lower outer quadrant. The figure shows the
excised mass (A) and the mammogram (B). Fat necrosis and infiltrating cancers are masses with irregular outlines.
What is the most likely diagnosis? Fibrocystic changes are generally irregular lesions, not discrete masses.
A. Fat necrosis Mastitis has a more diffuse involvement, without mass effect.
B. Fibroadenoma Phyllodes tumors are typically much larger and are far less common.
C. Fibrocystic changes
D. Infiltrating ductal carcinoma
E. Mastitis
F. Phyllodes tumor

27. A 27-year-old woman in the third trimester of her third pregnancy A. Fibroadenomas
discovers a lump in her left breast. On physical examination, a 2-cm, Fibroadenomas are common and may enlarge during pregnancy or late in
discrete, freely movable mass beneath the nipple is palpable. After the each menstrual cycle.
birth of a term infant, the mass appears to decrease in size. The infant is
breastfed without difficulty. What is the most likely diagnosis? Most intraductal papillomas are smaller than 1 cm and are not influenced
A. Fibroadenoma by hormonal changes.
B. Intraductal papilloma Lobular carcinoma in situ is typically an ill-defined lesion without a mass
C. Lobular carcinoma in situ effect.
D. Medullary carcinoma Medullary carcinomas tend to be large; they account for only about 1%
E. Phyllodes tumor of all breast carcinomas.
Phyllodes tumors are uncommon and tend to be larger than 4 cm.

28. A 24-year-old woman notes a lump in her right breast for the past A. Biopsy to obtain tissue from the lesion
month. She is concerned because her sister was diagnosed with a poorly Her age would suggest the lesion is probably benign, and even
differentiated “triple negative” breast cancer at age 31. Ultrasonography of fibroadenomas and fibrocystic changes can have calcifications. The
the breast shows a solid mass. Fine needle aspiration is attempted but no fibrous component of a fibroadenoma or fibrocystic changes can make it
diagnostic cells are obtained. Mammography is performed and there is a difficult to aspirate cells from them. However, the family history and the
single 1-cm density with small clustered calcifications in the right breast mammographic appearance of small clustered calcifications are
but no lesions of the opposite breast. Which of the following is the best concerning for carcinoma. A delay in diagnosis and treatment of breast
course of action for this patient? cancer decreases survival. Although BRCA1 mutations are associated
A. Biopsy to obtain tissue from the lesion with HER2 and ER- and PR-negative breast cancers, the lesion must still
B. Continued monthly breast self examination be diagnosed. Based upon the histologic findings and molecular markers,
C. Genetic testing for BRCA1 mutations a treatment plan can then be instituted that may include additional studies
D. Hormonal therapy with tamoxifen and pharmacologic therapies.
E. Radiologic imaging to detect metastases

29.A 48-year-old woman has felt a D. Phyllodes tumor


poorly defined lump in her right breast Phyllodes tumors, although grossly and microscopically similar to
for the past year. On examination, she fibroadenomas, occur at an older age, are larger, and are more cellular
has a nontender, firm, 6-cm mass in the than fibroadenomas; they can recur locally following excision, but rarely
upper inner quadrant of her right breast. metastasize. The figure shows cellular stroma protruding into spaces lined
There are no lesions of the overlying by a single layer of cuboidal epithelium.
skin and no axillary lymphadenopathy. A
biopsy is performed, and mi- croscopic In contrast, fibrocystic changes can produce a breast lump, but usually
examination of the specimen shows the not as large as 6 cm, and without firm areas of cellular stroma.
findings in the figure. The mass is A lobular carcinoma has malignant appearing epithelial cells in clus- ters
excised with a wide margin, but recurs 1 year later. After further excision, and rows and may not even produce a significant mass effect.
the lesion does not recur. What is the most likely diagnosis? Tubular carcinomas of the breast are uncommon, most are less than 1
A. Fibroadenoma cm in diameter, and most have small tubular structures in a noncellular
B. Fibrocystic changes stroma.
C. Lobular carcinoma
D. Phyllodes tumor
E. Tubular carcinoma

❗USE AT YOUR OWN RISK❗ page 19 of 59


ROBBINS REVIEW

ENDOCRINE SYSTEM

QUESTION RATIONALE

A 25-year-old woman has noted breast secretions for the past month. She D. Infertility
is not breastfeeding and has never been Pregnant. She has not !"#$%&'()#*%+,%"-,*#"-,&#**#),'.%),#'.-",.#"*#)-,+-&"-'()/,0('1('%"2,
menstruated for the past 5 months. Physical examination yields no
abnormal findings. MRI of the brain shows a 0.7-cm mass within the sella %3-)#*%+4,5),%33('(#),'#,.-",/%$%&'#"".-%,%)3,()6-"'($('27,'.(+,0%'(-)',%$+#,
turcica. Which of the following additional complications is she most likely *%2, .%8-, 3-&"-%+-3, $(9(3#7, %)3, .-", %*-)#"".-%, (+, +-&#)3%"2, '#, '.-,
to have?
-:&-++(8-, 0"#$%&'(), +-&"-'(#)4, ;(&"#%3-)#*%+, *(/.', )#', .%8-, 0"-++1"-,
A. Acromegaly -66-&'+, #), +1""#1)3()/, +'"1&'1"-+, +1&., %+, #0'(&, &.(%+*7, 91', '.-2, &%), 9-,
B. Cushing disease
C. Hyperthyroidism 3(+&#8-"-3,9-&%1+-,#6,'.-(",.#"*#)%$,-66-&'+4,<&"#*-/%$2,"-+1$'+,6"#*,%,
D. Infertility /"#='.,.#"*#)->+-&"-'()/,0('1('%"2,%3-)#*%,(),%),%31$'4,?1+.()/,3(+-%+-,
E. Neurologic dysfunction
#&&1"+, =.-), '.-"-, (+, %), <?@AB+-&"-'()/, 0('1('%"2, %3-)#*%4, <, '.2"#(3B
+'(*1$%'()/,.#"*#)-,C@DAEF+-&"-'()/,0('1('%"2,%3-)#*%,(+,1)&#**#)7,91',
(',&#1$3,%&&#1)',6#",-$-8%'-3,$-8-$+,#6,@G,%)3,@DA,(),%,.20-"'.2"#(3,0%'(-)'4,
H-1"#$#/(&, 32+61)&'(#), =('., .20#)%'"-*(%, 6"#*, '.-, +2)3"#*-, #6,
()%00"#0"(%'-,3(1"-'(&,.#"*#)-,$-%3()/,'#,)-1"#$#/(&,32+61)&'(#),(+,*#+',
#6'-), %, 0%"%)-#0$%+'(&, +2)3"#*-, &%1+-3, 92, %, +*%$$, &-$$, %)%0$%+'(&, $1)/,
&%"&()#*%7,91',*%2,%$+#,9-,31-,'#,.-%3,'"%1*%,'.%',3%*%/-+,'.-,)-"8-,
'"%&'+, %)3, )-1"#.20#0.2+(+, =('., 1)&#)'"#$$-3, "-$-%+-, #6, 8%+#0"-++(),
C%)'(3(1"-'(&,.#"*#)-7,<IAE4

A 20-year-old man’s closest friends tell him he looks different now than a A. Germline mutation of AIP
year ago, with coarse features. He bought new shoes with his usual size The aryl hydrocarbon receptor–interacting protein (AIP) gene mutations
and they do not fit. A year later his 23-year-old brother has similar account for a minority of growth hormone (GH)–producing pituitary
problems. Both of them have hyperglycemia. Which of the following adenomas, but may be present in younger persons in familial cases. GH
genetic alterations is most likely present in both brothers? opposes insulin, and excesses can lead to secondary diabetes mellitus. In
adults with closed epiphyses, gigantism does not occur, but acromegaly of
A. Germline mutation of AIP soft tissues does. Cyclin D1 mutations may be found in some parathyroid
B. Overexpression of cyclin D1 adenomas. The PAX8-PPARG
C. Fusion of PAX8-PPARG fusion gene may be found in some follicular thyroid neoplasms. The tumor
D. Loss of function mutation in PTEN suppressor gene PTEN may be
E. Loss of VHL heterozygosity mutated in some pancreatic neuroendocrine tumors. The VHL tumor
suppressor gene may be associated with some pheochromocytomas.

A 39-year-old woman has had no menstrual periods for the past year, D. Chromophobe E
along with malaise, cold intolerance, and loss of body hair. She has had She has a nonfunctioning (null-cell) pituitary adenoma that has enlarged to
headaches for the past 5 months. On physical examination her lateral compress and obliterate the normal adenohypophyseal cells resulting in
visual fields are reduced. She is most likely to have a neoplasm composed hypopituitarism; the adenoma also presses on the optic chiasm, producing
of which of the following cell types? bitemporal hemianopsia. About a fourth of all pituitary tumors are non-
functioning. Beta cells of the pancreatic islets of Langerhans produce
A. Beta B insulin. Chief cells in the parathyroids produce parathormone that
B. Chief C increases serum calcium. Chromaffin cells in the adrenal medulla produce
C. Chromaffin D Catecholamines. Glomerulosa cells of the adrenal cortex produce
D. Chromophobe E corticosteroids. Parafollicular cells (C cells) of the thyroid interstitium
E. Glomerulosa F produce calcitonin.
F. Parafollicular

❗USE AT YOUR OWN RISK❗ page 20 of 59


ROBBINS REVIEW

A 41-year-old woman notices that her gloves from the previous winter no B. Failure of growth hormone suppression
longer fit her hands. Her facial features have become coarse in the past J%($1"-, '#, +100"-++, /"#='., .#"*#)-, CKAE, $-8-$+, 92, /$1&#+-, ()61+(#),
year, and her voice seems deeper. On physical examination, her blood
pressure is 140/90 mm Hg. There is decreased sensation to pinprick over +1//-+'+, %1'#)#*#1+, KA, 0"#31&'(#)4, @.-, 0%'(-)'L+, +2*0'#*+, +1//-+',
the palms in the distribution of her thumb and first two fingers. A radiograph %&"#*-/%$27, %)3, %, KAB+-&"-'()/, %3-)#*%, (+, *#+', $(M-$24, <&"#*-/%$2,
of the foot shows an increased amount of soft tissue beneath the
calcaneus. A chest radiograph shows cardiomegaly. Laboratory studies &%1+-+,%),#8-"%$$,()&"-%+-,(),+#6','(++1-,(),%31$'+,9-&%1+-,#6,'.-,%)%9#$(&,
indicate a fasting serum glucose level of 138 mg/dL and hemoglobin A1c -66-&'+,#6,'.-,()&"-%+-,(),KA4,N-&%1+-,'.-,-0(0.2+-+,#6,'.-,$#)/,9#)-+,%"-,
level of 8.6%.
&$#+-3,(),%31$'+7,'.-"-,(+,)#','.-,()&"-%+-,(),.-(/.'7,#",/(/%)'(+*7,'.%',=#1$3,
Which of the following additional test results is most likely to indicate the 9-, +--), (), &.($3"-), =('., %, 0('1('%"2, %3-)#*%, '.%', (+, +-&"-'()/, -:&-++(8-,
cause of her physical and laboratory findings?
KA4,5)+'-%37,'.-,()&"-%+-,(),+#6','(++1-,*%++,*%)(6-+'+,%+,()&"-%+()/,+.#-,
A. Abnormal glucose tolerance test result #", /$#8-, +(O-7, &%"0%$, '1))-$, +2)3"#*-7, %)3, &#%"+-, 6%&(%$, 6-%'1"-+4, @.(+,
B. Failure of growth hormone suppression
C. Hyperprolactinemia =#*%), 0"#9%9$2, .%+, %), %9)#"*%$, /$1&#+-, '#$-"%)&-, '-+', "-+1$'7, 91', '.(+,
D. Increased serum TSH level 3#-+,)#',()3(&%'-,'.-,1)3-"$2()/,&%1+-,#6,3(%9-'-+,*-$$('1+7,=.(&.,(),'.(+,
E. Loss of diurnal serum cortisol levels
&%+-, (+, +-&#)3%"2, '#, %&"#*-/%$24, <, 0"#$%&'()#*%, =#1$3, &%1+-,
%*-)#"".-%,%)3,/%$%&'#"".-%,(),%,=#*%)4,<,'.2"#(3,+'(*1$%'()/,.#"*#)-,
C@DAEF+-&"-'()/,%3-)#*%,#6,'.-,0('1('%"2,&%),/(8-,"(+-,'#,.20-"'.2"#(3(+*,
=('.,%),()&"-%+-3,*-'%9#$(&,"%'-,'.%',=#1$3,*#+',$(M-$2,$-%3,'#,=-(/.',$#++7,
%)3, /$1&#+-, ()'#$-"%)&-, (+, )#', %, 6-%'1"-, #6, .20-"'.2"#(3(+*4, J1)&'(#)%$,
0('1('%"2, '1*#"+, &%), 9-, 3-'-&'-3, &$()(&%$$2, 9-6#"-, '.-2, 9-&#*-, $%"/-,
-)#1/.,'#,&%1+-,0"-++1"-,+2*0'#*+,+1&.,%+,8(+1%$,3(+'1"9%)&-+4,?1+.()/,
+2)3"#*-,6"#*,%),%3"-)%$,&#"'(&%$,)-#0$%+*,0"#31&()/,&#"'(+#$,&#1$3,9-,
%&&#*0%)(-3, 92, /$1&#+-, ()'#$-"%)&-7, .20-"'-)+(#)7, %)3, '"1)&%$, #9-+('27,
91','.-"-,(+,)#,#8-"%$$,()&"-%+-,(),+#6','(++1-+4

A 39-year-old G2, P2 woman, whose last pregnancy was 14 years ago, B. Empty sella syndrome
has had absent menstrual cycles for 6 months. She also reports the Empty sella syndrome is a rare condition, seen most frequently in obese
expression of milk from her breasts. On physical examination, she is women, and results from herniation of the arachnoid through the
normotensive. She is 150 cm tall and weighs 63 kg (body mass index 28). diaphragma sellae. Although the increased pressure can lead to reduction
Secondary sex characteristics are normal. Laboratory testing indicates in pituitary tissue through compression atrophy, there is typically adequate
that β-human chorionic gonadotropin level is normal. She has a normal functional anterior pituitary to prevent hypopituitarism. This herniation can
growth hormone stimulation test. CT scan of the head shows no cause a “stalk section” effect, however, with loss of prolactin inhibition and
abnormalities of bone and no hemorrhage. Brain MRI shows fluid density hyperprolactinemia. A craniopharyngioma is a destructive tumor mass that
within a normal-sized sella turcica. What is the most likely diagnosis? is usually seen at a younger age. Hemochromatosis can interfere with
organ function, including hypopituitarism; onset usually occurs later in
A. Craniopharyngioma women than in men (in the 60s in women compared with the 40s in men),
B. Empty sella syndrome owing to differences in physiologic iron losses (e.g., menstrual blood loss).
C. Hereditary hemochromatosis Prader-Willi syndrome is an example of genomic imprinting with
D. Prader-Willi syndrome hypothalamic dysfunction seen in prepubertal boys. A prolactinoma could
E. Prolactinoma be a microadenoma, but MRI in this case rules this out because of the fluid
F. Sheehan syndrome density in the sella (seen with T2 weighting). If she had Sheehan syndrome
after her pregnancy, she would have manifested hypopituitarism within
months, not years.

A 21-year-old woman delivers a term infant after an uncomplicated D Failure of growth hormone stimulation
pregnancy. The placenta cannot be delivered, however, and there is Sheehan syndrome, or postpartum pituitary necrosis, is caused by pituitary
substantial hemorrhage, requiring transfusion of 10 U of packed RBCs. enlargement during pregnancy, causing its blood supply to be more
She must undergo a hysterectomy. Over the next 3 months, she is unable tenuous so that intrapartum hypotension with obstetric bleeding
to produce sufficient milk to breastfeed her infant, and she becomes complications (e.g., the placenta accreta in this patient) predisposes to
increasingly fatigued. Laboratory studies show Na+, 134 mmol/L; K+, 5.2 infarction. The anterior pituitary is at greater risk than the posterior pituitary.
mmol/L; Cl–, 88 mmol/L; CO2, 23 mmol/L; glucose, 59 mg/dL; calcium, 9.3 The laboratory findings in this patient suggest adrenal insufficiency, and
mg/dL; phosphorus, 3.5 mg/ dL; and creatinine, 0.9 mg/dL. Over the next her inability to breastfeed is caused by lack of prolactin; loss of menstrual
5 months, her menstrual cycles do not return. cycles suggests that follicle-stimulating hormone and luteinizing hormone
Which of the following laboratory findings is now most likely to be reported levels are deficient. If she were to have primary adrenal failure, the
in this woman? corticotropin (ACTH) level would be increased, but in her case, ACTH is
A. Decreased corticotropin-releasing hormone low because of anterior pituitary failure.Because the hypothalamus is
B. Decreased oxytocin unaffected, corticotropin releasing hormone would still be present.
C. Failure of antidiuretic hormone release Oxytocin release from the posterior pituitary is probably not affected. She
D. Failure of growth hormone stimulation does not have diabetes insipidus from lack of antidiuretic hormone,
E. Increased corticotropin because the posterior pituitary is less likely to be involved. Dopamine
F. Increased dopamine production in the hypothalamus is not affected.

❗USE AT YOUR OWN RISK❗ page 21 of 59


ROBBINS REVIEW

A 42-year-old man has had polyuria and polydipsia for the past 4 months. E. Vasopressin
His medical history shows that he fell off a ladder and hit his head just Diabetes insipidus ensues from lack of antidiuretic hormone (ADH), also
before the onset of these problems. On physical examination, there are no called arginine vasopressin. There is failure of resorption of free water in
specific findings. Laboratory findings include serum Na+, 155 mmol/L; K+, the renal collecting tubules— hence the increased dilute urine with higher
3.9 mmol/L; Cl–, 111 mmol/L; CO2, 27 mmol/L; glucose, 84 mg/ dL; serum osmolality and hypernatremia. Corticotropin stimulates the adrenal
creatinine, 1 mg/dL; and osmolality, 350 mOsm/mL. The urine specific glands, mainly with the effect of increasing cortisol secretion. Oxytocin is
gravity is 1.002. This patient is most likely to have a deficiency of which of involved in lactation. Prolactin and melatonin deficiencies have no
the following hormones? identifiable specific clinical effects in men.
A. Corticotropin
B. Melatonin
C. Oxytocin
D. Prolactin
E. Vasopressin

A 69-year-old man has become progressively obtunded over the past D. Small cell lung carcinoma
week. He has an 80 pack-year history of smoking cigarettes. On physical The syndrome of inappropriate antidiuretic hormone(SIADH) secretion
examination, he is afebrile and normotensive. A head CT scan shows no results in increased free water resorption by the kidney and subsequent
intracerebral hemorrhages. Laboratory findings include serum Na+ of 115 hyponatremia. SIADH is most often a paraneoplastic effect, and small cell
mmol/L, K+ of 4.2 mmol/L, Cl– of 85 mmol/L, and bicarbonate of 23 (oat cell) anaplastic carcinoma of the lung (of neuroendocrine derivation
mmol/L. The serum glucose is 80 mg/dL, urea nitrogen is 19 mg/dL, and and seen almost exclusively in smokers) is the most likely candidate
creatinine is 1.7 mg/dL. Which of the following neoplasms is most likely to among probable malignant neoplasms. Anterior pituitary adenomas do not
be present in this man? produce antidiuretic hormone (ADH), which is released from the posterior
pituitary. Adrenal cortical carcinomas can secrete cortisol or sex steroids,
A. Adenohypophyseal adenoma but not ADH. Pheochromocytomas secrete catecholamines. Renal cell
B. Adrenocortical carcinoma carcinomas are known for various paraneoplastic effects, but SIADH is not
C. Pheochromocytoma a high probability.
D. Small cell lung carcinoma
E. Renal cell carcinoma

A 23-year-old man has experienced headaches, polyuria, and visual A. Craniopharyngioma


problems for the past 3 months. On physical examination, he has bilateral Craniopharyngiomas are uncommon, usually suprasellar neoplasms; they
temporal visual field deficits. CT scan of the head shows a large, partially are typically found in young individuals. They are thought to arise from
calcified, cystic mass occupying the sellar and suprasellar areas. embryological remnants of the Rathke pouch in the region of the pituitary.
Laboratory findings show a serum prolactin concentration of 60 ng/mL and These are aggressive neoplasms that infiltrate and destroy surrounding
serum sodium level of 152 mEq/L. Serum calcium, phosphate, and glucose tissues, making complete excision difficult. Despite their aggressive
levels are normal. The mass is excised, and histologic examination shows behavior, they are composed of benign appearing squamoid or primitive
a mixture of squamous epithelial elements and lipid-rich debris containing tooth structures. The increase in prolactin occurs as a “stalk section” effect,
cholesterol crystals. Which of the following lesions is most consistent with and the hypernatremia results from diabetes insipidus caused by
the clinical and laboratory findings in this patient? destruction of the hypothalamus, posterior pituitary, or both. A metastasis
to this location in a young individual is highly unlikely. Multiple endocrine
A. Craniopharyngioma neoplasia (MEN) type 1 includes pituitary adenomas, but not
B. Metastases from a lung neoplasm craniopharyngiomas. MEN 2 does not involve the pituitary. Prolactinomas,
C. Multiple endocrine neoplasia type 1 similar to pituitary adenomas, can enlarge the sella when they are
D. Multiple endocrine neoplasia type 2 macroadenomas, but are not typically suprasellar of destructive of
E. Prolactinoma surrounding structures.

A 42-year-old woman has a sudden onset of fever with headache, nausea, E. Tetracycline
diaphoresis, and palpitations. On physical examination her temperature is Thyroid storm is a medical emergency. There is not enough time to wait
39.2° C; pulse, 115/min; irregular respiratory rate, 30/min; and blood for confirmatory laboratory thyroid testing. There are increased
pressure, 150/85 mm Hg. Deep tendon reflexes are 4+ bilaterally. Her catecholamine levels, and the β-blocker propranolol will help prevent
outstretched hands exhibit a high frequency tremor. Which of the following emergent death from cardiac failure. Acetaminophen and ice packs are
drugs should she receive emergently? better choices to treat fever alone. Propylthiouracil (PTU) is the antithyroid
medication with the fastest onset of action (hours), along with iodine to help
A. Aspirin reduce preformed thyroid hormone output. The fever and ancillary findings
B. Hydrocortisone C Insulin here go beyond what would be expected with an acute infection.
D. Propranolol
E. Tetracycline

❗USE AT YOUR OWN RISK❗ page 22 of 59


ROBBINS REVIEW

A 47-year-old woman has had increasing fatigue with dyspnea and F. Thyroid follicular cells
reduced exercise tolerance for the past year. On examination she has The normal feedback loop of peripheral thyroid hormones (T3 and T4) onto
nonpitting edema of the lower extremities. Laboratory studies show a the basophils (thyrotrophs) of the adenohypophysis regulates TSH release
serum TSH level of 10 mU/L and T4 level of 2 μg/dL. She is most likely to (under tropic control of TRH from the hypothalamus). When patients with
have pathologic findings affecting which of the following cells? primary thyroid failure, the most common cause for hypothyroidism with
myxedema in adults, do not have sufficient residual functioning thyroid
A. Hypophyseal basophils follicular cells producing thyroid hormones, then the TSH will rise, as in this
B. Hypophyseal pituicytes case, in conjunction with a low T4 level. The levels of TRH from the
C. Hypothalamic glial cells hypothalamus are much lower and harder to measure for correlation with
D. Hypothalamic neurons thyroid gland function. The measurement of TSH is also the most useful
E. Thyroid C cells screening test for hyperthyroidism. Neurohypophyseal axons release ADH
F. Thyroid follicular cells and oxytocin produced in the hypothalamus, whereas modified glial cells
called pituicytes do not release hormones. Thyroid parafollicular, or “C”
cells, produce calcitonin.

A 2-year-old child has failure to thrive since infancy. Physical examination E. Thyroxine (T4)
shows that the child is short and has coarse facial features, a protruding Cretinism is a condition that is uncommon whenever routine newborn
tongue, and an umbilical hernia. As the child matures, profound intellectual screening is available for testing and treatment at birth for hypothyroidism.
disability becomes apparent. A deficiency of which of the following Hypothyroidism that develops in older children and adults is known as
hormones is most likely to explain these findings? myxedema. A lack of cortisol from primary adrenal failure leads to Addison
disease, or a 21-hydroxylase deficiency could produce congenital adrenal
A. Cortisol hyperplasia. An absolute deficiency of insulin leads to type 1 diabetes
B. Insulin mellitus, but this is more likely to develop in childhood or later, and there
C. Norepinephrine would be weight loss. There is no deficiency state caused by a lack of
D. Somatostatin norepinephrine or somatostatin.
E. Thyroxine (T4)

A 43-year-old woman has had increasing lethargy and weakness over the F. Anti–thyroid peroxidase antibody
past 3 years. She has cold intolerance and wears a sweater in the summer. The lymphoid follicles and the large, pink nodules of Hürthle cells in this
One year ago, she had menorrhagia, but now she has oligomenorrhea. photomicrograph are typical for Hashimoto thyroiditis. The anti–thyroid
She has difficulty concentrating, and her memory is poor. She has chronic peroxidase (antimicrosomal) and antithyroglobulin antibody titers typically
constipation. are increased in patients with Hashimoto thyroiditis when thyroid
enlargement is still present. In the later, “burnt-out” phase of Hashimoto
On physical examination, her temperature is 35.5° C, pulse is 54/min, thyroiditis, the antibodies are sometimes undetectable—only the
respirations are 13/min, and blood pressure is 110/70 mm Hg. She has hypothyroidism is. The thyroid stimulating hormone (TSH) level is an
alopecia, and her skin appears coarse and dry. Her face, hands, and feet indication of whether there is a primary disease in the thyroid. If the patient
appear puffy, with doughlike skin. Laboratory findings show hemoglobin, appears hypothyroid and a primary thyroid disease (e.g., Hashimoto
13.8 g/dL; hematocrit, 41.5%; glucose, 73 mg/dL; and creatinine, 1.1 thyroiditis) is suspected, the TSH level is elevated. If the patient appears
mg/dL. The representative microscopic appearance of her causative hyperthyroid and a primary thyroid disease (e.g., Graves disease) is
disease is shown in the figure. Which of the following serologic test findings suspected, the TSH level is decreased. Anticentromere antibody is
is most likely to be positive in this woman? characteristic of limited scleroderma (CREST syndrome). Anti– double-
stranded DNA antibody is very specific for systemic lupus erythematosus.
A. Anticentromere antibody Anti–Jo-1 antibody can be seen in polymyositis. Increased
B. Anti–double-stranded DNA antibody antimitochondrial antibody indicates primary biliary cirrhosis. Anti
C. Anti–Jo-1 antibody Ribonucleoprotein antibodies are seen in some collagen vascular
D. Antimitochondrial antibody diseases, such as mixed connective tissue disease.
E. Anti Ribonucleoprotein antibody
F. Anti–thyroid peroxidase antibody

A 37-year-old woman has had difficulty swallowing and a feeling of fullness C. Subacute granulomatous thyroiditis
in the anterior neck for the past week. She is recovering from a mild upper Subacute granulomatous thyroiditis (de Quervain thyroiditis) is a self-
respiratory tract infection 1 month ago. On physical limited condition that can be of viral origin because many cases are
examination, her temperature is 37.4° C, pulse is 74/min, respirations are preceded by an upper respiratory infection. The transient hyperthyroidism
16/min, and blood pressure is 122/80 mm Hg. Palpation of her diffusely results from inflammatory destruction of the thyroid follicles and release of
enlarged thyroid elicits pain. thyroid hormone. The released colloid acts as a foreign body, producing
Laboratory studies show an increased serum T4 level and a decreased florid granulomatous inflammation in the thyroid.
TSH level. Two months later, she no longer has these complaints. The T4 Hashimoto thyroiditis can enlarge the thyroid transiently, but there is
level is now normal. Which of the following conditions is most likely to have usually no pain or hyperthyroidism. Thyroid neoplasms are not typically
produced these findings? associated with signs and symptoms of inflammation and are rarely
A. Hashimoto thyroiditis functional. A toxic multinodular goiter likewise produces no signs of
B. Medullary thyroid carcinoma inflammation, and does not reverse functionality
C. Subacute granulomatous thyroiditis
D. Toxic follicular adenoma
E. Toxic multinodular goiter

❗USE AT YOUR OWN RISK❗ page 23 of 59


ROBBINS REVIEW

A 30-year-old woman has given birth to her second child. She develops B. Anti–thyroid peroxidase antibodies
heat intolerance and loses more weight than expected postpartum. On The presence of autoantibodies in the serum in this patient with transient
physical examination, her thyroid gland is enlarged but painless; there are hyperthyroidism would suggest Hashimoto thyroiditis (“hashitoxicosis”),
no other remarkable findings. Laboratory studies show a serum T4 level of but the variant called subacute lymphocytic painless thyroiditis may affect
12 μg/dL and a TSH level of 0.4 mU/L. A year later she is euthyroid. Which 1 in 20 postpartum women, and a minority progress to hypothyroidism.
of the following is most indicative of the pathogenesis of this patient’s Mutations in the RET proto-oncogene are associated with papillary
disease? carcinoma and medullary carcinoma of the thyroid. Irradiation of the thyroid
gland can give rise to hypothyroidism, but it is unlikely that irradiation in
A. Activational mutations in the RET proto-oncogene childhood would give rise to hypothyroidism at age 60 years. Irradiation
B. Anti–thyroid peroxidase antibodies also can predispose to the development of papillary carcinoma, but these
C. Irradiation of the neck during childhood tumors usually do not affect thyroid hormone secretion. An iodine
D. Prolonged iodine deficiency deficiency can lead to hypothyroidism, but a goiter would be present. A
E. Recent viral upper respiratory tract infection history of a viral infection sometimes precedes subacute granulomatous
thyroiditis, which is typically a self-limited disease that lasts for weeks to 2
months.

A 20-year-old woman and her twin sister both experience increasing B. Decreased thyroid-stimulating hormone level
diplopia. Their conditions develop within 3 years of each other. On physical Exophthalmos is a feature seen in about 40% of individuals with Graves
examination, they have exophthalmos and weak extraocular muscle disease. The hyperfunctioning thyroid gland leads to an increased T4 level,
movement. The thyroid gland is diffusely enlarged but painless in each with positive feedback from the pituitary to decrease thyroid-stimulating
sister, and there is no lymphadenopathy in either woman. Which of the hormone (TSH) secretion. There is about 50% concordance of Graves
following serum laboratory findings is most likely to be reported in these disease among identical twins. The autoimmune character of this disorder
sisters? is evidenced by an association with HLA-DR3 and by the presence of an
autoantibody against TSH receptor that activates T4 secretion. An
A. Decreased free thyroxine level increased thyrotropin-releasing hormone (TRH) level would increase the
B. Decreased thyroid-stimulating hormone level TSH level and increase the T4, but feedback typically occurs at the level
C. High titer thyroid peroxidase autoantibodies of the pituitary and the hypothalamus, and abnormal increases in TRH are
D. Increased thyrotropin-releasing hormone level uncommon. Anti–thyroid peroxidase antibodies can be seen in Hashimoto
E. Increased triiodothyronine level thyroiditis and in Graves disease, but the highest liters occur inHashimoto
thyroiditis. T3 levels are less likely to be elevated
than T4 levels.

A 21-year-old woman has noted increasing fatigue and a 7-kg weight loss A. Antibodies against TSH receptor
without dieting over the past 4 months. She also has increasing anxiety The tall columnar epithelium with papillary infoldings and scalloping of the
and nervousness with diarrhea. Physical examination shows a diffusely colloid is characteristic of Graves disease, which leads to hyperthyroidism.
enlarged thyroid gland. Her temperature is 37.5° C, pulse is 103/min, This disease is caused by autoantibodies that bind to the thyroid-
respirations are 28/min, and blood pressure is 140/75 mmHg. A stimulating hormone (TSH) receptor and mimic the action of TSH. Dietary
radionuclide scan of the thyroid shows a diffuse increase in uptake. The iodine deficiency can cause diffuse compensatory enlargement of thyroid,
figure shows the representative microscopic appearance of the thyroid but it does not cause hyperthyroidism. Irradiation of the neck is a
gland. What is most likely to produce these findings? predisposing factor for papillary carcinoma of the thyroid. Maternal thyroid
hormone deficiency may affect childhood mental development. Mutations
A. Antibodies against TSH receptor in the RET proto-oncogene occur in papillary carcinoma of the thyroid and
B. Dietary deficiency of iodine in medullary carcinomas of the thyroid. These neoplasms do not cause
C. Irradiation of the neck diffuse enlargement of the gland, hyperthyroidism, or diffuse increase in
D. Maternal deficiency in T4 radioiodine uptake. Tumors usually produce “cold nodules” on radioiodine
E. Mutation in the RET proto-oncogene scans.

A 40-year-old man notes weight loss, increased appetite, and double E. Papillary projections in thyroid follicles and lymphoid aggregates
vision for 6 months. On physical examination, his temperature is 37.7° C, in the stroma
pulse is 106/min, respirations are 20/min, and blood pressure is 140/80 The clinical findings in this case point to hyperthyroidism, and the
mm Hg. A fine tremor is observed in his outstretched hands. He has increased, diffuse uptake corroborates Graves disease as a probable
bilateral proptosis and corneal ulceration Laboratory findings include a cause because the thyroid-stimulating hormone (TSH) level is quite low.
serum TSH level of 0.1 mU/L. A radioiodine scan indicates increased The thyroid- stimulating immunoglobulins that appear in this autoimmune
diffuse uptake throughout the thyroid. He receives propylthiouracil therapy, condition result in diffuse thyroid enlargement and hyperfunction, and
and his condition improves. Which of the following best describes the papillary projections lined by tall columnar epithelial cells. Destruction of
microscopic appearance of his thyroid gland before thyroid follicles with lymphoid aggregates and Hürthle cell metaplasia is
therapy? characteristic of Hashimoto thyroiditis. A goiter has enlarged follicles and
flattened epithelial cells; most of these patients are euthyroid. Follicular
A. Destruction of follicles, lymphoid aggregates, and Hürthle cell destruction and the presence of giant cells occur in granulomatous
metaplasia thyroiditis. Nests of cells in a Congo red– positive hyaline stroma
B. Enlarged thyroid follicles lined by flattened epithe- lial cells characterize a medullary carcinoma, which can be multifocal, but is not
C. Follicular destruction with inflammatory infiltrates containing giant cells diffuse and does not lead to hyperthyroidism.
D. Nodules with nests of cells separated by hyaline stroma that stains with
Congo red
E. Papillary projections in thyroid follicles

❗USE AT YOUR OWN RISK❗ page 24 of 59


ROBBINS REVIEW

A 45-year-old woman from Kathmandu, Nepal, reports a feeling of fullness A. Diffuse nontoxic goiter
in her neck, but has no other concerns. The enlargement has been gradual Diffuse nontoxic goiter is most often caused by dietary iodine deficiency.
and painless for more than 1 year. Physical examination confirms diffuse This condition is endemic in regions of the world where there is a deficiency
enlargement of the thyroid gland without any apparent masses or of iodine (e.g., inland mountainous areas); it also may occur sporadically.
lymphadenopathy. As in this case, patients are typically euthyroid. A follicular adenoma rarely
functions to produce excess thyroid hormone; most are “cold,” non-
Laboratory studies of thyroid function show a normal free T4 level and a functioning nodules that do not involve the thyroid diffusely. A chronic
slightly increased TSH level. What is the most likely cause of these lymphocytic thyroiditis, such as Hashimoto thyroiditis, initially can produce
findings? thyroid enlargement, but atrophy eventually occurs with resulting
hypothyroidism. Papillary carcinomas most often produce a mass effect or
A. Diffuse nontoxic goiter metastases and do not affect thyroid function. Subacute granulomatous
B. Follicular adenoma thyroiditis can lead to diffuse enlargement, and transient hyperthyroidism
C. Hashimoto thyroiditis can occur, but the disease typically runs a course of no more than 6 to 8
D. Papillary carcinoma weeks. Plummer disease, or toxic multinodular goiter, occurs when there
E. Subacute granulomatous thyroiditis is a hyperfunctioning nodule in a goiter.
F. Toxic multinodular goiter

A 14-year-old girl noticed gradual neck enlargement during the past 8 A. Cabbage
months. On physical examination her thyroid gland is diffusely enlarged. She has developed a sporadic goiter. Vegetables of the Brassicaceae
Her serum TSH level is normal. A dietary history is most likely to reveal family, including cabbage, turnips, and Brussels sprouts, contain
that she has begun eating more of which of the following foods? glucosinolate, which can decompose to release thiocyanate, a by-product
that interferes with thyroid hormone synthesis. Thus such substances are
A. Cabbage known as goitrogens. Young persons with increased demand for thyroid
B. Fava beans hormone are at increased risk. Fava beans contain oxidizing agents that
C. Fish incite hemolysis in persons with glucose 6-phosphate dehydrogenase
D. Plantains (G6PD) deficiency. In addition, beans contain long-chain sugars that are
E. Rye bread indigestible with human intestinal enzymes, leaving them to be fermented
by colonic bacteria that release gas (flatus). Fish have omega-3 fatty acids
that protect against atherogenesis. Plantains are starchy, as anyone
mistaking them for sweet bananas soon discovers; rare food allergy
develops to them. Fungus growing on moldy rye produces ergot poisoning.

A 70-year-old man has had greater difficulty swallowing for the past 2 E. Toxic multinodular goiter
years. Over the past 6 months, he has lost 3 kg. On physical examination, A long-standing diffuse goiter can evolve into a multinodular goiter, and
his temperature is 37.3° C, and pulse is 102/min. There is fullness to the one of the nodules can begin hyperfunctioning to cause so-called Plummer
anterior neck, with a 5 × 10 cm irregular mass on palpation. Laboratory disease. This “toxic” nodule has acquired growth and functional
studies show serum TSH of 0.2 mU/L. A thyroid scintigraphic scan shows characteristics similar to a benign neoplasm, such as a follicular adenoma,
a 1.5-cm nodule with increased uptake in the right thyroid lobe, and but one that is functional. Rare toxic follicular adenomas can
decreased uptake into the remaining enlarged thyroid. What is the most function and produce “hot” nodules, but the remaining gland is often
likely diagnosis? atrophic, not enlarged. In Graves disease, the thyroid is enlarged, but
usually diffusely, without pronounced nodularity, so that there is increased
A. Follicular adenoma uptake into the entire gland. In addition, clinical features such as
B. Graves disease dermopathy and ophthalmopathy that are lacking with Plummer disease
C. Hashimoto thyroiditis are associated with Graves disease. There may be initial diffuse thyroid
D. Papillary carcinoma enlargement with Hashimoto thyroiditis and transient hyperfunction, but
E. Toxic multinodular goiter over time the thyroid atrophies, and hypothyroidism ensues. It is extremely
rare for a papillary carcinoma to function, and although this would be a hot
nodule, the remaining thyroid would not be enlarged.

A 38-year-old woman felt a small bump on the right side of her neck 1 D. Normal TSH
month ago, and it has not changed since then. Physical examination Solitary “cold” and solid (not cystic) thyroid nodules are likely to be
shows a 1-cm painless nodule palpable in the right lower pole of the thyroid neoplastic, and most are benign follicular adenomas that do not affect
gland. There is no lymphadenopathy. Radionuclide scanning shows that thyroid function. If the nodule was hyperfunctioning, and produced
the nodule does not absorb radioactive iodine, and no other nodules are hyperthyroidism, it would appear “hot” on the scan, and suppress thyroid-
present. A fine-needle aspiration biopsy of the nodule is done, and the stimulating hormone (TSH). Anti-TSH receptor immunoglobulins can be
cytologic features are those of a follicular neoplasm. Which of the following seen in Graves disease, as may high T4 and low TSH levels, but this is a
laboratory findings is most likely to be present in this patient? diffuse disease of the thyroid. T3 levels are rarely measured.
Antimicrosomal and antithyroglobulin antibodies are seen in Hashimoto
A. Anti–TSH receptor immunoglobulins thyroiditis (and Graves disease), but thyroiditis is a diffuse process and
B. High free T4 unlikely to produce a solitary nodule. As Hashimoto thyroiditis progresses,
C. Low T3 decreasing function of the thyroid can lead to decreasing T4 level and
D. Normal TSH increasing TSH level, typical of primary thyroid failure
E. Anti–thyroid peroxidase antibodies

❗USE AT YOUR OWN RISK❗ page 25 of 59


ROBBINS REVIEW

A 30-year-old woman has a 6-month history of weight loss (3 kg), hand C. Gain of function mutation of TSH receptor gene
tremors, mild watery diarrhea, and heat intolerance. On physical A thyroid nodule that is well differentiated, noninvasive, and
examination, vital signs are temperature, 37.3° C; pulse, 103/min with hyperfunctioning suggests a “toxic” follicular adenoma, and a large
sinus rhythm; respirations, 20/ min; and blood pressure, 125/85 mm Hg. proportion of these adenomas have activating mutations of the thyroid-
She has a 1-cm firm, painless nodule palpable on the left side of her neck. stimulating hormone (TSH) receptor–signaling pathway involving the TSH
There is no lymphadenopathy. No other abnormal findings are noted. receptor or associated G protein. Mutation of the GNAS1 gene is seen in
Laboratory findings include a total serum T4 of 11.6 μg/dL with TSH of 0.2 McCune-Albright syndrome and some pituitary adenomas. A PAX8-
mU/L. A scintigraphic scan shows more uptake of radioactive iodine into PPARG fusion gene is found in certain follicular carcinomas of the thyroid.
the nodule than the surrounding thyroid. A partial thyroidectomy is Activation of the RET gene occurs in papillary thyroid carcinomas.
performed, and microscopic examination of the excised nodule shows Overexpression of the cyclin D1 gene (CCND1) is characteristic of
well-differentiated thyroid follicles without vascular or capsular invasion. parathyroid adenomas. The AIRE gene regulates expression of self-
Molecular analysis of this nodule is most likely to reveal which of the antigens in the thymus, and mutation in this gene causes autoimmune
following genetic changes? polyendocrinopathy affecting adrenals, parathyroids, gonads, and gastric
parietal cells with destruction of these tissues and consequent
A. Activating missense mutation of GNAS1 gene hypofunction
B. Fusion gene formed by PAX8 and PPARG genes
C. Gain of function mutation of TSH receptor gene
D. Mutational activation of RET tyrosine kinase receptor
E. Overexpression of cyclin D1 (CCND1) gene
F. Stop codon mutation of autoimmune receptor (AIRE) gene

A 44-year-old, otherwise healthy woman feels a small lump on the left side D. Papillary carcinoma
of her neck. A firm, painless, 1.5-cm cervical lymph node is palpable. The Papillary thyroid carcinomas may present initially with metastases, and
thyroid gland is not enlarged. A chest radiograph is unremarkable. local lymph nodes are the most common sites for metastases. The primary
Laboratory findings include serum glucose, 83 mg/dL; creatinine, 1.2 mg/ site may not be detectable as a palpable nodule. Papillary carcinoma is the
dL; calcium, 9.1 mg/dL; phosphorus, 3.3 mg/dL; thyroxine, 8.7 μg/dL; and most common thyroid malignancy. Metastases to the thyroid are
TSH, 2.3 mU/L. The hemoglobin is 14 g/dL, platelet count is 240,400/mm3, uncommon. Anaplastic carcinomas are uncommon, but are very
and WBC count is 5830/mm3. A fine-needle aspiration biopsy of the aggressive, locally invasive lesions. Follicular and medullary carcinomas
thyroid gland is done. What is the most likely diagnosis? tend to spread hematogenously. A parathyroid carcinoma is a locally
infiltrative mass, and the serum calcium level is usually quite high in such
A. Anaplastic carcinoma patients. A small lymphocytic lymphoma, which is the tissue form of chronic
B. Follicular carcinoma lymphocytic leukemia, is uncommon at this patient’s age, usually involves
C. Medullary carcinoma multiple nodes, and is accompanied by an elevated WBC count.
D. Papillary carcinoma
E. Parathyroid carcinoma
F. Small lymphocytic lymphoma

A 45-year-old man has felt a lump on the left side of his neck for 4 months. D. RET gene mutation
Physical examination shows a nontender nodule on the left lobe of the The papillary architecture in this nodule, with cells that have clear nuclei,
thyroid gland. An adjacent cervical lymph node is enlarged and nontender. is a pattern typical for papillary carcinoma. There is no such thing as a
Laboratory studies show no thyroid autoantibodies in his serum, and the papillary adenoma. These nuclear changes, even if the pattern is follicular,
T4 and TSH levels are normal. A thyroidectomy is performed; the figure confirm the diagnosis of papillary carcinoma. About 30% of all papillary
shows the microscopic appearance of the nodule. thyroid carcinomas have mutational activation of RET or NTRK1 proto-
Which of the following etiologic factors is most likely to be involved in the oncogenes, which belong to the family of receptor tyrosine kinases that
pathogenesis of the thyroid nodule in this patient? transduce extracellular signals for cell growth and differentiation and exert
A. Autoimmunity many of their downstream effects through the ubiquitous MAP kinase
B. Chronic dietary iodine deficiency signaling pathway. Iodine deficiency gives rise to uniform thyroid
C. Consumption of goitrogens enlargement because the secretion of thyroid stimulating hormone (TSH)
D. RET gene mutation increases when there is reduced synthesis of T4. Autoimmunity plays a
E. Viral infection deficiency, with potential hypothyroidism, but not role in Hashimoto thyroiditis and Graves disease, and these do not
malignancy. Viral infections can cause subacute thyroiditis, not carcinoma. progress to carcinoma, although non-Hodgkin lymphoma may develop in
the former. Goitrogens interfere with thyroid hormone synthesis and have
an effect similar to that of iodine deficiency, with potential hypothyroidism,
but not malignancy. Viral infections can cause subacute thyroiditis, not
carcinoma.

A 62-year-old woman has felt a lump on the right side of her neck for 5 B. Follicular carcinoma
months. On physical examination, she has no palpable lymphadenopathy. Follicular carcinoma can be difficult to differentiate from follicular adenoma,
A fine-needle aspiration biopsy is done, and she undergoes a unless there is microscopic evidence of invasion, as shown in the figure,
thyroidectomy. A 3-cm solid mass within the right thyroid lobe has the so the term follicular neoplasm may be given for a fine-needle aspiration
microscopic appearance shown in the figure. Six months later, she has specimen. Follicular carcinomas are often indolent, but they can
pain in the right thigh. A radiograph shows a fracture of the right femur in metastasize, and then are easily distinguished from follicular adenomas.
an area of lytic bone destruction. A radioiodine scan shows uptake Follicular carcinomas are much less likely than papillary carcinomas to
localized to the region of the fracture. Which of the following is the most involve lymph nodes, but they are more likely to metastasize to distant
likely diagnosis? sites, such as bone, lung, and liver. If the metastatic lesions are functional,
they absorb radioactive iodine. Thyroiditis is unlikely to manifest as a mass
A. Papillary carcinoma because the entire gland is typically involved. Medullary carcinoma is less
B. Follicular carcinoma common than follicular carcinoma. There is an increased risk of non-
C. Granulomatous thyroiditis Hodgkin B-cell lymphoma in patients with Hashimoto thyroiditis, but
D. Hashimoto thyroiditis lymphomas are unlikely to destroy bone, although they can be present in
E. Non-Hodgkin lymphoma the marrow.

❗USE AT YOUR OWN RISK❗ page 26 of 59


ROBBINS REVIEW

A 44-year-old man with no previous illnesses has A. Anaplastic carcinoma


been bothered by progressive hoarseness, shortness of breath, and stridor The large size, histologic features, and aggressive nature of this neoplasm
for the past 3 weeks. On physical examination, he has a firm, large, tender are consistent with anaplastic carcinoma. The prognosis is poor. Other
mass involving the entire right thyroid lobe. CT scan shows extension of thyroid malignancies tend to form solitary or multifocal (in papillary and
this mass posterior to the trachea and into the upper mediastinum. A fine- medullary carcinomas) masses without spindle cells; they are less likely to
needle aspiration biopsy of the mass is done, and microscopically shows be extensively invasive, although metastases can occur, particularly to
pleomorphic spindle cells. The mass is noted at surgery to have infiltrated local lymph nodes in the case of papillary carcinomas or lung in the case
the adjacent skeletal muscle. Four of seven cervical lymph nodes have of follicular carcinomas. Malignant lymphomas do not have spindle cells
metastases. Pulmonary metastases also are identified on a chest and do not tend to be infiltrative.
radiograph. Which of the following neoplasms is most likely to be present
in this man?

A. Anaplastic carcinoma
B. Follicular carcinoma
C. Medullary carcinoma
D. Non-Hodgkin lymphoma
E. Papillary carcinoma

A 42-year-old woman has noted increasing fullness in her neck for the past A. Calcitonin
7 months. On physical examination, her thyroid gland is diffusely and Medullary carcinomas are derived from the C cells, or parafollicular cells,
asymmetrically enlarged and nodular. There is no lymphadenopathy. She of the thyroid, with embryologic origin from neural crest. Therefore they
undergoes thyroidectomy. Gross examination of the thyroid shows a have neuroendocrine function, including synthesis of calcitonin. An
multicentric thyroid neoplasm; microscopically, the neoplasm is composed amyloid stroma is a common feature of this tumor. These tumors occur
of polygonal- to spindle-shaped cells forming nests and trabeculae. There sporadically in about 70% of cases, but they can be familial and part of
is a prominent, pink hyaline stroma that stains positively with Congo red. multiple endocrine neoplasia types 2A and 2B. CD3 is a useful marker for
Electron microscopy shows varying numbers of intracytoplasmic, some lymphoid neoplasms. Although various tissues may show positivity
membrane-bound, electron-dense granules. Immunohistochemical for estrogen receptors, this finding has no clinical significance in thyroid.
staining for which of the following antigens is most useful for diagnosis of Staining for parathyroid hormone is useful to determine if a parathyroid
this neoplasm? carcinoma is present. Vimentin is a marker for sarcomatous neoplasms,
and cytokeratin is a useful marker to determine if a neoplasm is epithelial.
A. Calcitonin
B. CD3
C. Cytokeratin
D. Estrogen receptor
E. Parathyroid hormone
F. Vimentin

A 40-year-old man experiences weakness and easy fatigability of 2 D. Parathyroid adenoma


months’ duration. Physical examination yields no remarkable findings. When a patient develops hypercalcemia, a disorder of the parathyroid
Laboratory studies show serum calcium of 11.5 mg/dL, inorganic glands or a malignancy at a visceral location must be considered. The
phosphorus of 2.1 mg/dL, and serum parathyroid hormone of 58 pg/mL, elevated parathyroid hormone (PTH) suggests primary
which is near the top of the reference range. A radionuclide bone scan fails hyperparathyroidism. The most common cause of primary
to show any areas of increased uptake. What is the most likely cause of hyperparathyroidism is a parathyroid adenoma. Secondary
these findings? hyperparathyroidism, most commonly resulting from renal failure, is
A. Chronic renal failure excluded
B. Hypervitaminosis D when the serum inorganic phosphate level is low because phosphate is
C. Medullary thyroid carcinoma retained with chronic renal failure. Hypervitaminosis D can cause
D. Parathyroid adenoma hypercalcemia because of increased calcium absorption, but in these
E. Parathyroid carcinoma cases, the PTH levels are expected to be near the low end of the reference
F. Parathyroid hyperplasia range because of feedback suppression. Serum PTH levels near the high
end of the reference range indicate autonomous PTH secretion
unregulated by hypercalcemia. Although medullary carcinomas of the
thyroid often have
positive immunohistochemical staining for calcitonin, and plasma levels
are sometimes increased, there is typically no major reduction in serum
calcium as a result.

A 63-year-old woman had frequent headaches for 1 month. She now C. Metastatic breast carcinoma
suddenly experiences a generalized seizure and becomes obtunded. She A common cause of clinically significant hypercalcemia in adults is
is taken to the emergency department, where a physical examination malignancy. When a patient presents with hypercalcemia, a disorder of the
reveals an irregular heart rate. Laboratory findings include serum calcium parathyroid glands or a malignancy at a visceral location must be
of 15.4 mg/ dL, serum phosphorus of 1.9 mg/dL, and albumin of 4.2 g/ dL. considered. Hypercalcemia from malignancy can be caused by osteolytic
A chest radiograph shows multiple lung masses and lytic lesions of the metastases or a paraneoplastic syndrome from secretion of parathyroid
vertebral column. Which of the following conditions best accounts for these hormone–related protein by the tumor. Metastatic disease from common
findings? primary sites, such as the breast, lung, and kidney, is much more common
than parathyroid carcinoma, which tends to be local but aggressive.
A. Chronic renal failure Chronic renal failure causes phosphate retention, which tends to depress
B. Disseminated tuberculosis the serum calcium level and leads to secondary hyperparathyroidism; the
C. Metastatic breast carcinoma serum calcium level is maintained at near-normal levels. Tuberculosis, a
D. Parathyroid carcinoma granulomatous disease, can be associated with hypercalcemia from up-
E. Vitamin D toxicity regulation of 1,25-dihydroxycholecalciferol in activated macrophages; lytic
bone lesions from tuberculosis are uncommon. Parathyroid carcinomas
are an uncommon cause of hyperparathyroidism, and bone metastases
from
Parathyroid carcinomas are rare. Vitamin D toxicity theoretically can lead
to hypercalcemia, but this condition is uncommon.

❗USE AT YOUR OWN RISK❗ page 27 of 59


ROBBINS REVIEW

A 40-year-old woman notes lethargy, weakness, and constipation for the B. MEN1
past 6 months. On physical examination, she is afebrile and normotensive, <, 0%"%'.2"#(3, )-#0$%+*7, *#+', $(M-$2, %), %3-)#*%7, (+, &%1+()/,
and her heart rate is irregular. There is pain on palpation of the left third
proximal finger. An ECG shows a prolonged QT (corrected) interval. .20-"&%$&-*(%, &#*0$(&%'-3, 92, #+'-('(+, 6(9"#+%, &2+'(&%, #6, .-", 6()/-"4,
Laboratory studies show glucose, 73 mg/dL; creatinine, 1.2 mg/dL; !%"%'.2"#(3,.20-"0$%+(%,()8#$8()/,%$$,'.-,/$%)3+,(+,$-++,$(M-$24,!%"%'.2"#(3,
calcium, 11.6 mg/dL; phosphorus, 2.1 mg/dL; total protein, 7.1 g/dL;
albumin, 5.3 g/dL; and alkaline phosphatase, 202 U/L. A radiograph of the &%"&()#*%+,%"-,"%"-7,'.%)M61$$27,9-&%1+-,'.-2,%"-,%//"-++(8-7,%)3,+-"1*,
left hand shows focal expansion by a cystic lesion of the third proximal &%$&(1*,$-8-$+,*%2,9-,+#,.(/.,'.%',&%"3(%&,%"".2'.*(%+,#&&1"4,@.-,;PHQ,
phalanx. A technetium radionuclide scan shows a 1-cm area of increased
uptake in the right lateral neck. A mutation in which of the following genes *1'%'(#),(+,'.-,+-&#)3,*#+',&#**#),*1'%'(#),(),0%"%'.2"#(3,'1*#"+7,%6'-",
is most likely present in this woman? '.-,()8-"+(#),=('.,#8-"-:0"-++(#),#6,??HIQ7,'.-,/-)-,-)&#3()/,&2&$(),IQ4,
A. GNAS1 ;PHQ, (+, %, '1*#", +100"-++#", /-)-7, '.-, $#++, #6, =.(&., #&&1"+, )#', #)$2, (),
B. MEN1 +0#"%3(&, 0%"%'.2"#(3, '1*#"+7, 91', %$+#, (), *1$'(0$-, -)3#&"()-, )-#0$%+(%,
C. TP53
D. RET C;PHE,'20-,Q4,KH<DQ,*1'%'(#)+,%"-,+--),(),GRS,#6,&%+-+,#6,+#*%'#'"#0.,
E. VHL C/"#='.,.#"*#)->0"#31&()/E,0('1('%"2,%3-)#*%+4,@.-,@!TU,*1'%'(#),&%),
9-,+--),(),%)%0$%+'(&,'.2"#(3,&%"&()#*%+4,@.-,VP@,/-)-,*1'%'(#),(+,+--),
(),&%+-+,#6,*-31$$%"2,'.2"#(3,&%"&()#*%4,@.-,WAX,*1'%'(#),&%),9-,+--),(),
+#*-,0.-#&."#*#&2'#*%+,(),0%'(-)'+,=('.,8#),A(00-$FX()3%1,3(+-%+-4

A 68-year-old man has experienced increasing malaise for 3 years. D. Parathyroid hyperplasia
Physical examination shows no remarkable findings. Laboratory findings Chronic kidney injury, often leading to small end-stage Kidneys with
include a serum creatinine level of 4.9 mg/dL and urea nitrogen level of 45 chronic renal failure can lead to secondary hyperparathyroidism resulting
mg/dL. Abdominal CT scan shows small kidneys. Which of the following from decreased phosphate excretion by the kidneys. The resultant
endocrine glandular lesions has developed secondary to the underlying hyperphosphatemia depresses the serum calcium level and stimulates
disease in this patient? parathyroid gland activity. Because of reduced renal parenchymal function,
there also is less active vitamin D, which leads to decreased dietary
A. Adrenal atrophy calcium absorption. Renal failure does not lead to any of the other
B. Islet cell hyperplasia endocrine lesions listed.
C. Multinodular goiter
D. Parathyroid hyperplasia
E. Pituitary microadenoma

A 47-year-old woman noticed a lump in her neck 1 week ago. On physical C. Calcium B
examination, there is a 2-cm nodule in the right lobe of the thyroid gland. Inadvertent removal of or damage to the parathyroid glands during thyroid
A fine-needle aspiration biopsy is performed, and microscopic examination surgery can cause hypocalcemia secondary to hypoparathyroidism. This
of the specimen shows cells consistent with a follicular neoplasm. She is the most common cause of hypoparathyroidism. Individuals with
undergoes a subtotal thyroidectomy. Which of the following laboratory hypocalcemia exhibit neuromuscular irritability, carpopedal spasm, and
tests should be performed on this patient in the immediate postoperative sometimes seizures. Antithyroglobulin antibody levels are of no use in
period? diagnosing surgical diseases of the thyroid. Calcitonin quantitation is not a
useful measure to determine the status of calcium metabolism. Parathyroid
A. Antithyroglobulin antibody hormone levels decrease if the parathyroid glands are inadvertently
B. Calcitonin A removed during thyroid surgery, but the calcium level is the best immediate
C. Calcium B indicator of
D. Parathyroid hormone C hypoparathyroidism, and this test is more readily available in the
E. TSH laboratory. The thyroid-stimulating hormone (TSH) concentration can
increase if the patient becomes hypothyroid after surgery and is not
receiving thyroid hormone replacement, but this is not an immediate
problem.

A 27-year-old man has controlled his diabetes mellitus for the past 10 E. Hypoglycemic coma
years with insulin injections. This morning, his roommate is unable to An insulin overdose produces hypoglycemic coma. He does not have
awaken him. The man is unconscious when brought to the emergency detectable C peptide, which indicates that there is no endogenous insulin
department. On physical examination, his temperature is 37° C, pulse is production, typical for type 1 diabetes. The high insulin level is the result of
91/min, respirations are 30/min, and blood pressure is 90/65 mm Hg. the patient’s use of exogenous insulin to treat his diabetes mellitus.
Laboratory findings include a high plasma level of insulin and a lack of Because he has not eaten enough to maintain glucose at an adequate
detectable C peptide. Urinalysis shows no blood, protein, or glucose, but level, he has developed hypoglycemia. The ketosis in this case results from
4+ ketonuria. Which of the following conditions is most likely to be present? decreased food intake, and anyone not consuming enough calories will
develop ketosis. ketoacidosis in type 1 diabetes mellitus would be
A. Acute myocardial infarction accompanied by hyperglycemia. Acute myocardial Infarction is a
B. Bacteremia complication that generally occurs later in the course of diabetes when
C. Hepatic failure more atherosclerosis has developed. The patient has no obvious source
D. Hyperosmolar syndrome of sepsis. Insulin is not injected into the bloodstream, and the injections are
E. Hypoglycemic coma almost never complicated by infection. Hepatic failure is not a typical
F. Ketoacidosis complication of diabetes mellitus. Hyperosmolar coma can complicate type
2 diabetes mellitus.

❗USE AT YOUR OWN RISK❗ page 28 of 59


ROBBINS REVIEW

Blood relatives of individuals diagnosed with type 1 or type 2 diabetes E. Insulitis


mellitus are studied for 10 years. Laboratory testing for glucose and insulin The presence of HLA-DR3 and HLA-DR4 alleles of the MHC class II region
levels and autoantibody formation is performed on a periodic basis. The has the strongest linkage to type 1 diabetes mellitus. Autoantibodies to islet
HLA types of the subjects are determined. A cohort of the subjects who cell antigens such as glutamic acid decarboxylase are present years
are 8 to 22 years old has no overt clinical illnesses and no hyperglycemia; before overt clinical diabetes develops. An insulitis caused by T cell
however, autoantibodies to glutamic acid decarboxylase are present. infiltration occurs before the onset of symptoms or very early in the course
Many subjects in this cohort have the HLA-DR3 and HLA-DR4 alleles. of type 1 diabetes mellitus. The insulitis in type 1 diabetes mellitus is
Which of the following pancreatic abnormalities is most likely to be found associated with increased expression of class I MHC molecules and
in this cohort of study subjects? aberrant expression of class II MHC molecules on the beta cells of the
islets. These changes are mediated by cytokines such as interferon-
A. Acinar acute inflammation and necrosis elaborated by CD4+ cells (along with CD8+ cells). Acute neutrophilic
B. Acinar fibrosis and fatty replacement infiltration with necrosis and hemorrhage are characteristic of acute
C. Islet amyloid deposition pancreatitis. Extensive fibrosis and fatty replacement of the pancreas is
D. Islet hyperplasia seen in patients with cystic fibrosis surviving for decades. Islet hyperplasia
E. Insulitis occurs in infants of diabetic mothers. Amyloid deposition in islets may be
F. Normal islets in a fibrous stroma seen in some cases of type 2 diabetes mellitus. A fibrous stroma with
minimal chronic inflammation and scattered normal islets is seen with
chronic pancreatitis.

A 23-year-old woman has a routine health status examination. Her body B. Glucokinase
mass index is 22. Laboratory studies show fasting plasma glucose is 130 This patient has maturity-onset diabetes of the young (MODY), which
mg/dL. Urinalysis shows mild glycosuria, but no ketonuria or proteinuria. accounts for less than 5% of cases of diabetes mellitus and is linked to
She has no detectable insulin autoantibodies. Her father was similarly specific genetic defects involving islet cells and inherited in an autosomal
affected at age 20 years. She is most likely to have a mutation in a gene dominant fashion. In this patient with MODY2, there is an inactivating
encoding for which of the following? mutation in glucokinase that increases the beta cell threshold for insulin
release, and the hyperglycemia is mild MODY3 is more common, results
A. Glucagon from mutations in hepatocyte nuclear factor 1#, has a severe insulin
B. Glucokinase secretion defect in response to glucose, and results in more severe
C. GLUT4 hyperglycemia. Loss of glucagon secretion does not have significant
D. Insulin effects and does not lead to hyperglycemia. The GLUT4 receptors are
E. MHC DR found on target cells for insulin, such as adipocytes and myocytes, but
cases of diabetes related to mutations in GLUT receptors are rare.
Mutations affecting insulin itself are rare. Certain MHC alleles predispose
to type 1 and type 2 diabetes mellitus, but are not involved in pathways of
insulin metabolism.

A 13-year-old girl collapses while playing basketball. On arrival at the A. Loss of islet beta cells
emergency department, she is obtunded. On physical examination, she is Type 1 diabetes mellitus does not become overt until the beta cells are
hypotensive and tachycardic with deep, rapid, labored respirations. markedly depleted, and insulin levels are greatly reduced. In this case, the
Laboratory studies show serum Na+, 151 mmol/L; K+, 4.6 mmol/L; Cl–, 98 girl has ketoacidosis. Amyloid replacement of islets is a feature of type 2
mmol/L; CO2, 7 mmol/L; and glucose, 521 mg/dL. Urinalysis shows 4+ diabetes mellitus; ketoacidosis is not a feature of type 2 diabetes mellitus.
glucosuria and 4+ ketonuria levels, but no protein, blood, or nitrite. Which Acute or chronic pancreatitis diminishes exocrine pancreatic function, but
pathologic abnormality is most likely to be present in her pancreas at the rarely destroys enough islets to cause overt diabetes mellitus. Insulitis with
time of her collapse? inflammatory cells, mostly T cells, can be seen in the islets of patients with
type 1 diabetes mellitus before the diabetes is clinically overt. Eosinophils
A. Loss of islet beta cells are rare, however, with insulitis, but instead may be found in the islets of
B. Acute inflammation of islets diabetic infants who fail to survive the immediate postnatal period. A
C. Amyloid replacement of islet beta cells pancreatic neuroendocrine tumor may become hormonally active and can
D. Chronic inflammation of islets lead to hypoglycemia if insulin is produced in excess; production of
E. Hyperplasia of alpha cells glucagon by such an adenoma may lead to secondary diabetes mellitus
F. Pancreatic neuroendocrine tumor from the insulin-opposing effect of glucagon, but there is still insulin
secretion to prevent ketoacidosis. The most common finding with
glucagonoma is necrolytic migrating erythema. Alpha cell hyperplasia is
quite rare.

A study of patients more than 25 years of age with body mass index above B. Caloric restriction
30, dyslipidemia, hypertension, and fasting glucose averaging 115 mg/dL The findings are those of insulin resistance from obesity with metabolic
is performed. They have adipose tissue abnormalities including increased syndrome. Insulin resistance drives beta cell dysfunction, but other factors
nonesterified fatty acid release, altered adipokines with decreased such as the TCF7L2 gene play a role in eventual development of overt type
adiponectin, greater proinflammatory cytokine release, and diminished 2 diabetes mellitus. Excess free fatty acids may stimulate cytokine release
peroxisome proliferator-activated receptor gamma (PPARy) function. from beta cells to promote inflammation and islet cell dysfunction. Lifestyle
Which of the following is the best initial therapeutic intervention for these modification with dietary modification for weight reduction coupled with
patients? increased exercise will aid in reversing the insulin resistance so that no
drug therapy is needed to control hyperglycemia. Cushing syndrome may
A. Adrenalectomy occur from ACTH-independent adrenal cortical lesions, such as primary
B. Caloric restriction hyperplasia, adenoma, or carcinoma, and lead to secondary diabetes from
C. Insulin injection glucocorticoid-induced insulin resistance, but primary adrenal lesions are
D. L-Thyroxine less common than metabolic syndrome from obesity alone, and the study
E. Liposuction patients lack additional features of Cushing syndrome such as hirsutism,
osteoporosis, and easy bruisability. The absolute decrease of insulin with
type 1 diabetes mellitus must be treated with insulin injections. There is
modest weight gain with hypothyroidism, but without abnormalities of
adipocytes leading to insulin resistance. Liposuction is a plastic surgery
technique used for body contouring, not weight reduction.

❗USE AT YOUR OWN RISK❗ page 29 of 59


ROBBINS REVIEW

An infant is born following premature delivery. Multiple external congenital B. Diabetes mellitus, type 2
anomalies are noted. The infant exhibits a seizure soon after birth. The The findings are complications of diabetes with pregnancy, and the
blood glucose is 19 mg/dL. Which of the following maternal diseases is the malformations suggest that hyperglycemia preceded the pregnancy, and
most likely cause for the observed findings in this infant? type 2 diabetes is quite common, even now in women of childbearing age.
The neonatal hypoglycemia is a consequence of excessive islet beta-cell
A. Cystic fibrosis function from having been in a hyperglycemic environment. Though cystic
B. Diabetes mellitus, type 2 fibrosis is present from birth, the loss of pancreatic exocrine function takes
C. Gestational diabetes years, and loss of islets is a late finding. Gestational diabetes refers to
D. Maturity onset diabetes of the young glucose intolerance in pregnancy, and newborns are likely to have
E. Pancreatic neuroendocrine tumor hypoglycemia as a consequence of their own beta cell hyperfunction, but
not anomalies. Maturity-onset diabetes of the young (MODY; but not in
infancy) resembles type 2 diabetes and can occur from a variety of genetic
defects in pathways monitoring glucose levels, but is much less common.
Pancreatic neuroendocrine tumors are uncommon but could secrete
glucagon with secondary diabetes.

A clinical study is conducted in patients diagnosed with either type 1 or D. Nonenzymatic glycosylation of proteins
type 2 diabetes mellitus. Persons with either type develop complications Nonenzymatic glycosylation of proteins is a function of the level of blood
of accelerated and advanced atherosclerosis. All untreated patients have glucose, rather than the cause of hyperglycemia. This is the basis for an
an elevated hemoglobin A1c. Which of the following features common to elevated hemoglobin A1c. Type 1 and type 2 diabetes mellitus are
patients with either type 1 or type 2 diabetes mellitus is most likely to be characterized by hyperglycemia, but the underlying pathogenetic
found by this study? mechanisms are different. Type 1 diabetes mellitus is an autoimmune
A. Association with certain MHC class II alleles disease that is associated with certain alleles of the MHC class II
B. High concordance rate in monozygotic twins molecules. It is characterized by a very high concordance rate in twins and
C. Marked resistance to the action of insulin the presence of islet autoantibodies. Insulin resistance is a key feature of
D. Nonenzymatic glycosylation of proteins type 2 diabetes mellitus.
E. Presence of islet cell antibodies

A 50-year-old man with fasting blood glucose >140 mg/dL on two C. Glycosylated hemoglobin
occasions is put on a restricted caloric diet and started on a glucagon-like Nonenzymatic glycosylation refers to the chemical process whereby
peptide-1 (GLP-1) receptor agonist. Which of the following laboratory glucose attaches to proteins without the aid of enzymes. The degree of
studies is most likely to afford the best method of monitoring disease glycosylation is proportionate to the level of blood glucose. Many proteins,
control in this man? including hemoglobin, undergo nonenzymatic glycosylation.
Because RBCs have a lifespan of about 120 days, the amount of
A. Cholesterol, total glycosylated hemoglobin is a function of the blood glucose level over the
B. Fasting plasma glucose previous 120-day period. The level of glycosylated hemoglobin is not
C. Glycosylated hemoglobin appreciably affected by short-term changes in plasma glucose levels.
D. Microalbuminuria Random glucose testing is an immediate way for monitoring short-term
E. Random plasma glucose adjustments with diet and medications such as insulin and oral agents.
F. Serum fructosamine Fasting glucose testing affords a better way to diagnose diabetes mellitus
initially.
Measurements of cholesterol and fructosamine have no value in managing
diabetes mellitus. Microalbuminuria may presage the development of
diabetic renal disease. The “incretin effect” is diminished in patients with
type 2 diabetes, and use of GLP-1 receptor agonists can help to restore
incretin function and lead to improved glycemic control and loss of weight
via increased satiety.

A 50-year-old man has had a nonhealing ulcer on the bottom of his foot for A. Glucosuria
2 months. On examination, the 2-cm ulcer overlies the right first metatarsal Complications of diabetes mellitus are described in this man: vascular
head. There is reduced sensation to pinprick in his feet. His visual acuity disease, neuropathy, nephropathy, and retinopathy. Hyperglycemia
is reduced bilaterally. Laboratory studies show serum creatinine is 2.9 exceeds the capacity of renal tubular reabsorption, so glucose appears in
mg/dL. Which of the following laboratory test findings is he most likely to the urine. The other listed findings involve organs that are not typically
have? involved in diabetes mellitus: liver disease with decreased albumin
A. Glucosuria synthesis, hyperaldosteronism with hypokalemia, decreased marrow
B. Hypoalbuminemia function with leukopenia, exocrine pancreatic disease with steatorrhea, or
C. Hypokalemia D Leukopenia disordered uric acid
E. Steatorrhea metabolism.
F. Uricosuria

A 52-year-old man has been concerned about a gradual weight gain over A. Gangrene
the past 30 years. He is 174 cm (5 feet 7 inches) tall and weighs 91 kg Severe peripheral atherosclerotic disease is a common complication of
(body mass index 30). He is taking no medications. On physical long-standing diabetes mellitus. Atherosclerotic narrowing of the arteries
examination, he has decreased sensation to pinprick and light touch over to the lower legs can cause ischemia and gangrene. The foot is often
the lower extremities. Patellar reflexes are reduced. Motor strength seems involved with gangrene, which may necessitate amputation. Diabetic
to be normal in all extremities. Laboratory studies show blood glucose of neuropathy with decreased sensation increases the risk of repeated
169 mg/dL, creatinine of 1.9 mg/dL, total cholesterol of 220 mg/dL, HDL trauma, which enhances the risk of ulcerations that cause infection and
cholesterol of 27 mg/dL, and triglycerides of 261 mg/dL. A chest inflammation that promotes gangrene. Patients with type 2 diabetes
radiograph shows mild cardiomegaly. Five years later, he has claudication mellitus or obesity, or both, are at increased risk of developing nonalcoholic
in the lower extremities when he exercises. Based on these findings, which steatohepatitis. Because this patient is not taking medications such as
of the following complications is most likely to occur in this man? insulin, it is unlikely that he could become severely hypoglycemic. Because
A. Gangrene he is overweight, it is more likely that he has type 2 diabetes mellitus;
B. Hypoglycemic coma C Ketoacidosis ketoacidosis is unlikely. Infections with Mucor circinelloides are more likely
D. Mucormycosis to occur in ketoacidosis. Although pancreatic islets may have amyloid
E. Pancreatitis deposits, systemic amyloidosis and chronic pancreatitis (which involves
F. Systemic amyloidosis the parenchymal acini) do not occur with type 2 diabetes mellitus.

❗USE AT YOUR OWN RISK❗ page 30 of 59


ROBBINS REVIEW

A 45-year-old woman has had angina pectoris, polyuria, and polydipsia for C. Insulin resistance
the past 5 years. On physical examination, she has a body mass index of Nodular glomerulosclerosis, as shown in the figure, is a characteristic
32. Laboratory studies show her hemoglobin A1c is 10%. Urinalysis shows feature of renal involvement in diabetes mellitus and explains her
proteinuria, but no ketonuria. The representative microscopic appearance proteinuria (which may progress to nephrotic syndrome). Peripheral insulin
of her kidneys is shown in the figure. Which of the following is the most resistance is strongly linked to type 2 diabetes mellitus. Her history is
likely mechanism leading to the disease causing her findings? classic for type 2 diabetes, as is the elevated hemoglobin A1c. Note that
although premenopausal women are relatively protected from ischemic
A. Chronic pancreatitis heart disease, diabetes tilts the balance and can promote development of
B. Glucokinase gene mutation coronary artery disease in younger women. Chronic pancreatitis typically
C. Insulin resistance affects exocrine pancreatic function more than endocrine function.
D. Systemic amyloidosis Glucokinase gene mutations can lead to maturity onset diabetes of the
E. T-cell mediated B-cell destruction young (MODY), which is far less common than type 2 diabetes. Although
localized amyloid deposition in islets can be seen with type 2 diabetes, it
is not linked to conditions such as multiple myeloma that involve additional
organs with amyloid deposition. T-cell mediated destruction of islet beta
cells is a feature of type 1 diabetes

A 50-year-old man has a 35-year history of diabetes mellitus. During this F. Polyol-induced susceptibility to oxidative stress
time, he has had hemoglobin A1c values between 7% and 10%. He now Peripheral neuropathy, including autonomic neuropathy, can be caused by
has problems with sexual function, including difficulty attaining an erection. long-standing diabetes mellitus. It is thought that nerve cells do not require
He also is plagued by mild but recurrent low-volume diarrhea and difficulty insulin for glucose uptake. In the presence of hyperglycemia, excess
with urination. He has delayed gastric emptying. These problems are most glucose diffuses into the cell cytoplasm and accumulates. The excess
likely to originate from which of the following mechanisms of cellular injury? glucose is metabolized via the polyol pathway by intracellular aldose
reductase enzyme to sorbitol and then to fructose, a reaction that uses
A. Cross-linking of extracellular matrix proteins NADPH as a cofactor. NADPH is also required for a reaction that
B. Production of vascular endothelial growth factor regenerates reduced glutathione (GSH). GSH provides important
C. Abnormal transforming growth factor beta signaling antioxidant mechanisms in the cell. Thus persistence in intracellular
D. Increased endothelial procoagulant activity hyperglycemia through reduction of GSH makes the nerve cells
E. Nonenzymatic glycosylation susceptible to oxidative stress. Advanced glycosylation end products
F. Polyol-induced susceptibility to oxidative stress (AGEs) can directly cross-link extracellular matrix proteins to predispose
vessels to shear stress and endothelial injury. Glycosylation tends to affect
vascular walls and promote atherosclerosis. The downstream effects of
protein kinase C activation are numerous, including production of VEGF,
TGF-β, and the procoagulant protein plasminogen activator inhibitor-1 by
the vascular endothelium.

A 74-year-old woman is admitted to the hospital in an obtulty condition. D. Peripheral insulin resistance
Her temperature is 37° C, pulse is 95/ min, respirations are 22/min, and A complication of type 2 diabetes mellitus is hyperosmolar, nonketotic
blood pressure is 90/60 mm Hg. She appears dehydrated and has poor coma. In type 2 diabetes mellitus, the fundamental defect is insulin
skin turgor. Her serum glucose level is 872 mg/dL. Urinalysis shows 4+ resistance, leading to an eventual decrease in plasma insulin or a relative
glucosuria, but no ketones, protein, or blood. Which of the following factors lack of insulin, but there is still enough insulin to prevent ketosis. The
is most important in the pathogenesis of this patient’s condition? resulting hyperglycemia tends to produce polyuria, leading to dehydration,
which increases the serum glucose level further. If not enough fluids are
A. Autoimmune insulitis ingested, dehydration drives the serum glucose to very high levels.
B. Glucokinase gene mutation Glucokinase gene mutations can be present with maturity-onset diabetes
C. HLA-DR3/HLA-DR4 genotype of the young (MODY). The HLA-DR3/HLA-DR4 genotype is a predisposing
D. Peripheral insulin resistance factor for type 1 diabetes mellitus. Severe loss of beta cells with insulitis,
E. Virus-induced injury to beta cells in which may be triggered by viral infection, is a feature of autoimmune, or
islets type 1, diabetes mellitus.

A 40-year-old woman has experienced chest pain on exertion for the past A. Activation of PPAR nuclear receptor in adipocytes
2 months. A month ago, she had pneumonia with Streptococcus The clinical features of obesity with angina and glucose intolerance in this
pneumoniae cultured from her sputum. On physical examination, she has patient strongly suggest type 2 diabetes mellitus. This is confirmed by the
a body mass index of 35. A random blood glucose value is 132 mg/dL. oral glucose tolerance test (>200 mg/dL at 2 hours), useful in this case
The next day, a fasting blood glucose is 120 mg/dL, followed by a value of because her fasting blood glucose levels of 120 mg/dL and 122 mg/dL did
122 mg/ dL on the following day. She is given an oral glucose tolerance not quite reach the diagnostic criterion of 126 mg/dL. The fundamental
test, and her blood glucose is 240 mg/dL 2 hours after receiving the abnormality in type 2 diabetes mellitus is insulin resistance. Several
standard 75-g glucose dose. On the basis of these findings, she is adipocyte-derived molecules, such as adiponectin and resistin, have been
prescribed an oral thiazolidinedione (TZD) drug. After 2 months of therapy, implicated in the causation of insulin resistance, establishing the link
her fasting blood glucose is 90 mg/ dL. between obesity and type 2 diabetes mellitus. The nuclear receptor
peroxisome proliferator-activated receptor gamma (PPAR ) has emerged
The beneficial effect of TZD in this patient is most likely related to which of as a key molecule in the regulation of insulin resistance through its actions
the following processes? on adipocyte hormones. TZDs bind to and activate PPAR in adipocytes,
A. Activation of PPAR nuclear receptor in adipocytes and increase the levels of the insulin-sensitizing hormone adiponectin and
B. Decreased production of insulin autoantibodies reduce the levels of free fatty acids and resistin, both of which increase
C. Greater density of insulin receptors in adipocytes insulin resistance. Insulin autoantibodies are seen with type 1 diabetes
D. Increased half-life of circulating plasma insulin mellitus. Beta cell loss and density of insulin receptors are not major factors
E. Reduced secretion of glucagon by a cell in islets of Langerhans in the pathogenesis of type 2 diabetes mellitus. TZDs do not affect the
F Regeneration of beta cells in islets of Langerhans metabolism of insulin. Glucagon excess worsens diabetes, but TZDs do
not affect its secretion. Beta cells do not regenerate, but many anti
diabetogenic drugs in type 2 diabetes mellitus are designed to work with
the beta cells that are left.

❗USE AT YOUR OWN RISK❗ page 31 of 59


ROBBINS REVIEW

A family is followed longitudinally for two generations. Four of eight E. Loss of T-cell tolerance
children develop hyperglycemia by age 18 years. They are found to have These findings point to type 1 diabetes mellitus with childhood onset, MHC
serum islet autoantibodies. They have similar MHC I and MHC II loci. linkage, and evidence for autoimmune dysfunction mediated by T cells; the
Treatment with insulin injections normalizes their Hgb A1c levels. Which loss of islets leads to absolute lack of insulin, which requires insulin
of the following is the most likely mechanism leading to their disease? therapy. The remaining choices are not associated with autoimmunity.
Cystic fibrosis with CFTR gene mutations affecting chloride ion channels
A. Chloride ion channel abnormality may lead to chronic pancreatic acinar, and sometimes islet loss, in middle
B. Chromosome 21 trisomy age. Down syndrome may be accompanied by diabetes, but having
C. Glucokinase gene mutation multiple children involved is highly unlikely.
D. Peripheral insulin resistance
E. Loss of T-cell tolerance Glucokinase gene mutations lead to one form of maturity onset diabetes of
the young (MODY). Peripheral insulin resistance with obesity underlies
type 2 diabetes mellitus.

A 33-year-old woman has had several “fainting spells” over the past 6 D. Neuroendocrine tumor
months. Each time, she has a prodrome of light-headedness followed by The figure shows a circumscribed cellular lesion in the
a brief loss of consciousness. After each episode, she awakens and on pancreas, most suggestive of pancreatic neuroendocrine tumor (PanNET).
examination has no loss of motor or sensory function. Physical Secretion of insulin by these lesions causes hypoglycemia and the
examination after the current episode shows that she is afebrile, with a described symptoms. Many of these tumors are less than 1 cm in diameter,
pulse of 72/min, respirations of 17/min, and blood pressure of 120/80 mm making them difficult to detect. Most patients who have an insulin-secreting
Hg. Imaging studies reveal a 0.5-cm lesion in the head of the pancreas. PanNET have only mild insulin hypersecretion. The laboratory Finding an
The microscopic appearance of this lesion is shown in the figure. Which of increased insulin-to-glucose ratio is helpful. Surgical excision is necessary
the following pancreatic disorders is most likely to be present in this in patients with marked symptoms. Acute pancreatitis is unlikely to
patient? increase islet cell release of insulin. Adenocarcinomas of the pancreas are
derived from ductal epithelium and have no endocrine function. Fatty
A. Acute necrotizing pancreatitis replacement of the pancreas can occur with cystic fibrosis, but the number
B. Adenocarcinoma of islets also gradually diminishes. Pseudocysts are complications of
C. Cystic fibrosis pancreatitis
D. Neuroendocrine tumor that are focal and do not produce insulin hypersecretion
E. Pseudocyst

A 43-year-old man from Stockholm, Sweden, has had low-volume watery A. Gastrin
diarrhea for the past 3 months. He now has mid epigastric pain. Over-the- Zollinger-Ellison syndrome can be caused by one or more pancreatic
counter antacid medications do not relieve the pain. On physical neuroendocrine tumors (PanNETs) secreting gastrin (gastrinoma). This
examination, he is afebrile; on palpation, there is no abdominal tenderness secretion leads to intractable peptic ulcer disease, with multiple duodenal
and no masses. An upper gastrointestinal endoscopy shows multiple 0.5- or gastric ulcerations. The incidence is 1 to 3 per million in Sweden. Proton
to 1.1-cm, shallow, sharply demarcated ulcerations in the first and second pump inhibitors may help control the disease while evaluation for possible
portions of the duodenum. He is given omeprazole. Three months later, resection of tumors is undertaken. PanNETs may secrete various
repeat endoscopy shows that the ulcerations are still present. Which of the hormonally active compounds. Insulinomas may produce hypoglycemia.
following analytes is most likely to be increased in serum or plasma? Glucagonomas and somatostatinomas may produce a
syndrome characterized by mild diabetes mellitus. VIPomas may be
A. Gastrin associated with marked watery diarrhea, hypokalemia, and achlorhydria.
B. Glucagon
C. Insulin
D. Somatostatin
E. Vasoactive intestinal polypeptide (VIP)

A 39-year-old man has had a headache, weakness, and a 5-kg weight gain B Adrenal cortical adenoma
over the past 3 months. He has experienced mental depression during the The clinical and laboratory features of this case point to Cushing syndrome.
same period. On physical examination, his face is puffy. His temperature The dexamethasone suppression test is used to localize the source of
is 36.9° C and blood pressure is 160/75 mm Hg. He has cutaneous striae excess cortisol. When low-dose and high-dose dexamethasone trials fail
over the lower abdomen and ecchymoses scattered over the extremities. to suppress cortisol secretion, a pituitary corticotropin-secreting adenoma
A radiograph of the spine shows a compressed fracture of T11. Laboratory as the source of excess glucocorticoids is unlikely. The choice is an ectopic
findings show fasting plasma glucose level of 200 mg/dL, serum Na+ of source of corticotropin, such as a lung cancer, or a tumor of the adrenal
150 mmol/L, and serum K+ of 3.1 mmol/L. The cortex that is secreting glucocorticoids. The plasma corticotropin level
plasma cortisol level is 38 μg/dL at 8:00 am and 37 μg/dL at 6:00 pm. distinguishes between these two possibilities. Corticotropin levels are high
Administration of low and high doses of dexamethasone fails to suppress if there is an ectopic source, whereas glucocorticoid secretion from an
the plasma cortisol level and excretion of urinary 17- adrenal neoplasm suppresses corticotropin production by the pituitary,
hydroxycorticosteroids. The plasma corticotropin level is 0.8 pg/mL. Which leading to atrophy of the contralateral adrenal cortex. A
of the following lesions is most likely to be present in this man? pheochromocytoma secretes catecholamines, accounting for
hypertension, but not osteoporosis or the electrolyte changes noted.
A Adenohypophyseal adenoma Medullary thyroid carcinomas have neuroendocrine cells, but are unlikely
B Adrenal cortical adenoma to produce corticosteroids.
C Extra-adrenal pheochromocytoma
D Small cell carcinoma of the lung
E Thyroid medullary carcinoma

❗USE AT YOUR OWN RISK❗ page 32 of 59


ROBBINS REVIEW

A 43-year-old woman has had absent menstrual cycles along with A. Adrenal 10-cm solid mass with abdominal CT scan
increasing weakness and weight gain over the past 5 months. She notes The clinical findings of Cushing syndrome with masculinization suggest an
low back pain for the past week. On physical examination, vital signs adrenal cortical carcinoma, which is more likely to have endocrine function
include blood pressure of 155/95 mm Hg. She has a prominent fat pad in in women. A low ACTH level helps to rule out the possibility of an ACTH
the posterior neck and back. Facial plethora, hirsutism, and abdominal secreting pituitary adenoma in the sella turcica, or ectopic ACTH from a
cutaneous striae are present. Laboratory findings include Na+, 139 neoplasm such as lung carcinoma. A “cold” nodule of the thyroid gland can
mmol/L; K+, 4.1 mmol/L; Cl–, 96 mmol/L; CO2, 23 mmol/L; glucose, 163 represent a thyroid medullary carcinoma seen in multiple endocrine
mg/dL; creatinine, 1.3 mg/dL; calcium, 8.9 mg/ dL; and phosphorus, 4.1 neoplasia type 2 in association with adrenal pheochromocytomas, but
mg/dL. Her serum ACTH level is low. A radiograph of the spine shows Cushing syndrome is not part of this complex. The location of a mass at
decreased bone density with a compression fracture at T9. Which of the the aortic bifurcation could be the “infamous” extra-adrenal
following radiographic findings is most likely to be present in this patient? pheochromocytoma of the obscure organ of Zuckerkandl, which explains
hypertension with excess catecholamine release, but not the other features
A Adrenal 10-cm solid mass with abdominal CT scan of Cushing syndrome.
B Decreased radionuclide uptake in a thyroid gland nodule
C Pulmonary 6-cm hilar mass on chest radiograph
D Retroperitoneal 5-cm mass at the aortic bifurcation on pelvic MRI scan
E Sella turcica enlargement with erosion on head CT scan

A 73-year-old woman has experienced malaise and a 10-kg weight loss D. Metastatic carcinoma
over the past 4 months. She also has developed a chronic cough during The lung mass is likely a primary lung carcinoma, and the bilaterally
this time. She has a 100 pack-year history of smoking cigarettes. Physical enlarged adrenal glands can be explained by metastases to the adrenal
examination shows muscle wasting and 4/5 motor strength in all glands. Destruction of over 90% of the adrenal cortices is responsible for
extremities. Abdominal CT scan shows bilaterally enlarged adrenal glands. adrenal failure manifested by malaise and the low serum cortisol
A chest radiograph shows a 6-cm perihilar mass on the right and prominent concentration and the electrolyte disturbances. Amyloidosis can increase
hilar lymphadenopathy. Laboratory studies show Na+, 118 mmol/L; K+, 6 adrenal size, but does not produce a lung mass. In ectopic corticotropin
mmol/L; Cl– , 95 mmol/L; CO2, 21 mmol/L; and glucose, 49 mg/dL. Her syndrome, a lung cancer is likely finding, and typically the adrenal glands
8:00 am serum cortisol level is 9 ng/mL. What is the most likely diagnosis? are enlarged, but hypercortisolism also would be present. Waterhouse-
Friderichsen syndrome, caused by Neisseria meningitidis infection, can
A Amyloidosis increase the adrenal size secondary to marked hemorrhage (two to three
B Ectopic corticotropin syndrome times the normal size), but this does not explain the lung mass. This
C Meningococcemia syndrome has an abrupt onset. A pituitary adenoma that is secreting
D Metastatic carcinoma corticotropin could increase adrenal size bilaterally, but hypercortisolism
E Pituitary adenoma would occur.

A 28-year-old, otherwise healthy man has had headaches for the past 2 F. Low renin level
weeks. Physical examination yields no remarkable findings except for a The figure shows a circumscribed tumor that has arisen in the adrenal
blood pressure of 174/116 mm Hg. An abdominal CT scan shows an cortex. In young, otherwise healthy individuals who are hypertensive, a
enlarged right adrenal gland. A right adrenalectomy is done; the figure surgically curable cause of hypertension should be sought, because the
shows the gross appearance of the specimen. Which of the following far more common essential hypertension is found in an older patient
laboratory findings in his blood was most likely reported in this patient population. This patient had an adrenal cortical adenoma that secreted
before surgery? aldosterone (Conn syndrome). Hyperaldosteronism reduces the synthesis
of renin by the juxtaglomerular apparatus in the kidney. Adrenal adenomas
A Hyperglycemia can be nonfunctional or can secrete glucocorticoids or mineralocorticoids.
B Hyperkalemia Had this been a glucocorticoid-secreting adenoma, the patient could be
C Hyponatremia hypertensive, but he also would have some clinical features of Cushing
D Low corticotropin level syndrome. There is no effect on blood glucose or insulin levels. Patients
E Low insulin level with hyperaldosteronism have low serum potassium levels, and sodium
F Low renin level retention occurs. Aldosterone does not exhibit feedback suppression of the
anterior pituitary, and corticotropin levels are not affected.

A 40-year-old woman has experienced increasingly frequent episodes of A. Adrenal nodular enlargement
weakness accompanied by numbness and tingling in her hands and feet Hypokalemia with neuromuscular irritability, hypertension, and low plasma
for the past year. On examination her blood pressure is 168/112 mm Hg. renin suggests hyperaldosteronism. The most common cause for primary
Laboratory studies show sodium, 142 mmol/L; potassium, 2.9 mmol/L; hyperaldosteronism is idiopathic adrenal cortical nodular hyperplasia. An
chloride, 104 mmol/L; HCO3–, 28 mmol/L; and glucose, 74 mg/dL. Her insulinoma arising in the pancreas could account for episodic weakness,
plasma renin activity is low. Which of the following radiologic findings is but the glucose level would be low. About 10% of pheochromocytomas are
most likely to be present in this woman? extra-adrenal, including paraaortic, and could account for hypertension
from catecholamine excess, but there would not be hypokalemia. Thyroid
A Adrenal nodular enlargement enlargement could be Graves disease, though it is usually diffuse, and
B Pancreatic mass could account for weakness and hypertension, but with a wider pulse
C Retroperitoneal mass pressure, and without hypokalemia. A malignancy in the chest is more
D Thyroid nodular enlargement likely to be the cause for a paraneoplastic syndrome, but that is unlikely to
E Mediastinal mass be hypokalemia.

❗USE AT YOUR OWN RISK❗ page 33 of 59


ROBBINS REVIEW

A 5-year-boy has developed features that suggest puberty over the past 6 B. Cortical hyperplasia
months. On physical examination, the boy has secondary sex Adrenogenital syndrome can lead to precocious puberty, which is most
characteristics, including pubic hair and enlargement of the penis. Which commonly associated with a deficiency of 21- hydroxylase. The lack of this
of the following morphologic features is most likely to be seen in his enzyme reduces cortisol production, driving corticotropin production, which
adrenal glands? leads to adrenal hyperplasia and production of sex steroid hormones.
Bilateral adrenal cortical atrophy is typically seen in cases of Addison
A Cortical atrophy disease or after long-term exogenous glucocorticoid therapy. A nodule in
B Cortical hyperplasia the adrenal cortex that has zona glomerulosa cells produces primary
C Cortical nodule hyperaldosteronism; if it has zona fasciculata cells, it produces Cushing
D Medullary atrophy syndrome. Most adrenal nodules are nonfunctional and incidental findings.
E Medullary hyperplasia A nodule in the adrenal medulla, if functional, produces catecholamines,
F Medullary nodule and older patients with such nodules have hypertension. Medullary atrophy
is rare but might result from infections or toxins. Medullary hyperplasia is
uncommon but could also produce catecholamines.

A female infant is born at term to a 41-year-old Yupik woman after an C. 21-Hydroxylase


uncomplicated pregnancy. Soon after birth, the neonate develops A complete deficiency of 21-hydroxylase leads to the classic salt-wasting
hypotension. Physical examination shows ambiguous genitalia with a form of adrenogenital syndrome because the enzyme deficiency blocks
prominent clitoris. Laboratory studies show Na+, 131 mmol/L; K+, 5.1 formation of aldosterone and cortisol. Mutations may be deletions or
mmol/L; Cl–, 93 mmol/L; CO2, 18 mmol/L; glucose, 65 mg/dL; creatinine, duplications, or involve recombination between the CYP21 gene and a
0.4 mg/dL; testosterone, 50 mg/dL (normal <30 mg/dL); and cortisol, 2 pseudogene. These mutations may lead to severe deficiency with salt-
μg/dL. An abdominal ultrasound scan shows bilaterally enlarged adrenal wasting and prenatal virilization or partial deficiency with postnatal
glands. Which of the following enzyme deficiencies is most likely to be virilization, with an apparent autosomal recessive pattern of inheritance. A
present in this infant? deficiency of 11-hydroxylase blocks cortisol and aldosterone production as
well, although intermediate metabolites with some glucocorticoid activity
A Aromatase also are synthesized. Aromatase is involved with conversion of
B 11-Hydroxylase androstenedione to estrone, a pathway of steroid synthesis that does not
C 21-Hydroxylase affect cortisol production. A deficiency of 17-hydroxylase would lead to
D 17!-Hydroxylase reduction of both cortisol and sex steroid synthesis. Oxidase is the final
E Oxidase enzyme in the pathway to aldosterone production.

A 19-year-old, previously healthy woman collapsed after complaining of a D. Neisseria meningitidis


mild sore throat the previous day. On examination she is hypotensive and This is the typical adrenal finding in Waterhouse- Friderichsen syndrome,
febrile with purpuric skin lesions. Her peripheral blood smear shows and meningococcemia is the most likely cause of such a rapid course.
schistocytes. Imaging studies show her adrenal Chronic adrenocortical insufficiency can result from disseminated
glands are enlarged, and there are extensive bilateral cortical tuberculosis and from fungal infections, such as histoplasmosis, that
hemorrhages. Infection with which of the following organisms best involve the adrenal glands. Cytomegalovirus infections of the adrenals can
accounts for these findings? be seen in immunocompromised states and can be severe enough to
produce diminished adrenal function, although not acute failure.
A Cytomegalovirus Streptococcus pneumoniae can produce septicemia, but it is unlikely to
B Histoplasma capsulatum involve the adrenal glands specifically.
C Mycobacterium tuberculosis
D Neisseria meningitidis
E Streptococcus pneumoniae

A 29-year-old woman with systemic lupus erythematosus has been treated B. Atrophy
with corticosteroid therapy for several years because of recurrent lupus This woman has findings of acute adrenocortical Insufficiency (acute
nephritis. She undergoes an emergency appendectomy for acute Addisonian crisis). Long-term corticosteroid therapy shuts off corticotropin
appendicitis. On postoperative day 2, she becomes somnolent and stimulation to the adrenal glands, leading to adrenal atrophy. When this
develops severe nausea and vomiting. She then becomes hypotensive. history is not elicited, and the patient is not continued on the corticosteroid
Blood cultures are negative, and laboratory studies now show Na+ of 128 therapy, a crisis ensues, in this case made worse by the stress of surgery.
mmol/L, K+ of 4.9 mmol/L, Cl– of 89 mmol/L, CO2 of 19 mmol/L, glucose When tuberculosis is more prevalent and more severe without drug
of 52 mg/dL, and creatinine of 1.3 mg/dL. Which of the following therapy, dissemination to adrenals occurs more frequently. An adrenal
morphologic findings in the adrenal gland cortex is most likely to be present cortical adenoma without atrophy of the contralateral adrenal cortex could
in this patient? be a nonfunctioning adenoma or an aldosterone- secreting adenoma. If the
contralateral cortex is grossly atrophic, the adenoma on the opposite side
A Adenoma is probably secreting excess glucocorticoids. A carcinoma is most likely to
B Atrophy destroy one adrenal, be nonfunctioning, and leave the remaining adrenal
C Carcinoma intact. Addison disease from granulomatous destruction of the adrenals
D Caseating granulomas was more common, but this is a chronic process. Hemorrhagic necrosis
E Hemorrhagic necrosis suggests Waterhouse-Friderichsen syndrome, which can complicate
F Nodular hyperplasia septicemia with organisms such as Neisseria meningitidis. Cortical nodular
hyperplasia can be driven by an ACTH-secreting pituitary adenoma, or it
can be idiopathic; in either case, hypercortisolism ensues, not Addison
disease

❗USE AT YOUR OWN RISK❗ page 34 of 59


ROBBINS REVIEW

A 55-year-old man has experienced increasing lethargy for the past 7 C. Hashimoto thyroiditis
months. Physical examination shows hyperpigmentation of the skin. Vital Addison disease (primary chronic adrenocortical insufficiency) most often
signs include temperature of 36.9° C, pulse of 70/min, respirations of results from an idiopathic autoimmune condition (in areas of the world
14/min, and blood pressure of 95/65 mm Hg. Laboratory studies include a where the incidence of active tuberculosis is low). Autoimmune adrenalitis
serum cortisol level of 3 μg/mL at 8:00 AM with a serum corticotropin level is associated with the appearance of other autoimmune diseases in about
of 65 pg/mL. Which of the following diseases most often occurs in patients half of all cases. Such autoimmune phenomena are frequently seen in
with this disorder? other endocrine organs, such as the thyroid gland. Other presumed
autoimmune diseases, such as systemic lupus erythematosus, ulcerative
A Type 2 diabetes mellitus colitis, and the vasculitides, are usually not forerunners to adrenal failure,
B Classic polyarteritis nodosa although treatment of these conditions with corticosteroids can lead to
C Hashimoto thyroiditis iatrogenic adrenal atrophy. Type 2 diabetes mellitus, Unlike type 1, it does
D Systemic lupus erythematosus not have an autoimmune basis.
E Ulcerative colitis

A 44-year-old woman has become increasingly listless and weak and has B. Autoimmune destruction of the adrenals
had chronic diarrhea and a 5-kg weight loss over the past 7 months. She Chronic adrenal insufficiency (Addison disease) results in decreased
also notices that her skin seems darker, although she rarely goes outside cortisol production and decreased mineralocorticoid activity. The skin
because she is too tired for outdoor activities. On physical examination, hyperpigmentation results from increased corticotropin precursor hormone
she is afebrile, and her blood pressure is 85/50 mm Hg. A chest radiograph production, which also stimulates melanocytes. The most common cause
shows no abnormal findings. Laboratory findings include serum Na+, 120 of Addison disease, in areas where tuberculosis is not endemic, is
mmol/L; K+, 5.1 mmol/L; glucose, 58 mg/dL; urea nitrogen, 18 mg/dL; and autoimmune adrenalitis. This process causes gradual destruction of the
creatinine, 0.8 mg/ dL. The serum corticotropin level is 82 pg/mL. Which adrenal cortex, mediated most likely by infiltrating lymphocytes. A
of the following is most likely to account for these findings? pancreatic neuroendocrine tumor (islet cell adenoma) secreting insulin
could account for hypoglycemia, but not for the other metabolic changes.
A Adenohypophyseal adenoma Metastases occasionally can destroy enough adrenal cortex to cause
B Autoimmune destruction of the adrenals adrenal insufficiency, but the most common primary site is the lung, and
C Pancreatic neuroendocrine tumor there is no lung mass in her chest radiograph. Bilateral hemorrhages and
D Metastatic carcinoma with lung primary resultant destruction of the adrenal glands are typically caused by
E Neisseria meningitidis infection of the Adrenals meningococcemia, and this manifests as acute adrenocortical
F Sarcoidosis of the lung and adrenals insufficiency. Sarcoidosis also may involve the adrenals, but this is less
common, and in this woman’s case, the normal chest radiograph helps to
eliminate this possibility because hilar adenopathy is almost always
present in sarcoidosis.

A 37-year-old woman states that, although most of the time she feels fine, E. Increased urinary free catecholamines
she has had episodes of palpitations, tachycardia, tremor, diaphoresis, These findings suggest a pheochromocytoma of the adrenal medulla. This
and headache over the past 3 months. When her symptoms are worse, is a rare neoplasm, but in cases of episodic hypertension, this diagnosis
her blood pressure is measured in the range of 155/90 mm Hg. She should be considered. Screening for urinary free catecholamines,
collapses suddenly one day and is brought to the hospital, where her metanephrine, and vanillylmandelic acid (VMA) can help to determine the
ventricular fibrillation is converted successfully to sinus rhythm. On diagnosis. Up to 25% of cases may be associated with an underlying tumor
physical examination, there are no remarkable findings. Which of the suppressor gene mutation, such as RET, NF1, or VHL. The level of HVA
following laboratory findings is most likely to be reported in this patient? is more likely to be increased in a neuroblastoma, which is a tumor that
occurs in children. The serum cortisol is increased with Addison disease,
A Decreased serum cortisol level which is accompanied by hypotension. Hypoglycemia can also occur in
B Decreased serum glucose level Addison disease, as well as islet cell tumors. The serum potassium level
C Decreased serum potassium level can be decreased with aldosterone secreting adrenal adenomas. An
D Increased serum free T4 level increased T4 level occurs in patients with Graves disease; this disease can
E Increased urinary free catecholamines cause weight loss, heat intolerance, anxiety, tachycardia, tremors,
F Increased urinary homovanillic acid (HVA) level and cardiac arrhythmia

A 25-year-old woman gives birth to a term infant F. Urinary homovanillic acid (HVA) level
following an uncomplicated pregnancy. On physical examination, the Neuroblastomas are neoplasms that occur in children and may be
newborn is found to have an enlarged abdomen, but there are no other congenital. They arise most commonly in the retroperitoneum in the
abnormal findings except for slightly elevated blood pressure. An adrenal glands or in extra-adrenal paraganglia. They are primitive small
abdominal ultrasound scan shows a right retroperitoneal mass in the blue cell tumors that can produce high levels of catecholamine precursors
adrenal gland. An increase in which of the following substances is most and their metabolites such as HVA and vanillylmandelic acid (VMA). HVA
likely to be associated with the lesion in this neonate? is most specific. Because neuroblastomas are usually unilateral,
abnormalities of ACTH or cortisol production are unlikely. Neuroblastomas
A Serum corticotropin (ACTH) level B Serum cortisol level do not affect serum prolactin or growth hormone levels. Adult
C Serum growth hormone level pheochromocytomas are more likely to be detected by increased urinary
D Serum prolactin level free catecholamines, but the primitive cells of neuroblastoma are unlikely
E Urinary free catecholamine level to produce amounts of catecholamines in the range of
F Urinary homovanillic acid (HVA) level pheochromocytomas.

❗USE AT YOUR OWN RISK❗ page 35 of 59


ROBBINS REVIEW

A 27-year-old man has headaches that have occurred frequently for the A. Bilateral 4-cm medullary masses
past 3 months. On physical examination, he is afebrile, and his blood These findings suggest multiple endocrine neoplasia (MEN) type 2A
pressure is 140/85 mm Hg. There are no neurologic abnormalities and no (Sipple syndrome) or possibly MEN type 2B (Williams syndrome). These
visual defects; however, he has an enlarged thyroid. Laboratory studies patients have medullary carcinomas of the thyroid, pheochromocytomas,
show that his serum calcitonin level is elevated. A total thyroidectomy is and parathyroid adenomas. This patient’s headaches could be caused by
performed, and on sectioning, the thyroid has multiple tumor nodules in hypertension from a pheochromocytoma arising in the adrenal medulla.
both lobes. Microscopically, the thyroid nodules are composed of nests of More than 70% of cases of pheochromocytomas are bilateral when
neoplastic cells separated by amyloid-rich stroma. The endocrinologist familial. Medullary carcinoma also tends to be multifocal in this syndrome.
says that the patient’s family members could be at risk for development of This syndrome is associated with germline mutations in the RET proto-
similar tumors and advises that they undergo genetic screening. Which of oncogene. Family members who inherit the same mutation are at
the following morphologic findings in the adrenal glands is most likely to increased risk of developing similar cancers.
be present in this patient?
Genetic screening followed by increased surveillance of affected family
A Bilateral 4-cm medullary masses members is advised. Bilateral cortical atrophy from autoimmune
B Bilateral cortical atrophy destruction of the adrenals, leading to bilateral cortical atrophy, is now the
C Bilateral cortical nodular hyperplasia D Solitary caseating granuloma most common cause of Addison disease. Cortical nodular hyperplasia can
E Solitary 1-cm cortical mass with contralateral corti-cal atrophy be driven by an ACTH-secreting pituitary adenoma, or it can be idiopathic;
F Solitary 12-cm hemorrhagic cortical mass in either case, hypercortisolism ensues, not Addison disease.

Granulomatous destruction of the adrenal glands suggests disseminated


tuberculosis as a cause of Addison disease, which leads to adrenal
insufficiency if bilateral, not unilateral. An adrenal cortical adenoma with
atrophy of the contralateral adrenal cortex could be secreting excess
glucocorticoids. A large mass with hemorrhage and necrosis in an adrenal
suggests a cortical carcinoma.

A 26-year-old man developed sudden severe abdominal pain. On physical C. Pituitary adenoma
examination, he had marked abdominal tenderness and guarding. Multiple endocrine neoplasia (MEN) type 1 is also known as Wermer
Laboratory studies showed serum glucose, 76 mg/dL; calcium, 12.2 syndrome. (Remember the “three P’s” in neoplasia or hyperplasia—
mg/dL; phosphorus, 2.6 mg/dL; creatinine, 1.1 mg/dL; and parathyroid pancreas, pituitary, and parathyroids.)
hormone, 62 pg/mL (normal range 9 to 60 pg/mL). During surgery, four
enlarged parathyroid glands were found and excised, with reimplantation Adrenal pheochromocytomas are associated with MEN 2B. Endometrial
of one half of one gland. After the surgery, his serum calcium concentration carcinomas can arise in patients who have unopposed estrogen secretion,
returned to normal. Three years later, he had an episode of upper which can occur in estrogen-producing ovarian tumors. These are not part
gastrointestinal hemorrhage. An endoscopy and biopsy specimen showed of MEN 1. Small cell carcinomas of the lung are known for various
multiple benign gastric ulcerations. Abdominal MRI indicated multiple 1- to paraneoplastic syndromes, but not usually hypercalcemia. It also is
2-cm mass lesions in the pancreas. He underwent surgery, and multiple doubtful that this patient would have lived 5 years with a small cell
tumors were found. Which of the following additional neoplasm lesions is carcinoma. If her hypercalcemia had been a paraneoplastic syndrome, the
he most likely to have? parathyroid glands would not have been enlarged, and the serum calcium
level would not have returned to normal after surgery. Medullary thyroid
A Adrenal pheochromocytoma carcinomas are part of MEN 2A or 2B.
B Endometrial carcinoma
C Pituitary adenoma
D Pulmonary small cell anaplastic carcinoma
E Thyroid medullary carcinoma

A 10-year-old boy has been bothered by frequent headaches for the past E. Pineoblastoma
5 months. Physical examination yields no specific findings. Laboratory The anatomic location of the mass is the pineal gland. In children, the most
studies show normal electrolyte levels. CT scan of the head shows no bony common pineal tumor is a pineoblastoma, whereas in adults, it is a
abnormalities and no intracranial hemorrhage. MRI of the brain shows a 2- pineocytoma. Both are quite rare, and their location makes them difficult to
cm solid mass without calcifications or cystic change in the area inferior to remove completely. Craniopharyngiomas are aggressive neoplasms that
the splenium of the corpus callosum, superior to the collicular plate, and are often suprasellar and difficult to remove. Hypothalamic gliomas also
between the right and left thalamic pulvinar regions. Because of the are suprasellar. Central nervous system lymphomas are uncommon in
location, the mass is difficult to remove completely. Which of the following children and rare at this site. Metastatic carcinoma is rare in any location
neoplasms is most likely to be present in this child? in children because children do not have many malignancies, and the
malignancies that they do have are often not carcinomas. A prolactinoma
A Craniopharyngioma occurs in the sella turcica.
B Hypothalamic glioma
C Lymphoblastic lymphoma
D Metastatic carcinoma
E Pineoblastoma
F Prolactinoma

❗USE AT YOUR OWN RISK❗ page 36 of 59


ROBBINS REVIEW

CENTRAL NERVOUS SYSTEM

QUESTION RATIONALE

1. In a study of hypoglycemic shock, cellular changes in the brain are D. Neurons


analyzed. One cell type in the hippocampus is noted to exhibit intense Hypoxic and hypoglycemic injury leads to “red” neurons as the initial
cytoplasmic eosinophilia, central chromatolysis, spheroidal reaction. The larger pyramidal neurons are the most sensitive, particularly
swellings, and nuclear pyknosis. These changes appear 12 hours after in the hippocampus with hypoglycemia, as well as neocortical Betz cells
blood glucose levels drop below 20 mg/dL. What is this cell type most likely and cerebellar Purkinje cells.
to be?
A. Astrocytes ● Astrocytes proliferate in reaction to brain injury in the process
B. Endothelial cells called gliosis.
C. Microglia ● Endothelial cells are most likely to proliferate in neoplasms.
D. Neurons ● Microglia respond to brain injury with a macrophage-like
E. Oligodendroglia function.
● Oligodendroglial cells provide myelination for brain neuronal
axons

2. A 75-year-old man has a history of transient attacks of loss of vision. D. Microglia


The only abnormalities on physical examina- tion are bruits over the ● Microglial cells are part of the body’s fixed macrophage system,
carotids in the neck. Two days later he suddenly becomes hemiplegic and derived originally from mesoderm.
loses consciousness. He is rushed to the emergency room and a CT scan ● Microglia can respond to cerebral injuries by taking on a
shows evidence of cerebral infarction. He is put on life support but dies 5 macrophage-like function.
days later. At autopsy there is an area of necrosis and microscopically ● The arachnoid layer is part of the meninges covering the brain.
these lesions are noted to have increased numbers of cells distributed ● Astrocytes can proliferate in response to brain injuries, a
around the central zone of necrosis. Which of the following cell types is process called gliosis, but there is no phagocytosis.
most likely to have a phagocytic function in these lesions? ● Ependymal cells line the ventricular system containing CSF.
● Oligodendroglial cells provide the myelin sheaths for neuronal
A. Arachnoidal cells axons in the CNS.
B. Astrocytes
C. Ependymal cells
D. Microglia
E. Oligodendroglia

3. A 49-year-old woman has had a severe headache for 2 days. On C. Glioblastoma with edema
physical examination, she is afebrile and normotensive. Funduscopic The papilledema and the herniation are a consequence of brain swelling,
examination shows papilledema on the right. One day later, she has right typically the vasogenic form of edema from blood-brain barrier disruption
pupillary dilation and impaired ocular movement. She then becomes adjacent to the neoplasm. A large aggressive neoplasm, such as a
obtunded. Which of the following lesions best explains these findings? glioblastoma, may produce a mass effect via enlargement from rapid
growth, hemorrhage, and surrounding edema. The mass effect with
A. Chronic subdural hematoma herniation of the medial temporal lobe results in a third cranial nerve palsy
B. Frontal lobe abscess as the nerve is compressed.
C. Glioblastoma with edema ● A chronic subdural hemorrhage accumulates slowly enough that
D. Hydrocephalus ex vacuo herniation may not occur.
E. Occipital lobe infarction ● An abscess may cause a mass effect with some associated
F. Ruptured middle cerebral berry aneurysm brain swelling, but this patient is afebrile.
● There is no pressure effect with hydrocephalus ex vacuo, which
is a consequence of cere- bral atrophy.
● An infarct is not likely to produce pronounced associated brain
379 swelling, and a mass effect in the occipital lobe is unlikely
to affect the third nerve.
● Rupture of a berry aneurysm produces subarachnoid
hemorrhage at the base of the brain, which is less likely to cause
a mass effect.

4. A 16-year-old boy with no prior medical problems has complained of D. Ependymoma


headaches for the past 9 months. There are no abnormal findings on This is noncommunicating hydrocephalus with obstruction below the
physical examination. CT scan of the head shows enlargement of the level of the third ventricle.
lateral cerebral ventricles and third ventricle. A lumbar puncture is ● If hydrocephalus had been present at birth, there would be
performed with normal opening pressure, and clear CSF is obtained, which increasing head size because the sutures are not yet closed,
has a slightly elevated protein, normal glucose, and no leukocytes. Which and congenital aqueductal stenosis would be suspected.
of the following intracranial lesions is most likely to cause these findings? ● At his age, a neoplasm should be suspected, and
A. Aqueductal stenosis ependymomas arise in the ventricular system, often in the fourth
B. Cerebral abscess ventricle, to cause obstruction of CSF flow.
C. Cryptococcal meningitis ● The increased CSF protein comes from this tumor, but shedding
D. Ependymoma of cells from the mass into the CSF is unlikely. Except for
E. Multiple sclerosis vascular malformation, the other options are uncommon at his
F. Vascular malformation age.
● An abscess is typically accompanied by fever, and most would
be located in the cerebral hemispheres away from the
ventricular system.
● Cryptococcal meningitis is accompanied by fever, and exudate
can be found within the ventricular system and subarachnoid
space, but there is more likely to be cerebral edema, not
hydrocephalus. The demyelinating plaques of multiple sclerosis

❗USE AT YOUR OWN RISK❗ page 37 of 59


ROBBINS REVIEW

are small and do not usually act as mass lesions.


● Vascular malformations usually arise in the cerebral
hemispheres.

5. A 61-year-old man has had worsening mental function with F. Pneumococcal meningitis
confusion for the past year, along with headaches. At first the He has communicating hydrocephalus, and pneumococcal meningitis in
headaches occurred in the morning, but for the past 3 months they have adults often involves the vertex, where the arachnoid granulations that
become continuous, along with nausea and blurred vision. On physical reabsorb CSF are found. Overproduction of CSF from a choroid plexus
examination there is bilateral papilledema. A head CT scan shows papilloma may also produce hydrocephalus, but is rare.
enlargement of the entire ventricular system. Which of the following Non-communicating hydrocephalus occurs when there is obstruction
prior illnesses most likely led to his current problems? within the ventricular system. Both aqueductal stenosis and Dandy-
A. Aqueductal stenosis Walker malformation are congenital conditions causing ventricular system
B. Choroid plexus papilloma obstruction.
C. Cerebral infarction Cerebral infarcts and HIV infection produce lesions within the brain
D. Dandy-Walker malformation parenchyma, and are unlikely to obstruct the CSF flow unless there is a
E. HIV infection mass effect (from edema or hemorrhage with the infarction, or a CNS
F. Pneumococcal meningitis lymphoma with AIDS), but no mass was noted in this man’s CT scan.

6. A 67-year-old woman has had new onset headaches with nausea for A. Left inferior frontal lobe
the past month. She now has a worsening headache with weakness in Her contralateral right leg weakness is a consequence of subfalcine
her right leg. On physical examination she has 4/5 motor strength herniation with compression of the ipsilateral anterior cerebral artery
involving her right leg. Extraocular muscle movements are intact. In which caused by a lesion in the adjacent frontal lobe. A rapidly expanding mass
of the following locations is a neoplasm most likely to be found in this lesion can lead to herniation. The other listed options, above the tentorium,
woman? are much less likely to explain anterior cerebral artery compression.
A. Left inferior frontal lobe Frontal lobe lesions might account for transtentorial herniation. A very
B. Left lateral temporal lobe large mass effect above the tentorium could also produce tonsillar
C. Left superior occipital lobe herniation
D. Right inferior occipital lobe
E. Right medial temporal lobe
F. Right superior frontal lobe

7. An 81-year-old man with a history of poorly controlled atrial fibrillation B. Cytotoxic edema
suddenly collapses while watching television at home. Emergency medical Cerebral edema may be cytotoxic or vasogenic. In cytotoxic edema the
services arrive promptly, but multiple attempts at cardioversion over 15 blood-brain barrier remains intact, and edema is due to failure of ATP-
minutes are required to reestablish a stable pulse and blood pressure. dependent ion transport with cellular retention of sodium and water. Global
Over the next day he develops bilateral papilledema, and an MRI of his processes such as ischemia (from heart failure in this case) or metabolic
brain shows an indistinct cortical gray-white junction and narrowing of derangements are implicated, and consistent with diffuse swelling with
ventricles. Which of the following intracranial abnormalities most likely bilateral papilledema. Inflammation and neoplasms are more likely to
developed in this man? produce vasogenic edema. An acute meningitis might produce diffuse
A. Acute inflammation swelling, but there are no signs of infection here. Metastases are unlikely
B. Cytotoxic edema to produce rapidly expanding mass lesions. Obstruction of CSF flow leads
C. Metastatic adenocarcinoma to hydrocephalus with ventricular enlargement, not narrowing.
D. Obstruction of CSF flow Subarachnoid hemorrhage is most likely a consequence of a ruptured
E. Subarachnoid hemorrhage berry aneurysm, without a mass effect

8. A 45-year-old man develops a severe B. Herniation


headache and fever over 2 days. On physical The figure shows linear midline hemorrhages, called Duret hemorrhages,
examination, he has nuchal rigidity and in the pons. The acute bacterial meningitis led to brain swelling with edema
bilateral papilledema. His temperature is 38.5° and subsequent herniation of medial temporal lobe with Duret
C. A blood culture shows gram-positive cocci in hemorrhages in the pons. Although not seen in this figure, an infection
chains, and Streptococcus pneumoniae is could organize with scarring of foramina to produce a noncommunicating
identified. The figure shows the representative hydrocephalus, or it could scar the vertex and impair reabsorption of CSF
gross appearance of a section of his brain. Based at the arachnoid granulations to produce a communicating hydrocephalus.
on this appearance, which of the following An abscess infre- quently complicates meningitis; conversely, an abscess
complications most likely resulted from this in a paranasal sinus or mastoid air cell may extend into the cranial cavity
patient’s infection? to cause meningitis. Laminar necrosis could occur after brain death, but
this finding is not specific for meningitis. The small meningeal vessels do
not often bleed because of inflammation caused by meningitis.
A. Abscess formation
B. Herniation
C. Hydrocephalus
D. Laminar cortical necrosis
E. Subarachnoid hemorrhage

9. A 30-year-old woman, G3, P2, is in the third B. Elevated serum a-fetoprotein level
trimester of pregnancy. She has noted Anencephaly is a form of severe neural tube defect that results from
minimal fetal movement throughout the failure of formation of the fetal cranial vault. This is one of the most
pregnancy. A fetal ultrasound scan shows common CNS malformations seen at birth. The defect allows fetal-
normal amniotic fluid volume, normally fetoprotein to enter amniotic fluid and reach the maternal circulation. The
implanted placenta, and the abnormality karyotype listed is that of a Robertsonian Down syndrome carrier; Down
shown in the figure. Which of the following syndrome (trisomy 21) may be associated with brachycephaly, but rarely
laboratory findings is most likely to be present with anencephaly. Congenital cytomegalovirus infection can produce
in this woman? extensive fetal brain parenchymal necrosis, but not loss of the fetal cranial
vault. Neural tube defects are not associated with maternal or neonatal
jaundice. Diabetes mellitus, suggested by an elevated hemoglobin A1c

❗USE AT YOUR OWN RISK❗ page 38 of 59


ROBBINS REVIEW

A. 45, XX, t(14;21)(p11;q11) karyotype concentration, can increase the risk of malformations (e.g.,
B. Elevated serum !-fetoprotein level holoprosencephaly in the CNS), but not neural tube defects
C. High cytomegalovirus IgM titer
D. Hyperbilirubinemia with anemia
E. Increased hemoglobin A1c level

10. A 22-year-old primigravida had a fetal screening ultra- sound study at D. Holoprosencephaly
18 weeks showing a single large cerebral ventricle and fused thalami. Holoprosencephaly is a midline defect in which there is absent (alobar) or
On physical examination at birth at 36 weeks’ gestation, the infant is small partial (semilobar) cerebral hemispheric development. It can occur in
for gestational age and has multiple anomalies, including postaxial trisomy 13, as in this case, with other midline defects. It also may be seen
polydactyly of hands and feet, cyclopia, microcephaly, cleft lip and in cases of maternal diabetes mellitus. Anencephaly is the absence of a
palate, and rocker-bottom feet. The infant dies 1 hour after birth. Which fetal cranial vault, which leads to absence of most of the brain. Arnold-
of the following CNS abnormalities best explains these findings? Chiari II malformation results in a small posterior fossa, a misshapen
A. Anencephaly midline cerebellum with downward displacement of the vermis, and tenting
B. Arnold-Chiari II malformation of the tectal plate. Dandy-Walker malformation is characterized by aplasia
C. Dandy-Walker malformation or hypoplasia of the cerebellar vermis, cystic enlargement of the fourth
D. Holoprosencephaly ventricle, and hydrocephalus. Periventricular leukomalacia is a form of
E. Periventricular leukomalacia perinatal injury that is caused by hypoxic-ischemic events or infections.

11. A 24-year-old man incurs head and neck trauma in a motor vehicle E. Syringomyelia
accident. He now has impaired pain and temperature sensation from The syrinx is a tubular defect that extends laterally, disrupting the
the shoulders down to his feet, but proprioception and vibratory spinothalamic tracts and anterior horns, producing sensory and motor
sense is preserved. He has motor weakness with muscle atrophy starting deficits. When the cavity extends into the medulla, it is called
in his hands and extending to forearms and shoulders. An MRI of the syringobulbia. There is no treatment, but the lesion tends not to progress.
cervical spinal cord shows a transverse slit-like cavity extending from the Dandy-Walker malformation is a congenital condition with an enlarged
level of C2 to C7. What is the most likely diagnosis? posterior fossa. Diffuse axonal injury involves white matter tracts in the
A. Dandy-Walker malformation cerebrum. Rachischisis is a large open neural tube defect. Spina bifida is
B. Diffuse axonal injury a posterior neural tube defect
C. Rachischisis
D. Spina bifida
E. Syringomyelia

A. Arnold-Chiari II malformation
12. A 15-year-old girl has had Arnold-Chiari II malformation has features including in a small posterior
progressive difficulty speak- ing during fossa, misshapen midline cerebellum with downward displacement
the past 6 months. She becomes dizzy of the vermis, and tenting of the tectal plate, leading to
and falls frequently. She complains of hydrocephalus. MRI in this case is characteristic of hydromyelia. Cerebral
headache and facial and neck pain. palsy is a general term describing nonprogressive motor deficits that are
During the past month, she has had present from birth. The corpus callosum is seen in the figure; agene- sis
decreasing bladder and bowel control. may be associated with other anomalies, or may be found in normal
On physical examination, there is loss persons. Holoprosencephaly is a severe malforma- tion with total (alobar)
of pain and temperature sensation or incomplete (semilobar) separation of the cerebral hemispheres in brain
over the nape of the neck, shoulders, development. Polymicrogyria is characterized by numerous small,
and upper arms, but vibration and irregularly formed gyral contours
position sensation are preserved.
She has muscle wasting in the lower
neck and shoulders. MRI of the spinal
cord shows cervical and thoracic
enlargement with a CSF collection
dilating the central canal. MRI of the
brain shows gross findings similar to
those shown in the figure. Which of the
following is the most likely diagnosis?
A. Arnold-Chiari II malformation
B. Cerebral palsy
C. Corpus callosal agenesis
D. Holoprosencephaly
E. Polymicrogyria

13. A neonate is born prematurely at 28 weeks’ gestation to a 22-year-old C. Germinal matrix hemorrhage
primigravida. The infant is initially stable, and a newborn physical Germinal matrix hemorrhage is the most common cause of intraventricular
examination shows no abnormalities. The infant becomes severely hemorrhage in premature infants. The germinal matrix, composed of highly
hypoxic 24 hours later, and seizure activity is observed. There is poor vascularized tissue with primitive cells, is most prominent between 22 and
neurologic development during infancy. CT scan of the head shows 30 weeks’ gestation. Hemorrhages within this area read- ily occur with
symmetrically enlarged cerebral ventricles at 8 months of age. Which common neonatal problems such as hypoxemia, hypercarbia, acidosis,
of the following perinatal complications most likely produced these and changes in blood pressure. Hemorrhages in the germinal matrix can
findings? extend into the cerebral ventricles and from there into the subarachnoid
A. Congenital cytomegalovirus infection space. Smaller hemorrhages can resolve without sequelae. With larger
B. Down syndrome hemorrhages, organization of the blood in the aqueduct of Sylvius or the
C. Germinal matrix hemorrhage fourth ventricle or foramina of Luschka and Magendie may obstruct the
D. Kernicterus flow of CSF, producing hydrocephalus. Cytomegalovirus infection can
E. Medulloblastoma cause considerable necrosis of the brain parenchyma, particularly in a
periventricular location, but not hemorrhage with obstruction of CSF flow.
Infants with Down syndrome may have vascular malformations that can
bleed into the parenchyma. The bilirubin staining of kernicterus does not

❗USE AT YOUR OWN RISK❗ page 39 of 59


ROBBINS REVIEW

14. A 21-year-old woman incurs a blow to her head from a fall while B. Concussion
mountain biking. She then has a loss of consciousness for 5 minutes. Concussion leads to altered consciousness, starting with instantaneous
On examination her deep tendon reflexes are diminished. A head CT scan onset of transient neurologic dysfunction with head trauma, amnesia for
6 hours later shows no abnormalities. She recovers over the next week, the event, then neurologic recovery or post-concussion syndrome with
with no neurologic deficits, but cannot remember this event. During the disabling neuropsychiatric manifestations, worsening with repeated
next year she had irritability, headache, difficulty sleeping, trouble concussions. There are no radiologic or pathologic findings.
concentrating, and fatigue. Which of the following is the most likely Arteriolosclerosis is typically seen with hypertension. Hemorrhage
consequence from her injury? following the traumatic event could be organized to obstruct CSF flow or
A. Arteriolosclerosis absorption, but this would have been identified with the original CT scan.
B. Concussion The term leukoencephalopathy applies to degenerative white matter
C. Hydrocephalus changes, typically with meta- bolic and infectious insults. Post-traumatic
D. Leukoencephalopathy dementia from repeated blows to the head often has pathologic findings of
E. Post-traumatic dementia neurofibrillary tangles and neuritic plaques similar to Alzheimer disease.
F. Myelinolysis

15. An 83-year old woman slips in the bathtub in her home and falls E. Inferior Frontal Lobe
backward, striking her head. She is taken to the emergency department, This patient has the classic “contrecoup” type of injury, in which the
where examination shows a 3-cm reddish, slightly swollen area over moving head strikes an object, and the force is transmitted to the opposite
the occiput. She is arousable but somnolent. There are no motor or side of the head. A fall backward is most likely to produce contusions to
sensory deficits. There is no papilledema. CT scan of the head is the inferior frontal lobes, temporal tips, and inferior temporal lobes. A blow
performed. Acute hemorrhage in which of the following locations is most to a stationary head is more likely to produce a “coup” injury directly
likely to be seen? adjacent to the site of the blow. Basal ganglia putaminal hemorrhage is
A. Basal ganglia most likely to occur in hypertension. Hemorrhage into the pons is typical
B. Basis pontis of a Duret hemorrhage, seen in medial temporal lobe herniation.
C. Cerebral ventricle Hemorrhage into the cerebral ventricles may occur in premature infants
D. Epidural space from germinal matrix hemorrhage; in adults, it is uncommon, but may occur
E. Inferior frontal lobe with dissection of blood from an intraparenchymal lesion. A blow to the
F. Sella turcica side of the head is more likely to lacerate the middle meningeal artery and
produce an epidural hematoma. Subarachnoid hemorrhage at the base of
the brain may dissect into the sella.

16. A 19-year-old snowboarder wearing protective equipment consisting C. Middle meningeal artery laceration
of a baseball cap, baggy shorts, and a flak jacket flew off a jump and hit The “lucid” interval is a classic feature of an epidural hematoma with
a tree. He was initially unconscious, and then “came to” and wanted rapid accumulation of blood from the injured middle meningeal
to try another run, but his friends thought it best to call for help. On the artery. The dura is pushed against the brain, producing the lens-shaped
way to the emergency department, he became comatose. Physical blood collection seen with radiologic imaging, particularly CT imaging.
examination now shows left papilledema. Skull radiographs show a linear Leptomeningitis is not associated with trauma. Contusions do not
fracture of the left temporoparietal region. This clinical picture is most progressively worsen. In an acute subdural hematoma or a ruptured
consistent with which of the following lesions? aneurysm, there typically is no lucid interval, but instead a sudden and
A. Acute leptomeningitis progressive worsening of symptoms
B. Contusion of frontal lobes
C. Middle meningeal artery laceration
D. Ruptured berry aneurysm
E. Tearing of cerebral bridging veins

17. A 72-year-old woman trips and falls down the stairs. She does not lose E. Subdural
consciousness. She develops a headache and confusion 30 hours later A subdural hematoma results from tearing of the bridging veins beneath
and is taken to the emergency department. On physical examination, she the dura. These veins are at risk of tearing with head trauma, particularly
is conscious and has a scalp contusion on the occiput. What is the most in elderly individuals, in whom some degree of cerebral atrophy may be
likely location of an intracranial hemorrhage in this patient? present. Bleeding from low-pressure veins produces a variable time
course for appearance of signs and symptoms, from hours to days to
A. Basal ganglia weeks.
B. Epidural ● Basal ganglia hemorrhages are most often associated with
C. Pontine hypertension. Epidural hemorrhages are most often preceded
D. Subarachnoid by a blow to the head that tears the middle meningeal artery;
E. Subdural there is commonly a “lucid” interval between an initial loss of
consciousness occurring with trauma and the later accumulation
of blood.
● Pontine hemorrhages are likely to be Duret hemorrhages.
● Subarachnoid hemorrhage could occur in contusions with
trauma.

18. An 80-year-old resident of a nursing home is admitted to the hospital D. Tearing of the cerebral bridging veins
because of recent onset of fluctuating levels of consciousness with Tearing of bridging veins leads to an acute subdural hematoma; this
headache and confusion for the past 2 days. On physical examination, she almost always results from head trauma in which there has been a fall,
is arousable, but disoriented and irritable. Vital signs include temperature which may have been minor and gone unnoticed. The risk of hemorrhage
of 36.9° C and blood pressure of 130/85 mm Hg. There is papilledema on is greater in elderly individuals because of cerebral atrophy, which leaves
the right. CT scan of the head shows a collection of blood in the subdural the bridging veins beneath the dura at the vertex more vulner- able to
space on the right. Which of the following vascular lesions most likely traumatic tearing. Arteriovenous (vascular) malformations are often
produced these findings? located within the parenchyma of a hemisphere, and bleeding from them
A. Bleeding from an arteriovenous malformation occurs more often in young adults. When saccular (berry) aneurysms
B. Laceration of the middle meningeal artery rupture, the bleeding is typically subarachnoid and at the base of the brain.
C. Rupture of a saccular aneurysm A tear of the middle meningeal artery can occur in head trauma (e.g., a
D. Tearing of the cerebral bridging veins blow to the head), but it results in an acute epidural hematoma.
E. Thrombosis of the middle cerebral artery Thrombosis of an intracranial artery may result in an infarction, which can

❗USE AT YOUR OWN RISK❗ page 40 of 59


ROBBINS REVIEW

be hemorrhagic, but the hemorrhage typically does not extend into


subarachnoid or subdural locations.

19.A 22-year-old man is caught in a rip current off Cabo San Lucas. He C. Macrophages
becomes tired and overcome by the waves. Lifeguards get him to shore, Global cerebral ischemia has occurred in this man, leading to brain death.
but he has no pulse. Resuscitative measures over the next 20 minutes There is ongoing liquefactive necrosis, and in 2 to 3 weeks there will be
establish a pulse. However, he does not regain consciousness. A month numerous macrophages that phagocytize the cellular debris. These
later an electroencephalogram (EEG) shows no brain wave activity. Which macrophages can persist for months. The neocortex has more vulnerable
of the following cells most likely predominated in the cerebral cortex at the pyramidal cell layers with a pattern of pseudolaminar necrosis. Fibroblastic
time of this EEG? proliferation with collagen deposition is uncommon in the CNS, except
A. Fibroblasts around organizing abscesses. An abscess is where neutrophils would be
B. Lymphocytes found, as well as meninges with acute meningitis. Inflammation of brain
C. Macrophages parenchyma is most often lymphocytic. Oligodendrocytes provide
D. Neutrophils myelination to axons. Dying red neurons are seen after 12 hours following
E. Oligodendrocytes ischemic injury and may persist for days, but not weeks.
F. Red neurons

20. A 68-year-old woman with atrial B. Embolic arterial occlusion


fibrillation suddenly lost consciousness This cerebral infarction is a large hemorrhagic and softened area of
and fell to the ground. When she beginning liquefactive necrosis after vascular injury in the distribution of
became arousable, she was unable to the middle cerebral artery. Emboli likely emanates from the heart. It takes
move her left arm and had dif- ficulty weeks to months for macrophages to clear the debris of liquefactive
speaking. On physical examination, her necrosis and leave a cystic space. A vascular malformation has irregular
temperature was 37° C, pulse was vascular channels that give a mass effect with a dark red-to-bluish
81/min, respirations were 18/min, and appearance. Metastases may be solitary masses but are most often
blood pressure was 135/85 mm Hg. The multiple, and may have central hemorrhage or necrosis, but are often
figure shows the representative gross multiple. An organizing subdural hematoma would leave a uniform area of
appearance of her brain in radiologic ori- compression with flattening of the underlying hemisphere. A contusion
entation. An MRI 3 months later shows could produce some minimal focal loss of cortex with brown hemosiderin
a cystic space. The development of staining.
such a lesion most likely resulted from
which of the following conditions?

A. Arteriovenous malformation
B. Embolic arterial occlusion
C. Metastatic carcinoma
D. Organizing subdural hematoma
E. Superficial cortical contusion

21.A 79-year-old man with metabolic syndrome has had 6 episodes of B. Cerebral atherosclerosis
sudden dysarthria, a feeling of weakness in his hand, and dizziness in the The brief episodes of neurologic dysfunction repre- sent transient ischemic
past 3 months. These episodes usually last less than 1 hour, and then he attacks (TIAs) and are a prodrome to stroke in many cases. Atherosclerotic
feels fine. Today, he suddenly lost consciousness while walking to the cerebrovascular disease is a common antecedent to cerebral infarction,
bathroom in his house and fell to the floor. On regaining consciousness 4 and metabolic syndrome with dyslipidemia and hyperglycemia is a risk
minutes later, he was unable to move his right arm. Which of the following factor. A vascular malformation most often produces symptoms caused by
underlying lesions is most likely to be found in his brain? bleeding in young adults. A neoplasm is unlikely to produce such sudden,
A. Arteriovenous malformation episodic symptoms and signs. Meningoencephalitis may produce general
B. Cerebral atherosclerosis features such as fever, headache, confusion, and seizures, but not sudden
C. Frontal lobe astrocytoma localizing signs. A subdural hematoma, which most frequently results from
D. Meningoencephalitis head trauma sustained in a fall, is unlikely to develop in a few minutes and
E. Subdural hematoma would not explain the TIAs

22. A study is conducted to identify causes of neuronal loss in patients 18 D. Global hypoxia
to 90 years old who died in the hospital from a natural manner of death The neurons that are most sensitive to anoxia reside in the hippocampus,
and who had autopsies performed. Subsequent microscopic examination along with the cerebellar Purkinje cells and the larger neocortical neurons.
of sections revealed red, shrunken neurons, decreased numbers of In addition, the first areas in the neocortex to be affected are the
neurons, or absent neurons. The hippocampal pyramidal cells, the “watershed” areas between the three major cerebral circulations, including
cerebellar Purkinje cells, and the superior parasagittal neocortical the watershed located superiorly between the anterior cerebral and middle
pyramidal cells are affected. What condition is most likely to be the major cerebral circulations, as in this study. “Red” shrunken neurons, especially
cause of neuronal loss in these patients? in the areas mentioned, are typically seen in the early stages of global
A. Autoimmunity hypoxia, as may occur in a severe hypotensive episode. Focal hypoxia
B. Chemotherapy leads to infarcts. Autoimmunity can lead to vasculitis and subsequent
C. Diabetes mellitus hypoxemia. Chemotherapy tends to damage actively dividing cells more
D. Global hypoxia severely. Diabetes mellitus causes ath- erosclerosis, which can lead to
E. Lead ingestion hypoxemia. Lead poisoning leads to encephalopathy, not ischemia. Poor
F. Poor nutrition nutrition with a deficiency of thiamine (vitamin B1) can lead to Wernicke
disease, which affects the mammillary bodies and periaque- ductal gray
matter most severely

23. A 70-year-old woman had an episode 2 days earlier during which she E. Hypercholesterolemia
lost consciousness for several minutes. On physical examination, there is An acute cerebral infarction results from obstruction of blood flow causing
4/5 motor strength in the right upper extremity and decreased sensation to focal cerebral ischemia. After 2 days, there would be some cerebral
pinprick on the right arm and hand. There are bilateral carotid bruits. CT softening and edema with ischemia of neurons, but little else.
scan of the head shows no intracranial hemorrhage, but there is a slight Hypercholesterolemia is a risk factor for atherosclerosis, which is the major

❗USE AT YOUR OWN RISK❗ page 41 of 59


ROBBINS REVIEW

midline shift; MRI of the brain shows edema near the left internal capsule. cause of thrombotic cerebral arterial occlusions. Antiphospholipid
A lumbar puncture is performed with normal opening pressure. Laboratory syndrome can produce thrombotic and embolic disease, but an embolic
studies on 10 mL of clear, colorless CSF show two mononuclear “stroke” is typically hemorrhagic, and antiphospholipid syndrome is
WBCs/mm3, no RBCs, protein concentration of 40 mg/dL, and glucose uncommon at this age. A positive blood culture suggests sepsis with the
con- centration of 70 mg/dL. The serum glucose concentration is 95 mg/dL. possibility of meningitis or cerebral abscess formation, but abscesses
Which of the following laboratory findings is most suggestive of the risk typically have ring enhancement on CT scans. Elevated serum levels of
factor for her disease? very long chain fatty acids are present in patients with adrenoleu-
A. Antiphospholipid antibody kodystrophy, a rare disorder that leads to myelin loss at an early age.
B. Blood culture positive for Streptococcus pneumoniae Hyperammonemia occurs in hepatic encephalopa- thy with liver failure; it
C. Elevated serum concentration of very long chain fatty acids produces Alzheimer type II gliosis, but no focal or gross lesions.
D. Hyperammonemia Neurosyphilis is now rare; it does not produce focal ischemic lesions
E. Hypercholesterolemia
F. Positive serologic test for syphilis

24. A 59-year-old woman had sudden loss of consciousness 4 months E. Ischemic heart disease with left ventricular thrombosis
ago. On physical examination, she now has left hemiplegia. CT imaging A remote cerebral infarction is present. Thromboembolic disease with
shows a large, cystic space in the right parietal region. MR angiography cerebral infarction most often results from a cardiac disease (e.g.,
shows occlusion of a peripheral cerebral artery branch at the gray-white endocarditis, mural thrombo- sis, prosthetic valvular thrombosis).
junction near the lesion. What underlying disease process is this woman Hemorrhagic infarcts are more likely to occur when emboli partially occlude
most likely to have? a vessel or undergo dissolution. Thrombosis over atheroscle- rotic lesions
A. AIDS with a low CD4+ T-lymphocyte count is more likely to cause non-hemorrhagic infarcts. AIDS is not often
B. Chronic alcoholism with micronodular cirrhosis associated with significant cardiovascular or cerebrovascular diseases.
C. Chronic renal failure with hypertension Head trauma with hemorrhage is more common in individuals with chronic
D. Colonic adenocarcinoma with Trousseau syndrome alcoholism. Hypertension is most often associated with basal ganglia,
E. Ischemic heart disease with left ventricular thrombosis pontine, and cerebellar hemorrhages and with small lacunar infarcts.
Malignant neoplasms may be associated with para- neoplastic syndromes,
including hypercoagulability and thrombosis, but this is much less common
than heart disease as a cause for cerebrovascular disease and stroke.

25. A 39-year-old man presents with headache and altered mental status D. Immunosuppressives
of 60 hours’ duration. On examination he is afebrile and normotensive and Primary angiitis of the CNS is uncommon. It involves small peripheral
has a reduced level of consciousness with aphasia. Lumbar puncture is vessels. The cause is unknown, but it appears to be immunologically
performed and the CSF obtained has a lymphocytic pleocytosis with mediated, so immunosuppressive therapy is of benefit, such as
modest protein elevation. An MRI of the brain shows focal hyperintense cyclophosphamide and methylprednisolone. Infections are unlikely to
cortical lesions. A stereotaxic brain biopsy shows chronic inflammation produce such focal involvement of small vessels. Hypertensive
with granulomas involving arterioles and venules. Which of the following encephalopathy produces fibrinoid necrosis of arterioles. NSAIDs or
therapeutic options is most appropriate for this man? aspirin as anti-inflammatory or antipyretic agents are not strong enough to
A. Antibiotics address the inflammatory process in this case. Mannitol infusion is used
B. Anti-hypertensives to treat marked cerebral edema, but that is not the major finding in this
C. Antipyretics agents case.
D. Immunosuppressives
E. Mannitol infusion

26. A 55-year-old man suddenly loses D. Systemic hypertension


consciousness while driving his truck, but he is Hypertensive hemorrhages are most likely to arise in the basal ganglia
traveling at a slow speed and comes to a stop (shown in the figure), thalamus, cerebral white matter, pons, or cerebellum.
without a collision. Paramedics arrive but are Multiple hemorrhages are uncommon. The small vessels weakened by
unable to arouse him. On physical examination, hyaline arteriolosclerosis are prone to rupture. Chronic alcoholism
there is bilateral papilledema. He has no predisposes to falls with subdural hematomas or contusions. Metastases
spontaneous movements. The figure shows the are typically multiple and peripheral at the graywhite junction. The plaques
gross appearance of the brain at autopsy. What of multiple sclerosis occur in white matter and do not bleed.
underlying condition is most likely to have Thromboemboli can produce ischemic cerebral infarctions, sometimes
resulted in this lesion? with hemorrhage, and they usually involve the cerebral cortex.

A. Chronic alcoholism
B. Metastatic carcinoma
C. Multiple sclerosis
D. Systemic hypertension
E. Thromboembolism

27. A 72-year-old man has had poorly controlled hypertension for the past A. Arteriolar fibrinoid necrosis
20 years. Over the past day he has had a severe headache with nausea, He developed acute hypertensive encephalopathy from the rapid and
followed by confusion, then convulsions. On examination he is afebrile, but marked increase in blood pressure. There is extensive cerebral edema
his blood pressure is now 260/150 mm Hg. There is bilateral papilledema. with increased intracranial pressure, but no localizing signs. The arterioles
Which of the following pathologic lesions is most likely to have developed are involved, and like malignant hypertension involving the kidney, there is
in his brain during the past day? fibrinoid necrosis and petechial hemorrhages. Telangiectasias may be one
A. Arteriolar fibrinoid necrosis form of vascular malformation, typically a localized lesion not related to
B. Cortical telangiectasia hypertension. Lacunar infarcts develop with hypertension, but they tend to
C. Lacunar infarction be small and are often silent, but may cause significant neurologic
D. Putaminal hematoma impairment, though more likely a focal deficit due to focal involvement.
E. Subarachnoid hemorrhage Hypertensive hemorrhages may be large, and the basal ganglia region is
a common location, but such a lesion could produce a unilateral mass
effect with papilledema on one side. Subarachnoid hemorrhages are more
likely to occur from ruptured saccular aneurysms.

❗USE AT YOUR OWN RISK❗ page 42 of 59


ROBBINS REVIEW

28. A 72-year-old man with diabetes mellitus has had step- wise cognitive A. Lacunes
decline for the past 5 years. On multiple occasions he has had an acute This patient’s stepwise loss of cognitive function is consistent with vascular
event, such as loss of consciousness or confusion, followed by worsening dementia, and his history of diabetes mellitus makes underlying heart
ability to perform activities of daily living. On physical examination, he has disease with embolic events, or vascular disease involving the brain, more
mild right hemiparesis, ataxia, and dysarthria. Which of the following likely. Some cases have diffuse cortical laminar necrosis from global
pathologic findings is most likely to be present and numerous in this man? hypoxic events, whereas others have multiple lacunar infarcts, and still
A. Lacunes others have embolic infarcts. Lewy body dementia mimics Alzheimer
B. Lewy bodies disease (AD), with progressive cognitive decline, and a parkinsonian
C. Neuritic plaques movement disorder is often pres- ent. Neuritic plaques and neurofibrillary
D. Neurofibrillary tangles tangles are charac- teristic for AD, which has progressive cognitive
E. Pick bodies decline. Pick bodies are seen with Pick disease, which is similar to AD.
F. Plaques of demyelination Plaques of white matter demyelination are seen with multi- ple sclerosis,
and the gray matter is relatively spared, so that severe cognitive
impairment is not a usual feature

29. An 86-year-old man has become progressively unable to live A. Aggregation of peptide
independently for the past 10 years, and he now requires assistance with Alzheimer disease (AD) can be complicated by cere- bral amyloid
bathing, dressing, toileting, feeding, and trans- fers in and out of chairs and angiopathy and terminal hemorrhagic stroke. Formation and aggregation
bed. On physical examination, he has no motor or sensory deficits. He of the Aà peptide is now consid- ered central to the pathogenesis of AD.
cannot give the cur- rent date or state where he is. Six months later, he Aà peptide is derived from abnormal processing of amyloid precursor
suddenly becomes comatose and dies. At autopsy, there is a large super- protein (APP). When APP, a transmembrane protein, is cleaved by !-
ficial left parietal lobe hemorrhage. Histologic examination of the brain secretase within the Aà sequence, followed by -secretases, a soluble
shows numerous neocortical neuritic plaques and neurofibrillary tangles. nontoxic fragment is formed. Cleavage of the Aà sequence by à-secretase
The peripheral cerebral arteries and the core of each plaque stain and then - secretase gives rise to Aà peptides that aggregate and form the
positively with Congo red. Which of the following mechanisms is most likely amyloid cores that elicit a microglial and astrocytic response to form
responsible for his disease? neuritic plaques. Conformational change in prion protein leads to
A. Aggregation of peptide Creutzfeldt-Jakob disease, a rapidly progressive dementia with
B. Conformational change in the prion protein (PrP) spongiform encephalopathy but not neuritic plaques or amyloid deposition.
C. Dopamine deficiency Loss of dopaminergic neurons with deficiency of dopamine is central to the
D. Expansion of polyglutamine repeats pathogenesis of Par- kinson disease. Expansion of polyglutamine repeats
E. Mutations in the tau gene owing to CAG trinucleotide repeat-expansion underlies Hunting- ton
disease. Although abnormally phosphorylated forms of tau protein are
found in neurofibrillary tangles seen in AD, there is no mutation of the tau
gene, and the tangles are not considered primary in the pathogenesis of
AD. Mutation of the tau gene can be found in frontotemporal lobe
dementias.

30. A 50-year-old woman develops a sudden, severe headache and is D. Rupture of an intracranial berry aneurysm
taken to the emergency department. On examination, she has nuchal About 1 in 50 individuals has a saccular (berry) aneurysm. Although this
rigidity. Her blood pressure is 115/83 mm Hg. A lumbar puncture is done; lesion is present at birth as a congenital defect in the arterial media at
the CSF shows numerous RBCs, no neutrophils, a few mononuclear cells, intracerebral arterial branch points, it can manifest later in life with
and a normal glucose level. The Gram stain result is negative. CT imaging aneurysmal dilation and possible rupture. These aneurysms are the most
shows subarachnoid hemorrhage at the base of the brain. Which of the common cause of spontaneous subarachnoid hemorrhage in adults. The
following vascular events has most likely occurred in this woman? bleeding from amyloid angiopathy is in peripheral cortex, and most likely
A. Bleeding from cerebral amyloid angiopathy to occur in association with Alzheimer disease. A hypertensive
B. Hematoma formation from arteriolosclerosis hemorrhage from arteriolosclerosis tends to remain within the brain
C. Middle cerebral artery thromboembolism parenchyma. Thromboemboli can cause infarctions, most often in the
D. Rupture of an intracranial berry aneurysm distribution of the middle cerebral artery in cortex, and embolic infarcts can
E. Tear of subdural bridging veins be hemorrhagic, but the blood typically does not reach the CSF. A subdural
hematoma over the brain surface results from a tear of bridging veins.

31. A 45-year-old, previously healthy man has developed headaches over D. Subarachnoid hemorrhage
the past month. There are no remarkable findings on physical examination. Intracranial aneurysms are typically saccular and enlarge slowly over time.
A cerebral MR angiogram shows a 7-mm saccular aneurysm at the Aneurysms that grow to 4 to 7 mm are at the greatest risk of rupture.
trifurcation of the right middle cerebral artery. Which of the following is the Rupture occurs into the subarachnoid space at the base of the brain,
most likely complication from this lesion? where the cerebral arterial distribution originates around the circle of Willis,
A. Cerebellar tonsillar herniation and where saccular aneurysms are most likely to arise. Neither a berry
B. Hydrocephalus aneurysm nor the bleeding that results is likely to cause a mass effect and
C. Epidural hematoma herniation. In some cases of survival after rupture of a berry aneurysm, a
D. Subarachnoid hemorrhage noncommunicating hydrocephalus results from organization of the
E. Subdural hematoma subarachnoid hemorrhage occluding foramina of Luschka and Magendie.
Epidural hematomas arise from a tear of the middle meningeal artery,
typically as a result of head trauma. Trauma also can cause a tear of
bridging veins that produces a subdural hematoma.

32. A 25-year-old man has complained of headaches for the past 5 B. Arteriovenous malformation
months. During that time, family members noticed that he was not as Arteriovenous (vascular) malformations most often occur in the cerebral
mentally sharp as he has been in the past, and that he has become more hemisphere of a young adult and can often be completely resected without
emotionally labile. Over a 2-week period, he has 4 generalized seizures. complication. There may be slow leakage of blood from the lesion over
On physical examination, he now has no papilledema or movement time, resulting in the clinical symptoms and the gliosis seen on biopsy in
disorder. CT scan of the head shows a 2-cm mass in the right frontal lobe. this case. An angiosarcoma with malignant-appearing cells is not a primary
A stereotactic biopsy specimen of this lesion shows only gliosis and lesion in the brain parenchyma. A plaque of demyelination in multiple
evidence of recent and remote hemorrhage. The mass is removed, and sclerosis can appear as a mass, but its features include loss of myelin with
histologic examination shows a conglomerate of various-sized tortuous gliosis and macrophages, not vascular abnormalities. An abscess would
vessels surrounded by gliosis. What is the most likely diagnosis? have an organizing wall with collagen and gliosis, but no prominent larger

❗USE AT YOUR OWN RISK❗ page 43 of 59


ROBBINS REVIEW

A. Angiosarcoma vessels. Head trauma generally produces contusions and hematomas on


B. Arteriovenous malformation the surface, but not hemorrhages in the brain parenchyma. A ruptured
C. Multiple sclerosis plaque aneurysm can extend upward into the parenchyma in some cases; the
D. Organizing abscess outcome is always fatal with this complication.
E. Prior head trauma
F. Ruptured saccular aneurysm

33.A 19-year-old man has a sore throat C. Neisseria meningitidis


followed a day later by sudden onset of a Acute meningitis, with a purulent exudate on the cerebral convexities
severe headache. Physical examination shown in the figure, is indicative of bacterial infection. At his age, a
shows mild pharyngitis and nuchal rigidity. common etiologic agent is Neisseria meningitidis, initially presenting as
His skin shows petechial hemorrhages. His pharyngitis, and untreated proceeding to Waterhouse-Friderichsen
temperature is 38.8° C, pulse is 98/min, syndrome with disseminated intravascular coagulopathy. Cryptococcosis
respirations are 26/min, and blood pressure should be considered in immunocompromised patients, but some cases
is 95/45 mm Hg. The figure shows the occur in immunocompetent patients; a meningoencephalitis can occur.
representative gross appearance of the Tuberculous meningitis does not manifest so acutely, and the exudate is
surface of his brain. Which of the following typically on the base of the brain. Poliomyelitis could occur after
infectious organisms is most likely to have pharyngitis, but onset is insidious, with increasing paralysis from loss of
produced his disease? motor neurons. It does not cause meningitis. Cerebral toxoplasmosis may
A. Cryptococcus neoformans occur in immunocompromised patients, but the lesions are parenchymal
B. Mycobacterium tuberculosis abscesses, not meningitis.
C. Neisseria meningitidis
D. Poliovirus
E. Toxoplasma gondii

34. A 44-year-old woman who is an intravenous drug user is admitted to D. Staphylococcus aureus
the hospital with increasing headache and high fever for the past 24 hours. Headache, fever, pronounced neutrophilia, a high CSF protein level, and
On physical examination, her temperature is 38.4° C, pulse is 85/min, a low glucose concentration all point to bacterial meningitis.
respirations are 18/min, and blood pressure is 125/85 mm Hg. CT scan of Staphylococcus aureus is a common infection among injection drug users.
the head shows no mass lesion or midline shift. A lumbar puncture is Herpes simplex virus produces encephalitis, not meningitis. Tuberculous
performed. The CSF shows 70,000 neutrophils/mm3, an increased protein meningitis has a more insidious onset. This patient does not have a mass
concentration, and a decreased glucose level. Which of the following lesion of toxoplasmosis or focal lesions of progressive multifocal
infectious agents is most likely to produce these findings? leukoencephalopathy (PML) that may be associated with AIDS. PML is
A. Herpes simplex virus associated with the JC polyomavirus. Toxoplasmosis, which may occur in
B. JC polyomavirus immunocompromised patients, produces parenchymal abscesses, not
C. Mycobacterium tuberculosis meningitis.
D. Staphylococcus aureus
E. Toxoplasma gondii

35. A 77-year-old man has been irritable for the past 2 days. He is C. Gram-positive cocci
otherwise healthy. On physical examination, he has a temperature of 39.1° The most common etiologic organism for acute bacterial meningitis in his
C. Laboratory examination of the CSF from a lumbar puncture shows age group, because of vaccinations, is now Streptococcus pneumoniae, a
numerous neutrophils, slightly increased protein level, and decreased gram-positive coccus. Pneumococci also are likely to be seen in an adult.
glucose concentration. On Gram staining of the CSF, which of the following The number of CNS infections with Haemophilus influenzae, a gram-
is most likely to be seen microscopically? negative bacillus, in this age group has decreased because of widespread
A. Gram-negative bacilli immunization. The gram-negative diplococci of Neisseria meningitidis are
B. Gram-negative diplococci seen in young adults. The gram-negative bacilli of Escherichia coli are
C. Gram-positive cocci most often seen in neonates. The short, gram-positive rods of Listeria
D. Gram-positive short rods monocytogenes appear sporadically or in epidemics caused by food
E. No organisms contamination. With these clinical and CSF features, it would be unusual
if bacteria were not present in the CSF

36. A 43-year-old woman has had a headache and fever for the past 2 A. Cerebral abscess
weeks following a severe respiratory tract infection accompanying A cerebral abscess is most often a complication of an infection, such as
bronchiectasis. On physical examination, her temperature is 38.3° C. pneumonia or endocarditis, with onset days to weeks earlier. The bacteria
There is no papilledema. She has no loss of sensation or motor function, spread hematogenously. As the abscess organizes, it is ringed by
but there is decreased vision in the left half of her visual fields. CT scan of fibroblasts that deposit collagen; this feature is characteristic of an abscess
the head shows a sharply demarcated, 3-cm, ring-enhancing lesion in the in the CNS. A neoplasm occasionally may be ring enhancing, but a
right occipital region. A lumbar puncture is done, and laboratory analysis glioblastoma is an aggressive malignancy that is not well demarcated.
of the CSF shows numerous leukocytes, increased protein, and normal Metastases are mass lesions that are typically multifocal. A multiple
glucose levels. What is the most likely diagnosis? sclerosis plaque is generally not large, is found in white matter, and does
A. Cerebral abscess not typically have ring enhancement. An infarct would produce sudden
B. Glioblastoma signs and symptoms that improve over time, and the CSF protein would
C. Metastatic carcinoma not be increased
D. Multiple sclerosis
E. Subacute infarction

37.An 11-year-old boy has had pain in his right ear for 1 week and a severe B. Epidural abscess
headache for 1 day. On physical examination his temperature is 37.5° C. Acute otitis media may be complicated by spread of the infection to the
He has marked tenderness on palpation posterior to the right ear. Pus mastoid air cells, and then to skull bone, and then to the epidural space.
exudes from the right tympanic membrane, and Streptococcus This also may be termed pachymeningitis or epidural empyema. Spread
pneumoniae is cultured. Which of the following intracranial complications to the underlying meninges may next occur. Encephalitis involves the sub-
is he most likely to develop if untreated? stance of the brain. Multicystic encephalopathy is a severe complication of
A. Encephalitis perinatal brain injury with infarction. Subdural hematomas are most often
B. Epidural abscess caused by trauma. The tri- geminal nerve is located at a distance from the

❗USE AT YOUR OWN RISK❗ page 44 of 59


ROBBINS REVIEW

C. Multicystic encephalopathy mastoid air cells so that it is unlikely to be involved; but the facial nerve
D. Subdural hematoma adjacent to the ear may be involved
E. Trigeminal neuralgia

38. A 4-year-old girl residing near Cape Town, South Africa, has had a C. Mycobacterium tuberculosis
worsening headache and irritability for the past week, and now exhibits Tuberculous meningitis is a complication of disseminated tuberculosis,
nausea, vomiting, and diminished responsiveness to verbal commands. and young children are at increased risk in locations where tuberculosis is
On examination she has a temperature of 37.2° C. A tremor is observed in prevalent. It tends to produce a more chronic course. Cranial nerves, such
her extremities. Her eyes do not move laterally. A lumbar puncture is as the abducens in this case, can be involved. A large granulomatous
performed and examination of the CSF shows 100 leukocytes/mm3, and mass, a tuberculoma, may complicate some cases. Viral “aseptic”
75% of them mononuclears. The CSF glucose is decreased, but the meningitis is marked by mononuclear cells, but the protein is not usually
protein is markedly elevated. CT imaging with enhancement shows basilar markedly elevated, and no mass develops. Bacterial meningitis is often
meningeal thickening and a focal 2-cm mass involving the right cerebellar associated with a low glucose, but not a mass, and the inflammatory
hemisphere. Which of the following infectious agents is most likely to response is predominantly polymorphonuclear. T. pallidum is the
produce these findings? causative agent for neurosyphilis with a long, insidious course and no
A. Coxsackievirus B mass effect. T. solium can lead to cysticercosis, with cystic masses in the
B. Haemophilus influenzae brain
C. Mycobacterium tuberculosis
D. Treponema pallidum
E. Taenia solium

39. A 65-year-old man has been noted by his family to be more apathetic, F. Treponema pallidum
irritable, and withdrawn over the past year. He has worsening mental He has neurosyphilis from infection by Treponema pallidum. This form of
function. Neurologic examination shows a positive Romberg sign. His tertiary syphilis occurs years following primary infection. His eye finding is
pupils constrict with near focusing but not with exposure to light. He has consistent with Argyll Robertson pupil; his positive Romberg test is the
delusions of grandeur. He can remember only 1 of 3 objects after 5 result of tabes dorsalis. His mentation has changed from general paresis.
minutes. CSF obtained from lumbar puncture shows a lymphocytic Listeriosis and streptococcal infections are most likely to produce an acute
pleocytosis and elevated protein with increased IgG. Which of the following meningitis. Rabies may have an incubation period of 1 to 3 months, but
organisms has most likely caused his illness? then manifests with an acute severe encephalitis. Rubeola (measles)
A. Listeria monocytogenes infection rarely leads to subacute sclerosing panencephalitis years later,
B. Rabies virus marked by spasticity and seizures. Toxoplasmosis is an opportunistic
C. Rubeola (measles) virus infection with encephalitis and abscess developing over weeks.
D. Streptococcus pneumoniae
E. Toxoplasma gondii
F. Treponema pallidum

40.A 26-year-old woman has headaches for 4 weeks along with increasing B. Echovirus
malaise. Physical examination yields no remarkable findings. CT scan of An acute lymphocytic meningitis is most typically caused by a virus, such
the head shows no abnormalities. A lumbar puncture yields clear, colorless as West Nile virus, an equine encephalitis virus, or an echovirus. It is
CSF with a normal opening pressure. Laboratory analysis of the CSF sometimes referred to as aseptic meningitis because routine Gram
shows a normal glucose concentration and a minimally increased protein staining and bacterial cultures are negative. Most cases are self-limited,
level. A few lymphocytes are present, but there are no neutrophils. A Gram occur in immunocompetent individuals, and resolve without significant
stain and India ink preparation of the CSF are negative. Her condition sequelae. Cryptococcal and toxoplasmosis infections can occur in
gradually improves over the next 6 months. Serum serologic tests are most immunocompromised patients, but the India ink test would be positive in
likely to show an elevated titer of antibodies to which of the following the former, and CT scan would show focal ring-enhancing lesions in the
infectious agents? latter. Listerial and meningococcal infections
A. Cryptococcus neoformans
B. Echovirus
C. Listeria monocytogenes
D. Neisseria meningitidis
E. Toxoplasma gondii

41.A 25-year-old, previously healthy woman has acute onset of confusion C. HSV
and disorientation followed by a generalized tonic-clonic seizure. On Hemorrhagic lesions of the temporal lobes are characteristic of herpes
admission to the hospital, she is afebrile, and her blood pressure is 110/65 simplex virus encephalitis; cases are few and usually sporadic, occurring
mm Hg. No papilledema is observed. Serum and urine drug screening in apparently healthy individuals. The lesions of aspergillosis can be
results are negative. CT scan of the head shows a 3-cm recent hemorrhagic, but they are typically seen in immunocompromised
hemorrhage in the left temporal lobe. A lumbar puncture is done, and the individuals. Cytomegalovirus infection occurs in neonates and in
CSF shows only a few mononuclear cells and normal glucose and protein immunocompromised adults, but it does not produce hemorrhagic lesions.
levels. Infection with which of the following organisms is the most likely Arboviral infections produce focal lesions that may have an associated
cause of her disease? vasculitis with hemorrhage, but the CSF protein is usually elevated, and
A. Aspergillus niger there is a neutrophilic pleocytosis. Meningococcal infections produce
B. Cytomegalovirus meningitis.
C. Herpes simplex virus
D. Eastern equine encephalitis virus
E. Neisseria meningitidis

42. A 33-year-old woman, G3, P2, had two previous pregnancies that A. CMV
resulted in normal term infants, but now she gives birth at 34 weeks’ Cytomegalovirus (CMV) infection is one of the con- genital TORCH
gestation to a stillborn fetus. On examination, the fetus is observed to be infections, and it can become widely disseminated to affect the CNS.
hydropic. Autopsy of the fetus shows marked organomegaly, and the brain Periventricular leukomalacia is characteristic of CMV infection. Group B
has extensive necrosis in a periventricular pattern, with focal calcifications. Streptococcus infections cause premature rupture of membranes and sep-
What congenital infection is most likely to produce these findings? sis without significant CNS findings. Heart failure in utero causes hydrops
A. Cytomegalovirus fetalis. Herpes simplex virus infection of neonates typically occurs during
B. Group B Streptococcus passage through the infected birth canal, not in utero. HIV infection

❗USE AT YOUR OWN RISK❗ page 45 of 59


ROBBINS REVIEW

C. Herpes simplex virus produces no significant CNS findings in the perinatal period. Listeriosis can
D. HIV produce focal microabscesses in various organs, but usually there is
E. Listeria monocytogenes minimal necrosis

43. A 14-year-old girl, living in Nigeria, who has received poor prophylactic A. Anterior Horns
vaccines, develops mild diarrhea over 3 days, then has fever with neck Poliomyelitis is described, starting with the minor gastrointestinal illness,
stiffness and bilateral muscle weakness. On examination muscle followed by the acute major illness, then recovery, then the late post-polio
tenderness is present, with 3/5 motor strength in all extremities. She has syndrome. This enterovirus attacks lower motor neurons, leading to
difficulty breathing that requires mechanical support of respiration. Over neurogenic muscle weakness and atrophy. The anterior horns of the spinal
the next 6 months the pain disappears and some muscle strength returns. cord include lower motor neurons. Cranial nerve nuclei with lower motor
Which of the following nervous system structures has been affected most neurons may also be involved. The basal ganglia modulate muscle
by her illness? movement, and involvement of these structures may result in movement
A. Anterior horns disorders, but not paralysis. The corticospinal tracts are axons from upper
B. Basal ganglia motor neurons found in the neocortex, and neither is involved with polio.
C. Corticospinal tracts The dorsal root ganglia are involved with sensory pathways. Myoneural
D. Dorsal root ganglia junctions may be involved with myasthenia gravis or clostridial infections
E. Myoneural junctions F Neocortex (botulism, tetanus)

44. A 12-year-old boy develops fever, accompanied by occasional B. Neari bodies


headaches, malaise, fatigue, and nausea a month after being bitten by a The child had rabies, an infectious disease in which the virus travels from
dog. One day later, he experiences episodes of rigidity, hallucinations, the site of an infected animal bite up nerves to the CNS. Pathognomonic
breath holding, and difficulty swallowing because of uncontrollable oral Negri bodies are cytoplasmic inclusions found in the hippocampal
secretions. Dr. Louis Pasteur is consulted. He writes: “The death of this pyramidal cells and cerebellar Purkinje cells. Worldwide, unvaccinated
child appearing to be inevitable, I decided, not without lively and sore dogs remain the most common vector for transmission of this bullet-
anxiety, as may well be believed, to try ... the method which I had found shaped virus to man. The suspected animal must be sacrificed to identify
consistently successful with dogs. Consequently, 60 hours after the bites the virus in the brain. Infected neurons generally do not undergo cell death;
[the child] was inoculated under a fold of skin with half a syringeful of the instead the virus induces neurotoxin production that affects surface
spinal cord of a rabbit. In the following days, fresh inoculations were made. receptor interaction with neurotransmitters. After symptoms develop,
I thus made 13 inoculations.” The boy survives. Which of the following survival is rare. Multinucleated cells may be seen in microglial nodules
histologic findings in the brain of the dog is most likely to be present? associated with HIV infection. Perivascular lymphocytic infiltrates are often
A. Multinucleated giant cells seen with a variety of viruses, such as arboviruses or echoviruses, causing
B. Negri bodies encephalitis. Toxoplasma gondii infection is characterized by finding
C. Perivascular lymphocytes enlarged, infected cells filled with bradyzoites. Spongiform change is
D. Pseudocysts with bradyzoites characteristic for prion disease.
E. Spongiform change

45. A 37-year-old man who is HIV-1-positive has had increasing memory B. Cortical microglial nodules
problems for the past year. He is depressed. During the past 3 months, he AIDS dementia complex occurs late in the course of HIV infection. HIV-1
has had increasing problems with motor function and is now unable to produces an encephalitis characterized by a collection of reactive
stand or walk. For the past 3 days, he has had fever, cough, and dyspnea. microglial cells (microglial nodules). HIV-1-infected mononuclear cells,
A bronchoalveolar lavage shows cysts of Pneumocystis jiroveci. MRI of particularly macrophages, can fuse to form multinucleate cells, which are
the brain shows diffuse cerebral atrophy; no focal lesions are identified. seen within microglial nodules. Spongiform change suggests Creutzfeldt-
On microscopic examination of his brain, which of the following findings is Jakob disease, a rapidly progressive dementia unrelated to HIV infection.
most likely to be present? Lacunar infarcts are small, cavitary infarcts that result from
A. Cerebellar spongiform changes arteriolosclerosis of the deep penetrating arteries and arterioles. Such
B. Cortical microglial nodules arteriolar lesions occur in individuals with long-standing hypertension and
C. Lentiform nuclear lacunar infarcts are unlikely to be found in a 37-year-old man. Neocortical neuritic plaques
D. Neocortical neuritic plaques and neurofibrillary tangles are typical of Alzheimer disease and are unlikely
E. White matter plaques of demyelination to manifest at this patient’s age. Plaques of demyelination are typical of
multiple sclerosis.

46. A 52-year-old woman with leukemia undergoes chemo- therapy. Two C. JC polyomavirus
months later, she developed neurologic deficits with ataxia, motor Progressive multifocal leukoencephalopathy is caused by the JC
weakness in the right arm, difficulty swallowing, and sensory changes in polyomavirus and occurs in immunocompromised individuals, including
the left leg. MRI of the brain shows irregular areas of increased attenuation individuals with AIDS. The patient was treated for chronic myelogenous
in white matter of the cerebral hemispheres and the cerebellum. A leukemia. Cytomegalovirus infection also complicates the course of
stereotaxic biopsy specimen shows perivascular chronic inflammation, immunocompromised patients, but it causes large intranuclear inclusions,
marked gliosis, large reactive astrocytes with bizarre nuclei, and most often in endothelial cells. Herpes simplex virus is uncommon, even
intranuclear inclusions within oligodendroglia. What virus most likely in immunocompromised patients, and it most often produces hemorrhagic
caused these findings? encephalitis in temporal lobes. Rabies virus produces CNS excitability with
A. Cytomegalovirus convulsions, meningismus, and hydrophobia. Subacute sclerosing
B. Herpes simplex virus panencephalitis is a rare complication of measles (rubeola) virus infection
C. JC polyomavirus and leads to progressive mental decline, spasticity, and seizures. West
D. Rabies virus Nile virus, similar to many arboviruses, can cause meningoencephalitis.
E. Rubeola virus
F. West Nile virus

47. A 38-year-old man with chronic renal failure received a kidney D. Cryptococcus neoformans
transplant. While being treated with cyclosporine, azathioprine, and high Cryptococcal meningoencephalitis is a complication of his
doses of corticosteroids, he began to experience headaches and became immunocompromised state. Cryptococcus neoformans typically has a
lethargic. On physical examination, he now has a fever and nuchal rigidity. thick capsule, making it easily visible with the India ink preparation, a
A lumbar puncture is performed and the opening pressure is increased. A procedure that can be performed within a few minutes on a CSF sample.
CSF cell count shows increased leukocytes. An India ink preparation A cryptococcal antigen test on the CSF would also be useful for this
shows the findings in the figure. Which of the following organisms is most patient. Brain biopsies are not commonly performed, and other, less
invasive methods should be pursued first. Bacterial meningitis is possible,

❗USE AT YOUR OWN RISK❗ page 46 of 59


ROBBINS REVIEW

likely infecting this man? and pneumococcus would be a common bacterial cause, but this
description is consistent with cryptococcosis. Of the remaining fungal
organisms listed, none are readily identified with India ink. Aspergillus spp.
A. Aspergillus niger appear as branching septate hyphae, not as yeasts. Blastomyces
B. Blastomyces dermatitidis dermatitidis is characterized by broad-based budding yeasts. Coccidioides
C. Coccidioides immitis immitis yeast forms appear as large spherules containing endospores.
D. Cryptococcus neoformans Histoplasma capsulatum organisms are small and tend to be found within
E. Histoplasma capsulatum macrophages

48. A previously healthy 21-year-old man with a D. Taenia solium


a severe headache for 5 days now has a new onset seizure. Papilledema Cysticercosis from eating uncooked pork can result in the release of larvae
is noted on Funduscopic examination. An MRI of the brain shows multiple that penetrate the gut wall and disseminate hematogenously, often settling
0.5- to 1.5-cm cystic periventricular and meningeal lesions. Which of the in gray and white cerebral tissue, where they develop into cysts. The cysts
following infectious organisms is most likely to may cause obstructive hydrocephalus. Neurocysticercosis is a major
produce these findings? cause for seizures in parts of the world where pork tapeworms are
A Aspergillus fumigatus ingested. Aspergillosis is a fungal disease in which the foci of inflammation
B Cryptococcus neoformans grossly resemble granulomas, but there is often minimal inflammatory
C Plasmodium falciparum response, and the propensity for vascular invasion often produces a
D Taenia solium hemorrhagic border to the lesions. Cryptococcosis most often involves the
E Toxoplasma gondii lungs and meninges. Malaria caused by Plasmodium falciparum produces
F Trypanosoma gambiense hemolytic anemia, splenomegaly, and cerebral thrombosis.
Toxoplasmosis can be a congenital infection. In immunocompromised
adults, it can produce inflammation in multiple tissues, but most often it
causes chronic abscessing inflammation in the brain. African
trypanosomiasis produces sleeping sickness.

49. A 20-year-old HIV-positive man has had a decreased level of E. Toxoplasma pseudocysts
consciousness for the past week. He now experiences a generalized tonic Toxoplasmosis is a common opportunistic infection that affects the CNS in
clonic seizure. On physical examination, his temperature is 37.6° C. MRI immunocompromised patients, including those with AIDS, and produces
of the brain shows several 1- to 3-cm, ring-enhancing lesions in the abscesses that organize on the periphery to produce a bright ring on CT
cerebral gray matter bilaterally. A stereotaxic biopsy is performed. What and MRI. Candida fungal infections of the CNS are rare, and disseminated
pathologic finding is most likely to be present on microscopic examination candidiasis in AIDS is uncommon. Malignant lymphomas also can produce
of the biopsy specimen? this picture, but without fever, and they generally occur as fewer, larger
A. Budding cells with pseudohyphae masses or as a solitary mass. Metastatic carcinoma, which typically
B. Large atypical lymphocytes presents with multiple lesions, is uncommon at this age. A spongiform
C. Metastatic squamous cell carcinoma encephalopathy, such as Creutzfeldt-Jakob disease, typically has no
D. Spongiform encephalopathy grossly visible or radiographic findings.
E. Toxoplasma pseudocysts

50. A 63-year-old previously healthy woman has become more forgetful D. Plaques of demyelination
over a period of 6 weeks. One month later, she has difficulty ambulating A rapidly progressive dementia is most consistent with Creutzfeldt-Jakob
and is unable to care for herself. On physical examination, she has disease (CJD). CJD is one of a group of diseases that are called
myoclonus. She is afebrile. CT scan of the head shows minimal cerebral spongiform encephalopathies because they produce a microscopic
atrophy. An EEG shows low-amplitude, slow background activity with peri- vacuolated appearance of the neocortex. CJD occurs in sporadic or
odic complexes and occasional repetitive sharp waves with intervals of 0.5 familial forms and is caused by prion protein. The normal prion protein of
to 1 second. Which of the following histologic abnormalities is most likely the brain, designated PrPc, can undergo conformational change to PrPsc,
to be found in her cerebral cortex? which then induces further change in PrPc to PrPsc. This patient’s age and
A. Lewy bodies clinical findings are characteristic of the sporadic type of CJD, which may
B. Microglial nodules arise from spontaneous mutation or from exposure to PrPsc. Variant CJD,
C. Numerous neuritic plaques which may be linked to expo- sure to bovine spongiform encephalopathy,
D. Plaques of demyelination occurs in much younger patients and does not produce the characteristic
E. Spongiform encephalopathy EEG findings. Microglial nodules may be seen in patients with AIDS.
Neuritic plaques are seen in Alzheimer disease, which occurs over many
years. The demyelinating plaques of multiple sclerosis develop over years,
as do the Lewy bodies of Parkinson disease and diffuse Lewy body
disease.

51. A 27-year-old woman had an episode of weakness 3 months ago, F. Visual impairment
which she attributed to job stress and fatigue. The neurologic examination Multiple sclerosis (MS) produces white matter plaques of demyelination,
shows mild residual weakness, with 4/5 motor strength in the right lower and most patients develop optic neuritis with visual difficulties, often
extremity. A lumbar puncture is done, and laboratory examination of the unilateral. The course of MS varies, with many relapses and remissions,
CSF shows increased IgG levels with prominent oligoclonal bands. MRI of and some patients are affected more severely than others. Some patients
the brain shows small, scattered, 0.5-cm areas consistent with have minimal problems, but most can be expected to have further
demyelination, most of which are located in periventricular white matter. neurologic problems. Severe neurologic impairment and death are
Which of the following complications is she most likely to develop? unlikely, and most patients live for decades. There is no defined
A. Non-Hodgkin lymphoma inheritance pattern. Although MS is an immunologically mediated disease,
B. Progressive dementia immune dysregulation leading to development of lymphoid neoplasms is
C. Quadriplegia unlikely to occur. The focality of white matter lesions and the sparing of
D. Seizure disorder gray matter make dementia and quadriplegia unlikely complications of

❗USE AT YOUR OWN RISK❗ page 47 of 59


ROBBINS REVIEW

E. Tremor at rest MS. Seizure disorders in MS are uncommon. Tremor at rest is a feature of
F. Visual impairment Parkinson disease

52. A 28-year-old man states that 3 years ago he experienced D. Oligoclonal bands
paresthesias of his left arm and had difficulty walking, but these problems This shifting spectrum of clinical findings over time in a young adult
were resolved. During the past year, he developed difficulty seeing from suggests the diagnosis of multiple sclerosis. The plaques of demyelination
his left eye. Six months ago, he had difficulty writing with his right hand. that give rise to the differing symptoms can be found in various locations,
On physical examination, there is decreased visual acuity on the left, no but they most often occur in periventricular white matter. CSF
papilledema, and no retinal lesions. There is decreased motor strength and immunoglobulins are increased, and most patients show oligoclonal bands
decreased sensation in the right hand and fore- arm. MRI of the brain of IgG. Neurosyphilis, a form of tertiary syphilis, is rare at this age and
shows focal areas of increased signal intensity in periventricular white probably would not cause localizing signs that change. Cryptococcal
matter and in the left optic nerve. A lumbar puncture is performed. What meningitis would manifest more acutely with meningeal signs. Primary or
finding is most likely to be present on examination of the CSF? malignant brain tumors are uncommon at this age and would not have
A. Anti Treponemal antibodies such a long course without serious sequelae. Xanthochromia from
B. Cryptococcal antigen hemorrhage would suggest a more acute problem.
C. Malignant cells
D. Oligoclonal bands
E. Xanthochromia

TU4, <, YGB2-%"B#$3, =#*%), +1+'%()+, 9$1)', .-%3, A. Central pontine myelinolysis
The rapid correction of hyponatremia is a common antecedent to central
'"%1*%, (), %, *#'#", 8-.(&$-, %&&(3-)'4, Z), pontine myelinolysis, with demyelination in the basis pontis, with the round
%3*(++(#),'#,'.-,.#+0('%$7,+.-,(+,&#)+&(#1+,91', area of decreased signal intensity shown in the figure. Extrapontine
myelin- olysis may also occur. A history of alcohol abuse increases the
3(+#"(-)'-34,?@,+&%),#6,'.-,.-%3,+.#=+,%,"(/.', risk. Cerebral edema sufficient to produce herniation would cause
'-*0#"%$, 9#)-, 6"%&'1"-, %)3, *($3, &-"-9"%$, papilledema, not noted in this case. Intracranial hemorrhages would not
result from electrolyte and fluid disturbances. A subacute combined
-3-*%4, A-", 9$##3, %$&#.#$, $-8-$, (+, R4[G, /*S4, degeneration of the spinal cord occurs slowly as a consequence of vitamin
@=#,3%2+,$%'-"7,$%9#"%'#"2,+'13(-+,+.#=,+-"1*, B12 (cobalamin) deficiency. Wernicke-Korsakoff syndrome is now a rare
accompaniment to chronic alcoholism that affects mammillary bodies and
H%\7, QR], **#$^X_, `\7, U4], **#$^X_, ?$F7, a[, periaqueductal gray matter
**#$^X_, ?Z[7, [U, **#$^X_, /$1&#+-7, YU, */^3X_,
%)3, &"-%'()()-7, Q, */^3X4, @.-, .20#)%'"-*(%, (+,
&#""-&'-3,#8-",'.-,)-:',[,.#1"+,=('.,()'"%8-)#1+,
6$1(3,%)3,-$-&'"#$2'-,'.-"%02,%)3,3(1"-'(&+4,D.-,
'.-), "%0(3$2, 9-&#*-+, &#)61+-3, %)3, -:.(9('+, $(*9, =-%M)-++4, H#,
0%0($$-3-*%, (+, +--), #), 61)31+&#0(&, -:%*()%'(#)4, <), ;V5, +.#=+, '.-,
6()3()/, (), '.-, 6(/1"-4, b.%', &#*0$(&%'(#), .%+, *#+', $(M-$2, #&&1""-3, (), '.(+,
=#*%)c

A. Central pontine myelinolysis


B. Cerebellar tonsillar herniation
C. Intraventricular hemorrhage
D. Subacute combined degeneration of the cord
E. Wernicke-Korsakoff syndrome

54. An 8-year-old boy recovered uneventfully from a viral upper respiratory B. Demyelination
infection 2 weeks ago, but now has the abrupt onset of lethargy and Acute disseminated encephalomyelitis is an immunologically mediated
irritability. On neurologic examination he has diminished pupillary reflexes demyelinating disease that has some features similar to multiple sclerosis,
bilaterally along with ataxia of his extremities. CSF obtained by lumbar but may occur in younger patients, have an abrupt onset, and be rapidly
puncture microscopically shows small numbers of lymphocytes and fatal. Most cases are preceded by an infection. The remaining choices are
erythrocytes. MRI shows multiple hyperintense lesions at the gray-white unlikely to have bilateral optic neuritis. Abscesses are likely to be bacterial
junction. Which of the following pathogenic mechanisms is most likely in origin. Emboli are uncommon in children, and likely to produce
causing this child’s brain lesions? infarctions. Metastases are unlikely in children. Thiamine deficiency may
A. Abscess formation lead to Wernicke disease, which has hemorrhagic lesions and is unrelated
B. Demyelination to infection. Vasculitis may produce focal lesions but is uncommon in
C. Embolization children.
D. Metastases
E. Thiamine deficiency
F. Vasculitis

55. A study is conducted of patients who had increased phosphorylated D. Progressive memory loss
tau and decreased Aà peptide in their CSF 5 to 10 years prior to death at The figure shows a neuritic plaque with a rim of dystrophic neurites
ages ranging from 55 to 80 years. At autopsy their brain weights are less surrounding an amyloid core consistent with Alzheimer disease (AD), the
than normal for age and body size. On gross examination, these brains most common form of progressive dementia. AD is marked by increased
show hydrocephalus ex vacuo and cortical atrophy but no focal lesions. numbers of microscopic neuritic plaques and neurofibrillary tangles
The figure shows the high power microscopic appearance of the cerebral compared to controls for age. Choreiform movements suggest Huntington
neocortex with Bielschowsky silver stain. Which of the following symptoms disease. Gait disturbances occur in Parkinson disease. Seizures are
associated with many lesions, but often a pathologic finding is not
discernible. Symmetric muscular weakness suggests amyotrophic lateral
sclerosis

❗USE AT YOUR OWN RISK❗ page 48 of 59


ROBBINS REVIEW

is most likely to be recorded in the


medical histories of these patients?

A. Choreiform movements
B. Gait disturbances
C. Grand mal seizures
D. Progressive memory loss
E. Symmetric muscular weakness

56. A 68-year-old woman with a 7-year history of progressive dementia E. Presence of the e4 allele at the ApoE gene
dies of bronchopneumonia. At autopsy, there is cerebral atrophy in a
predominantly frontal and parietal lobe distribution. Microscopic The clinical history of dementia and the presence of numerous neuritic
examination of the brain shows numerous neuritic plaques in the plaques and amyloid deposition in blood vessel walls are characteristic of
hippocampus, amygdala, and neocortex. Neurofibrillary tangles in the Alzheimer disease (AD). The e4 allele of the ApoE4 gene increases the
hippocampus contain tau protein. Congo red staining shows amyloid in the risk of developing AD by unknown mechanisms. Expansion of CAG
media of the small peripheral cerebral arteries. Which of the following repeats on chromosome 4p16 causes Huntington disease. There is no
genetic abnormalities is the most important factor in the development of association between HLA genes and AD. Increased repeats in the FMR1
her disease? gene occur with fragile X syndrome. Mutant prion genes give rise to
A. Expansion of CAG repeats on chromosome 4p16 spongiform encephalopathies, such as Creutzfeldt-Jakob disease
B. HLA-DR3/DR4 alleles
C. Increased tandem repeats in the FMR1 gene
D. Mutation of a prion protein gene
E. Presence of the e4 allele at the ApoE gene

57. A 63-year-old man had increasing irritability over 3 years. He wandered F. Pick disease
about his neighborhood, complaining to the neighbors about everything. Pick disease is the best known form of frontotemporal lobar degeneration
He had no memory loss and was always able to find his way home. The with tau-containing inclusions (FTLD-tau) and has clinical features similar
neighbors were pleased when he developed aphasia. On physical to the features of Alzheimer disease (AD), but initially FTLD-tau causes
examination, there were no motor or sensory deficits and no gait less memory loss and more behavioral changes. The “knifelike” gyral
disturbances or tremor. MRI of the brain showed bilateral marked temporal atrophy of frontal and temporal lobes and relative sparing of parietal and
and frontal lobe gyral atrophy. He died of pneumonia 1 year later. At occipital lobes are characteristic of Pick disease. Pick bodies with tau
autopsy, the frontal cortex microscopically shows extensive neuronal loss, protein are seen in remaining neurons, but the neuritic plaques and
and some remaining neurons show intracytoplasmic, faintly eosinophilic, neurofibrillary tangles seen in AD are not increased. Huntington disease
rounded inclusions that stain immunohistochemically for tau protein. What affects mainly the caudate nuclei and basal ganglia; onset occurs in middle
is the most likely diagnosis? age, and choreiform movements are common. Leigh disease is a
A. Alzheimer disease mitochondrial encephalomyopathy that can cause muscular weakness and
B. Huntington disease neurologic deterioration beginning at a young age. Multiple system atrophy
C. Leigh disease (MSA) has features that overlap those of striatonigral degeneration,
D. Multiple system atrophy olivopontocerebellar atrophy, and ShyDrager syndrome; most patients
E. Parkinson disease with MSA exhibit symptoms similar to those of Parkinson disease. MSA is
F. Pick disease characterized microscopically by the appearance of glial cytoplasmic
G. Vascular dementia inclusions. In Parkinson disease, loss of pigmented neurons in the
substantia nigra leads to movement problems. Vascular dementia, or
multi-infarct dementia, can have clinical features that mimic those of AD,
but there are multiple small infarcts that collectively produce dementia, and
the neurologic decline occurs in a stepwise fashion.

58. A 60-year-old woman had problems related to movement for 5 years. A. a-Synuclein
Physical examination showed cogwheel rigidity of limbs and a festinating Dementia with Lewy bodies (DLB) combines clinical features of Alzheimer
gait, which she had difficulty initiating. Her face was expressionless. She disease (AD) and idiopathic Parkin- son disease. Mutations in the gene for
was given levodopa/carbidopa, and her condition improved. Two years alpha-synuclein have been linked to idiopathic Parkinson disease, and
later, she had difficulty performing activities of daily living and showed Lewy bodies can be found in the substantia nigra neurons, but the clinical
marked cognitive decline. She died of aspiration pneumonia. Autopsy dementia and cortical Lewy bodies point to DLB. Amyloid precursor protein
findings include mild cerebral atrophy and loss of substantia nigra (APP) is encoded by a gene on chromosome 21 (perhaps explaining early
pigmentation. Microscopically, cortical neurons show spheroidal, AD in trisomy 21) and is processed to form the Aà amyloid of neuritic
intraneuronal, cytoplasmic, and eosinophilic inclusions. plaques in AD. The e4 allele of apolipoprotein E can bind Aà and increase
Immunohistochemical staining for which of the following proteins is most the risk of AD. Huntingtin is the protein product of the HD gene in
likely to be positive in these inclusions? Huntington's disease. Presenilin 1 and 2 mutations can increase
A. a-Synuclein production of Aà and increase the risk of early onset AD. Tau protein is
B. Amyloid precursor protein found in neurofibrillary tangles of AD and in Pick bodies of Pick disease
C. Apolipoprotein E
D. Huntingtin
E. Presenilin

❗USE AT YOUR OWN RISK❗ page 49 of 59


ROBBINS REVIEW

F. Tau protein

59. A 55-year-old man has had E. Tremor at rest


increasing difficulty with Loss of pigmented dopaminergic neurons in the substantia nigra of the
initiation of voluntary midbrain is most characteristic of Parkinson disease. Pill-rolling tremors at
movements and increasing rest are typical of this disorder. A variety of genetic abnormalities have
inability to perform activities of been associated with forms of Parkinson disease, including !-synuclein,
daily living for 1 year. On parkin, DJ-1, and PINK1 gene mutations and mitochondrial dysfunction.
physical examination, he has Ataxia suggests a disruption in the motor control pathways, such as the
difficulty initiating movement, cerebellum, or proprioception, from dorsal spinal cord columns.
but he can keep moving if he Choreiform movements suggest Huntington disease, which affects the
follows someone walking ahead caudate, not the substantia nigra. Short-term memory problems suggest
of him. He has an hippocampal lesions. Symmetric weakness suggests a motor neuron
expressionless face. The left disease
side of the figure shows the
gross appearance of the
midbrain of this patient; on the right is a section through the normal
midbrain. What additional clinical feature is most closely associated with
this abnormality?

A. Ataxia with ambulation


B. Choreiform movements
C. Loss of short-term memory
D. Symmetric weakness in the extremities
E. Tremor at rest

60. A 47-year-old woman from Venezuela has had difficulty performing D. Expansion of CAG repeats
activities of daily living for the past year. She is emotionally labile and often Huntington disease is a progressive degenerative disorder that affects
cries. She is disturbed and depressed by these developments because her basal ganglia, including the putamen and caudate nucleus through loss of
mother and brother died 5 years after experiencing the same symptoms. spiny striatal neurons that normally dampen motor activity. This disease,
On physical examination, she has choreiform movements of her inherited in an autosomal dominant pattern, is caused by an abnormal
extremities. Cranial nerves are intact. She has no motor weakness and no expansion of the trinucleotide CAG in the huntingtin gene on chromosome
sensory deficits. Her memory remains intact. Which of the following 4, which normally has 6 to 35 CAG copies. There may be 40 to 55 copies
genetic abnormalities is most likely to be present in this woman? in patients with typical Huntington disease and 70 repeats in patients with
A. Abnormal prion protein earlier onset of the disease. A “premutation” with 36 to 39 repeats may
B. Decreased level of hexosaminidase A enzyme have reduced penetrance. These patients have atrophy, with loss of
C. Extra chromosome 21 neurons and gliosis starting in the caudate, as well as putamen and globus
D. Expansion of CAG repeats pallidus. About 10% of cases of Creutzfeldt-Jakob disease are genetically
E. Mutation in presenilin genes determined, with the inheritance of an abnormal prion protein that leads to
spongiform encephalopathy in later adult life. Tay-Sachs disease of
infancy and childhood is caused by a deficiency of hexosaminidase A.
Trisomy 21 results in mental retardation present at birth. Mutations in
presenilin genes cause familial Alzheimer disease (AD). The risk of AD
increases with increased levels of apolipoprotein E.

61. A 4-year-old girl developed clumsiness and difficulty ambulating over D. GAA repeats in the frataxin gene
6 months. On physical examination, she showed difficulty with balance She had Friedreich ataxia, an autosomal recessive progressive illness that
while walking, dysarthria, poor hand coordination, absent deep tendon most often has an onset in the first decade of life. The frataxin gene
reflexes, and a bilateral Babinski sign. Light touch and vibratory sensation encodes for a protein involved in iron regulation in cells, and a GAA
were greatly diminished. There was no muscular weakness. Over the next trinucleotide repeat expansion results in decreased protein and decreased
5 years, she developed congestive heart failure from hypertrophic mitochondrial oxidative phosphorylation. The other options listed have no
cardiomyopathy. She also had hyperglycemia. At autopsy, there was cardiac involvement. Mutations of the huntingtin gene are seen with
increased perinuclear iron deposition within cardiac myocytes. Which of Huntington disease marked by choreoathetosis beginning in young to
the following genetic abnormalities with trinucleotide repeat expansions middle aged adults. Increased tandem repeats in the FMR1 gene account
was most likely present in this patient? for cases of fragile X syndrome characterized by mental retardation. The
A. CAG repeats in the huntingtin gene dystrophila myotonia–protein kinase gene is abnormal in cases of
B. CGG repeats in the FMR1 gene myotonic dystrophy with muscular weakness and dementia
C. CTG repeats in the dystrophila myotonia protein kinase gene
D. GAA repeats in the frataxin gene

62. A 36-year-old man who had been healthy all his life now has B. Lower motor neuron
progressive, symmetric muscular weakness. A year ago, he noted The progressive and symmetric nature of this patient’s disease is a classic
weakness in the area of the head and neck, which caused difficulty with feature of amyotrophic lateral sclerosis (ALS). The muscles show a
speech, eye movements, and swallowing. In the past year, the weakness denervation type of grouped atrophy from loss of lower motor neurons in
in the upper and lower extremities has increased, and he can no longer anterior horns of the spinal cord. The “bulbar” form of ALS affects mainly
stand, walk, or feed himself. His mental function remains intact. Which of cranial nerve nuclei and has a more aggressive course. Cortical upper
the following cells is most likely being destroyed in this man? motor neurons may also be lost, but mental function is preserved in ALS.
A. Ependymal cell Ependymal cells line ventricles, which are normal in ALS. Microglial cells
B. Lower motor neuron have a macrophage-like function and may be involved in demyelinating
C. Microglial cell plaques of multiple sclerosis that can produce various motor signs and
D. Oligodendrocyte symptoms over time, but symmetry is not a feature of this disease.
E. Pigmented neuron Oligodendrocytes provide myelin to axons in the CNS. Parkinson disease
F. Spiny neuron with loss of substantia nigra pigmented neurons is characterized by rigidity
and involuntary movements, not by muscular weakness. Huntington
disease with loss of spiny neurons in the caudate nucleus causes

❗USE AT YOUR OWN RISK❗ page 50 of 59


ROBBINS REVIEW

abnormal movements, not weakness, and there can be associated


dementia over time

63. A 12-year-old girl has had progressively diminishing neurologic B. Metachromatic leukodystrophy
function over 3 years. She has difficulty with ambulation, decreased mental In children, an inherited form of CNS disease that accounts for a
ability, seizures, and loss of control over bladder and bowel functions. An progressively worsening course should be suspected. The
MRI of her brain shows atrophy, and the centrum semiovale and central leukodystrophies are various inborn errors of metabolism involving
white matter are shrunken. These findings correlate with widespread lysosomal (arylsulfatase A in this case) or peroxisomal enzymes that affect
microscopic myelin loss, but subcortical myelin is spared. Which of the white matter extensively and cause myelin loss and abnormal
following degenerative CNS diseases best explains her illness? accumulations of myelin from failure of generation, maintenance, or
A. Acute disseminated encephalomyelitis catabolism of myelin. There are no discrete plaques of demyelination,
B. Metachromatic leukodystrophy however, in contrast to multiple sclerosis. Sparing of subcortical myelin (U
C. Multiple sclerosis fiber) is often seen in leukodystrophies. Acute disseminated
D. Progressive multifocal leukoencephalopathy encephalomyelitis is a postinfectious process with abrupt onset.
E. Tay-Sachs disease Progressive multifocal leukoencephalopathy is an infectious lesion that
occurs in immunocompromised adults. Tay-Sachs disease affects infants.

64. A 49-year-old man develops an acute psychosis. He has a lengthy E. Thiamine


history of chronic alcoholism. He has difficulty performing a finger-to-nose Wernicke disease results from a deficiency of vitamin B1 (thiamine).
test, and there is paralysis of the lateral rectus muscles. A deficiency of Wernicke disease is uncommon in individuals who have a varied diet, but
which of the following nutrients is most likely to produce these findings? individuals with a history of chronic alcoholism may not. Capillary
A. Cobalamin proliferation, hemorrhage, necrosis, and hemosiderin deposition are often
B. Folate found in the mammillary bodies and the periaqueductal gray matter,
C. Niacin resulting in paralysis of the extraocular muscles. If memory problems with
D. Pyridoxine confabulation are observed, the thalamus is involved and the diagnosis is
E. Thiamine Wernicke-Korsakoff syndrome. Though deficiency of either B12 or folate
may produce macrocytic anemia, subacute combined degeneration of the
spinal cord is seen with vitamin B12 (cobalamin) deficiency, but folate
deficiency does not produce CNS signs. Dementia may be present in
individuals with niacin deficiency. A deficiency of pyridoxine may result in
peripheral neuropathy.

65. A 53-year-old man with a lengthy history of chronic alcohol abuse has A. Ammonia level of 100 μmol/L
had an increasingly clouded sensorium over the past 2 days. On physical Liver failure with hepatic encephalopathy can occur in severe liver disease
examination, he has a flapping tremor of his outstretched hands. MRI of from various causes, including commonly chronic alcoholism.
the brain shows no abnormalities. Microscopic examination of his brain Hyperammonemia is a feature of liver failure. Carbon monoxide poisoning
would show increased numbers of neocortical and basal ganglia can produce obtundation and coma. Severe hypoglycemia can damage
astrocytes with pale, swollen nuclei (Alzheimer type II cells). Which of the neurons in the hippocampus and neocortex. An elevated hemoglobin A1c
following laboratory findings in his blood is most likely to be associated level suggests a diagnosis of diabetes mellitus, and diabetic patients are
with these findings? most prone to peripheral neuropathies and autonomic neuropathies.
A. Ammonia level of 100 μmol/L Hyponatremia from diabetes insipidus may result in obtundation.
B. Carboxyhemoglobin level of 5%
C. Glucose of 30 mg/dL
D. Hemoglobin A1c level of 10%
E. Sodium of 111 mmol/L

66. A 55-year-old man has experienced headaches for the first time in his B. Glioblastoma
life beginning 2 months ago. He comes to the emergency department He may initially have had an infiltrating astrocytoma, which is the most
following a generalized tonicclonic seizure. On physical examination, he common primary brain neoplasm in adults, typically arising in a cerebral
has weakness on the left side. An MRI of his brain shows a large, irregular, hemisphere. Lower grade astrocytomas in adults may have a more
6-cm mass in the centrum semiovale of the right cerebral hemisphere that indolent course. Some of these patients go on to develop a high-grade
extends across the corpus callosum. A stereotaxic biopsy of the mass is glioma known as secondary glioblastoma. Most glioblastomas arise de
done and microscopically shows pleomorphic cells positive for glial novo (primary glioblastoma) and have MDM2 mutations. Regardless of
fibrillary acidic protein (GFAP). Molecular analysis shows abnormalities of origin, glioblastomas are aggressive and have a poor prognosis. Diffuse
TP53 and platelet-derived growth factor alpha (PDGF-!). Which of the large B cell lymphoma is the most common type of “primary” CNS
following neoplasms is he most likely to have? lymphoma (without evidence for disease elsewhere); some arise in
A. Diffuse large B-cell lymphoma immunocompromised patients. Hemangioblastomas are uncommon
B. Glioblastoma neoplasms arising in the cerebellum, often with von Hippel–Lindau
C. Hemangioblastoma disease, and associated with polycythemia. Medulloblastomas and
D. Medulloblastoma pilocytic astrocytomas are usually child- hood brain tumors arising in the
E. Pilocytic astrocytoma posterior fossa

67. A 10-year-old boy has had persistent headaches for the past 3 months. A. Astrocytoma
On physical examination, he is afebrile. He has an ataxic gait and Primary malignant neoplasms of the brain in children most often occur in
dysdiadochokinesia. CT scan of the head shows a 4-cm cystic mass in the the posterior fossa. The two most common neoplasms at this site are
right cerebellar hemisphere. Cerebral lateral ventricles are enlarged. A pilocytic (cystic cerebellar) astrocytoma and medulloblastoma. Pilocytic
lumbar puncture is done. The CSF protein concentration is elevated, but astrocytoma is slow growing and has a better overall prognosis than glial
the glucose level is normal. Neurosurgery is performed, and the mass is neoplasms in adults. Both may enlarge and block CSF flow, causing
removed and sectioned. On gross examination, the mass is a cyst filled hydrocephalus. Ependymomas can occur in childhood, but are most likely
with gelatinous material. The cyst has a thin wall and a 1-cm mural nodule. to arise in the fourth ventricle. A hemangioblastoma is a rare cystic mass
Microscopically, the mass is composed of cells that stain positive for glial in adults, typically arising in the cerebellum, and may be associated with
fibrillary acidic protein (GFAP) and have long, hairlike processes. What is polycythemia. Medulloblastomas often occur in the cerebellar midline, are
the most likely diagnosis? composed of primitive round blue cells, and have a poor prognosis.
A. Astrocytoma Meningiomas occur in adults; they are circumscribed, solid mass lesions
B. Ependymoma adjacent to the dura, and may be multiple in neurofibromatosis. A

❗USE AT YOUR OWN RISK❗ page 51 of 59


ROBBINS REVIEW

C. Hemangioblastoma schwannoma typically arises in cranial nerve VIII in adults.


D. Medulloblastoma
E. Meningioma
F. Schwannoma

68. A 40-year-old man has been experiencing headaches for the past 6 E. 1p and 19q co-deletions
months. He had a seizure 1 day ago. On physical examination, there are Oligodendrogliomas tend to have a better prognosis than most other glial
no remarkable findings. MRI of the brain shows a solitary, circumscribed neoplasms. Pilocytic astrocytomas with BRAF mutations tend to be less
3-cm mass in the right parietal centrum semiovale. The mass has small circumscribed. Diffuse large B-cell lymphomas can occur in association
cysts and areas of calcification and hemorrhage. Neurosurgery is per- with AIDS; they are negative for glial fibrillary acidic protein (GFAP), but
formed, and the mass is removed. Microscopically, the mass consists of positive for CD19 and CD20. Medulloblastomas are posterior fossa tumors
sheets of cells with round nuclei that have granular chromatin. The cells that occur in children, and those with poor prognosis often have c-MYC
have a moderate amount of clear cytoplasm, and they are marked with amplification. Classic glioblastomas are highly aggressive, infiltrative
GFAP by immunohistochemical staining. The patient receives adjuvant gliomas, often with EGFR alterations
radiation and chemotherapy, and there is no recurrence. Which of the
following molecular markers is most likely to be found in the cells of this
mass?
A. BRAF mutation
B. CD20 expression
C. c-MYC amplification
D. EGFR amplification
E. 1p and 19q co-deletions

69. A 46-year-old woman has had increasing weakness and loss of B. Ependymoma
sensation in the lower extremities for the past 5 months. She has been The myxopapillary variant of ependymoma is more common in adults than
unable to walk without assistance for the past week. On physical in children. Ependymomas that arise in the ventricles (usually the fourth
examination, there is 4/5 motor strength in the right lower extremity and ventricle) are more common in the first 2 decades of life. Choroid plexus
3/5 motor strength in the left lower extremity. There is bilateral loss of papillomas are rare tumors that arise in the cerebral ventricles.
sensation to light touch from the lateral midthigh distally. MRI of the spine Meningiomas most often arise in the cranial cavity and have plump, pink,
shows a 1 × 4 cm lesion in the filum terminale. The mass is removed. round to spindle-shaped cells, often with psammoma bodies. Metastases
Microscopically, the mass is composed of cuboidal cells around papillary to the spinal cord are uncommon. Neurofibromas are more likely to arise
cores in a myxoid background. Which of the following lesions was most in peripheral nerves, although in neurofibromatosis type 1, they can arise
likely present in this patient? in many sites. A pilocytic astrocytoma is a common pediatric primary
A. Choroid plexus papilloma intracranial neoplasm; it arises in the posterior fossa. A schwannoma most
B. Ependymoma often occurs in cranial nerve VIII, although neurofibromatosis may be
C. Meningioma associated with multiple schwannomas.
D. Neurofibroma
E. Pilocytic astrocytoma
F. Schwannoma

70. An 11-year-old girl has had increasing headaches upon awakening for B. Ependymoma
the past month. On examination, papilledema is present bilaterally. An MRI In children, ependymomas most often arise in the floor of the fourth
of her brain reveals a 3-cm solid circumscribed mass within the fourth ventricle and can obstruct the flow of CSF (obstructive, noncommunicating
ventricle. There is third and lateral cerebral ventricular dilation. The mass hydrocephalus). Most childhood brain neoplasms are found in the
is excised and microscopically shows perivascular pseudorosettes with posterior fossa, including pilocytic astrocytomas that arise in a cerebellar
round, regular tumor cells arranged around vessels. Which of the following hemisphere and appear as a cystic mass with a mural nodule and tumor
neoplasms is she most likely to have? cells with hairlike processes. Medulloblastomas are also childhood tumors,
A. Astrocytoma and they arise in the midline cerebellum and are composed of
B. Ependymoma undifferentiated, primitive blue cells. Glioblastomas are adult neoplasms
C. Glioblastoma arising in the cerebral hemispheres. Schwannomas most often arise at the
D. Medulloblastoma cerebellopontine angle in the eighth cranial nerve
E. Schwannoma

71. A 5-year-old boy has complained of headaches for the past week. His B. Cerebellar vermis
gait has become ataxic. After a sudden onset of vomiting, he is brought to The MRI shows a 4-cm mass in the cerebellar vermis, along with dilation
the emergency department, where he becomes comatose. On physical of the third and lateral cerebral ventricles, consistent with
examination, he is afebrile. A lumbar puncture is done and cytologic medulloblastoma. Most intracranial neoplasms in children are located in
examination of the CSF shows anaplastic cells with dark blue nuclei and the posterior fossa. The medulloblastoma, one of the “blue cell tumors” of
scant cytoplasm. An MRI is most likely to show a mass in which of the childhood, arises in the midline, and the cells can seed into the CSF. A
following locations? glioblastoma is also a high-grade malignancy that could seed the CSF, but
A. Centrum semiovale this neoplasm occurs in adults in a cerebral hemisphere. Schwannomas
B. Cerebellar vermis most often involve the eighth cranial nerve, are benign neoplasms, and do
C. Cranial nerve VIII not seed the CSF. Ependymomas can occur in children, but they typically
D. Fourth ventricle arise in the ventricles. Metastatic lesions may involve the gray-white
E. Parietal lobe gray-white junction junction, but are uncommon in children.

72. A 39-year-old HIV-positive man has received no anti- retroviral therapy. D. Large B-cell lymphoma
He has had left-sided weakness for the past month and experienced a Non-Hodgkin lymphomas, including large B-cell lymphomas, are
generalized seizure a day ago. On physical examination, he is afebrile. CT uncommon in the brain, but they may be seen in immunocompromised
scan of the head shows no intracranial hemorrhage, but there is a midline patients, particularly patients with AIDS. They tend to be multifocal.
shift. MRI of the brain shows a 4-cm mass in the region of the puta- men Cytomegalovirus infection is common in AIDS, but it is unlikely to produce
near the right internal capsule, a 3-cm mass in the right centrum mass lesions. A glioblastoma can be a large infiltrative and destructive
semiovale, and a 1-cm mass near the splenium of the corpus callosum. mass, but the cells are typically glial fibrillary acidic protein (GFAP) positive
These masses are circumscribed and solid. CSF from a lumbar puncture and CD19 negative. Kaposi sarcoma can occur in association with HIV
shows an elevated protein concentration and a normal glucose level. infection, but CNS involvement is rare, and the cells are CD34 positive.

❗USE AT YOUR OWN RISK❗ page 52 of 59


ROBBINS REVIEW

Cytologic examination shows large cells with large nuclei and scant Progressive multifocal leukoencephalopathy produces granular white
cytoplasm that mark with CD19, but not with GFAP or cytokeratin. What is matter lesions with large, bizarre oligodendrocytes infected with JC
the most likely diagnosis? polyomavirus. Toxoplasmosis can produce multiple mass lesions, but they
A. Cytomegalovirus encephalitis are chronic abscesses that are filled with necrotic material and surrounded
B. Glioblastoma by gliosis; Toxoplasma pseudocysts may be found in the abscesses
C. Kaposi sarcoma
D. Large B-cell lymphoma
E. Progressive multifocal leukoencephalopathy
F. Toxoplasmosis

73. A 45-year-old woman has had unilateral C. Meningioma


headaches on the right for the past 5 months.
Physical examination yields no remarkable This tan-yellow mass is a meningioma that typically is a circumscribed
findings. The representative gross appearance of lesion that arises from meningothelial cells of the arachnoid and appears
the lesion seen on CT scan of the head is shown as though it is grossly attached to the overlying dura. They are most often
in the figure. The mass is surgically removed and seen in women. A parasagittal, sphenoid ridge, or subfrontal location, is
microscopic examination shows elongated cells typical. Most meningiomas are biologically benign, but they can invade and
with pale, oblong nuclei and pink cytoplasm with some are more aggressive. Gliomas, including astrocytomas, are most
occasional psammoma bodies. Cytogenetic often found within a cerebral hemi- sphere in an adult. Ependymomas are
analysis shows 22q-. What is the most likely seen within ventricles or on the distal spinal cord. The parasagittal location
diagnosis? is unusual for a metastatic lesion, although some malignancies, such as
breast carcinomas, may involve the meninges in a diffuse fashion (so-
called carcinomatous meningitis). Tuberculomas are granulomas large
A. Astrocytoma enough to produce a mass effect and are rare complications of
B. Ependymoma disseminated tuberculosis and often appear at the base of the brain
C. Meningioma
D. Metastasis
E. Tuberculoma

74. A 76-year-old man has a single episode of grand mal seizure. On D. Metastatic carcinoma
physical examination, he is afebrile and normotensive. Motor strength is Multiple discrete neoplasms in the CNS found at the graywhite junction are
intact, and there is no loss of sensation. Cranial nerves are intact. His more likely to be metastases than a primary brain tumor. Tumor cells may
mental function is not diminished. There is a 1-cm, darkly pigmented skin reach the brain in the form of emboli through the cerebral arterial
lesion on the upper back. Brain MRI shows three solid, 1- to 3-cm mass circulation. Most embolic events occur at the gray-white junction, where
lesions, without ring enhancement or surrounding edema, located at the narrowing and acute branching of the vessels tend to trap emboli. The
gray-white junction in the right and left frontal lobes. The cerebral ventricles distribution of the middle cerebral artery, which receives the most blood, is
appear normal in size. What is the most likely diagnosis? the most likely location. Metastases from malignant melanomas are often
A. Glioblastoma widely disseminated, with multiple mass lesions in organ sites of
B. Hemangioblastoma involvement. Glioblastomas are large, malignant primary glial neoplasms;
C. Meningioma they are invasive, but do not typically appear as multiple small lesions. A
D. Metastatic carcinoma hemangioblastoma is a rare cystic mass in adults, typically arising in the
E. Non-Hodgkin lymphoma cerebellum, and may be associated with polycythemia. Meningiomas are
F. Oligodendroglioma circumscribed, solid mass lesions adjacent to the dura; they may be
multiple in patients with neurofibromatosis. Cerebral malignant lymphomas
are rare except in HIV-positive patients. Oligodendrogliomas are solitary,
circumscribed mass lesions that occur in the cerebral hemispheres of
adults

75. An 18-year-old student has had decreased vision in her right eye for 6 E. Schwannoma
months. On physical examination, there is papilledema on the right. She The multiple pigmented skin lesions and optic nerve glioma strongly
has 14 scattered, 2- to 5-cm flat, hyperpigmented skin lesions with irregular suggest the diagnosis of neurofibromatosis type 1. Patients with this
borders on the extremities and torso. CT scan of the head shows no condition, which has an autosomal dominant inheritance, have multiple
intracranial hemorrhage and no edema or midline shift, but there is a mass large café-au-lait spots on the skin, and there is a propensity for
in the region of the right optic nerve. An optic nerve glioma is excised. Eight development of multiple nerve sheath tumors (schwannomas or
months later, she returns for a follow-up examination, and a mass is neurofibromas). CNS gliomas also may occur. The nerve sheath tumors
palpated on the right wrist. Histologic examination of the mass is most may become malignant and metastasize, most commonly to the lungs. Of
likely to show which of the following neoplasms? the other neoplasms listed, only meningioma is associated with
A. Fibrosarcoma neurofibromatosis type 1, but it occurs intracranially.
B. Lipoma
C. Hemangioma
D. Meningioma
E. Schwannoma

76. A 41-year old has had diminished hearing for the past four months. On A. NF2
physical examination, she has decreased hearing on the left. Sound A schwannoma in this location also is known as a cerebellopontine angle
lateralizes to the right ear on the Weber tuning fork test. A head MRI shows tumor. Schwannomas in this location arise from cranial nerve VIII; they are
a sharply circumscribed, 4-cm mass adjacent to the left pons that extends also called acoustic neuro- mas. Most schwannomas act as benign, slow
toward the left inferior cerebellar hemisphere. A smaller 1-cm lesion is in a growing tumors that can be completely resected. Having bilateral acoustic
similar location on the right. Family screening reveals a similarly affected schwannomas is virtually pathognomonic for neurofibromatosis type 2.
38-year old sibling. An inherited mutation involving which of the following Other familial tumor syndromes involving brain include Li Fraumeni
genes is most likely to be present in this patient? syndrome with p53 mutations and medulloblastomas, Gorlin syndrome
A. NF2 with PTCH mutations and medulloblastomas, tuberous sclerosis with
B. TP53 TSC1 mutations and cortical hamartomas, and von Hippel–Lindau
C. PTCH syndrome with VHL mutations and hemangioblastomas.
D. TSC1

❗USE AT YOUR OWN RISK❗ page 53 of 59


ROBBINS REVIEW

E. VHL

77. A 20-year-old woman with learning difficulties had flank pain for 1 E. Tuberous sclerosis
week. Physical examination showed right costovertebral angle Tuberous sclerosis is one of the phakomatoses, a group of rare inherited
tenderness. Patches of leathery-appearing (shagreen patches) and disorders in which hamartomas and neoplasms develop throughout the
hypopigmented (ashleaf patches) skin were scattered over her body. body, along with cutaneous abnormalities. Patients with tuberous sclerosis
There was a subungual nodule on her right index finger. Abdominal CT have cortical tubers, which are hamartomas of neuronal and glial tissue;
scan showed bilateral renal cysts and tumor masses. MRI of the brain other characteristic findings include renal angiomyolipomas, renal cysts,
showed subependymal nodules and 1- to 4-cm cortical foci with loss of the subungual fibromas, and cardiac rhabdomyomas. In Down syndrome
gray-white distinction. CT scan of the chest showed a 3-cm mass involving (trisomy 21), patients may develop acute leukemia, but not brain
the interventricular septum. Two years later, she now has a sudden, neoplasms, and individuals who survive to middle age develop Alzheimer
severe headache. MRI now shows a nodule obstructing the cerebral disease. Krabbe disease is a leukodystrophy that results in deficiency of
aqueduct. Neurosurgery is performed, and a subependymal giant cell galactocerebroside à-galactosidase and an onset of neurologic
astrocytoma is removed. What is the most likely diagnosis? deterioration in infancy. Neurofibromatosis type 1 is characterized by
A. Down syndrome deforming cutaneous and visceral neurofibromas, cutaneous café-au-lait
B. Krabbe disease spots, and neurofibrosarcomas. In neurofibromatosis type 2, acoustic
C. Neurofibromatosis type 1 schwannomas, meningiomas, gliomas, and ependymomas are present.
D. Neurofibromatosis type 2 Von Hippel–Lindau disease is characterized by hemangioblastomas in the
E. Tuberous sclerosis cerebellum, retina, and spinal cord, and by pheochromocytomas
F. Von Hippel–Lindau disease

78. A study of adults with cerebellar neoplasms reveals that some of them C. PTCH
have an autosomal dominant inheritance pattern with von Hippel– Lindau Hemangioblastomas are part of the spectrum of neoplasms seen in
disease. Their cerebellar tumors are cystic with a mural nodule. Molecular association with von Hippel–Lindau dis- ease. The VHL gene encodes for
analysis of tumor cells shows increased amounts of hypoxia-inducible a ubiquitin-ligase component important in the degradation pathway of HIF,
factor (HIF). The incidence of renal cell carcinomas is increased in these and this increases vascular endothelial growth factor (VEGF) expression
persons. Which of the following paraneoplastic manifestations is most to drive vascular proliferation. Cushing syndrome with increased
likely to be found in these adults? corticosteroid production can be related to ectopic ACTH production by a
A. NF2 carcinoma. Cushing disease is caused by an ACTH-producing
B. TP53 adenohypophyseal adenoma. Hypercalcemia of malignancy often results
C. PTCH from parathormone-related peptide production by a carcinoma. Syndrome
D. SIADH of inappropriate antidiuretic hormone (SIADH) may be caused by
E. Trousseau syndrome neuroendocrine tumors such as small cell lung carcinomas. Carcinomas,
particularly adenocarcinomas, may produce hypercoagulability
(Trousseau syndrome)

PERIPHERAL NERVES AND SKELETAL MUSCLES

QUESTION RATIONALE

1.A 62-year-old woman has a slowly enlarging mass anterior to the right E. Fragmentation of distal axons and myelin sheaths
ear. Surgery is performed to remove a pleomorphic adenoma of the parotid The axons distal to the point of injury or transection degenerate. The
gland. The tumor has infiltrated the overlying soft tissue, and the surgeon Schwann cells that remain can guide the re- growing nerves. Macrophages
must remove a portion of the facial nerve to obtain an adequate margin. help to remove myelin-derived debris from the area of nerve injury, but
He places a 2-cm nerve graft in the excised area. Which of the following acute inflammation is not a typical feature of diseases involving peripheral
best describes the most likely outcome during the first week after surgery? nerves. Traumatic neuromas may occur after transection, but careful
dissection prevents this, and the purpose of the graft is to guide orderly
A Acute inflammatory cells around the graft regrowth. A tumor is unlikely to follow a nerve, although a feature of a
B Formation of a traumatic neuroma malignant tumor is a tendency to invade nerves. Segmental demyelination
C Recurrent tumor along the nerve graft is more typical for diabetic neuropathy.
D Segmental demyelination and axonal loss of the nerve proximal to the
graft
E Fragmentation of distal axons and myelin sheaths

2.A 16-year-old boy has a deep laceration of the left lower thigh. The D. 6 months
bleeding is stopped. On physical examination, he has lost sensation in the Laceration of the tibial nerve results in Wallerian degeneration distal to the
lateral left foot and movement in the left foot. The wound is surgically injury. Realignment of the nerve is accompanied by axonal sprouting. The
repaired, including the nerve, and he receives physical therapy. How long new axons find the residual myelin sheaths and grow down at the rate of
will it take him to regain the use of his left foot? about 2 mm/day, taking 1 year to traverse the length of the calf. There can
be reinnervation of the muscle, but there is type grouping of the muscle
A 1 day fascicles that are reinnervated. Physical therapy can aid in the interim.
B 1 week Complete restoration of motor and sensory functions may not occur.
C 1 month
D 6 months

3.A 60-year-old man has pain in the left neck, shoulder, and forearm that B. Axon
has been worsening for a month. Over the next month there is increasing The distribution is that of C7, likely from a herniated disc compressing the
weakness of the left pectoralis major, latissimus dorsi, and triceps nerve root. The digital branch of the median nerve is purely C7. Nerve
muscles. Paresthesias of the tips of the thumb, forefinger, and ring finger entrapment or compression typically involves the axon. If the nerve is not
are present. There is no warmth, tenderness, or swelling. A lesion involving severed, then axonal regrowth can occur. Neurapraxia occurs with
which of the following structures is most likely to be present in this man? transient injury (e.g., when your foot “falls asleep”) because there is
conduction loss but no axonal damage. Axonotmesis involves axonal loss
A Vasculature and Wallerian degeneration, but with- out damage to the myelin sheath, so

❗USE AT YOUR OWN RISK❗ page 54 of 59


ROBBINS REVIEW

B Axon axonal regrowth can occur. Neurotmesis involves transection of the nerve,
C Ganglion and regrowth can only occur with exact alignment of the severed nerve so
D Myofiber axonal sprouts can find the proper myelin sheath. Vasculitis could involve
E Schwann cell an arteriole supplying a nerve, but is unlikely to involve a single nerve root.
The motor and sensory loss here is not consistent with injury to a ganglion.
The sensory component here means more than myofibers are in- volved,
and the myofibers supplied by C7 have neurogenic atrophy. The
paresthesias, not anesthesia, suggest the nerve was not severed, and
Schwann cells remained.

4.A 41-year-old man had an influenza-like illness for 1 week, followed 4 B. GBS
days later by rapidly progressive, ascending motor weakness requiring Guillain-Barré syndrome is an uncommon disorder that most often follows
mechanical ventilation. On physical examination, he is now afebrile and a bacterial (Campylobacter jejuni), viral (cytomegalovirus), or
has 3/5 motor strength in his extremities. A lumbar puncture is done and mycoplasmal infection and is thought to be caused by generation of
yields clear, colorless cerebrospinal fluid under normal pressure. This fluid myelin-reactive T cells, or by molecular mimicry, somehow triggered by the
has a slightly elevated protein concentration, but a normal glucose level, infection. The paralysis of respiratory muscles may be life threatening,
and a cell count with only a few mononuclear cells. He recovers in 3 weeks. although many patients recover after weeks of ventilatory support.
If lymphocytic infiltrates were seen in peripheral nerves along with Amyotrophic lateral sclerosis is associated with slowly progressive muscle
segmental demyelination at the time he initially saw his physician, what weakness. Various presentations are possible in multiple sclerosis, but the
would be the most likely diagnosis? plaques of demyelination are generally not large or diffuse enough to
cause paralysis of the respiratory muscles. Mechanical ventilation may be
A Amyotrophic lateral sclerosis necessary eventually. Varicella-zoster virus infection most often involves
B Guillain-Barré syndrome the skin in a dermatomal distribution from a spinal nerve root. Vitamin B12
C Multiple sclerosis deficiency results in subacute progressive degeneration of the spinal cord
D Varicella-zoster virus infection plus sensorimotor disturbances in the extremities
E Vitamin B12 (cobalamin) deficiency

5.A 37-year-old HIV-positive man has had a relapsing and remitting course D. Immune dysregulation
of motor and sensory problems for the past year, including difficulty with Chronic inflammatory demyelinating polyneuropathy (CIDP) can be seen
ambulation as well as symmetric numbness and tingling in all extremities. in patients with immunologic diseases. Treatment may aid recovery, but a
Nerve conduction studies show findings consistent with demyelination and chronic course may ensue. Bacterial infections produce signs of acute
remyelination. He is treated with plasmapheresis. Which of the following inflammation with redness and swelling, and tend not to involve nerves
disorders is most likely to cause this man’s neurologic disease? specifically. Some paraneoplastic syndromes may occur with carcinomas,
such as Lambert-Eaton myasthenic syndrome, with weakness.
A Bacterial infection Carcinomas are less likely at his age, and are not frequent complications
B Carcinoma of HIV infection. Hyperglycemia is characteristic of diabetes mellitus, and
C Hyperglycemia diabetic neuropathy is likely to be progressive
D Immune dysregulation and unremitting. Traumatic injury is unlikely to produce such widespread
E Traumatic injury findings and unlikely to be followed by a variable course.

6.A 93-year-old woman has been bothered by continuing outbreaks of E. VZV


painful lesions on the skin of her right chest for the past year. On physical Following infection with chickenpox, the varicella- zoster virus (VZV)
examination, there is a vesicular eruption in a 1 × 8 cm area over the right becomes dormant in dorsal root ganglia, only to reactivate later when the
seventh rib. She is treated with acyclovir, and partial resolution of the skin immune system no longer contains it. Such containment failures are more
lesions occurs, but the pain persists for the next 3 months. Which of the likely with age and with immunocompromised states, and this produces
following is the most likely cause for her findings? the classic appearances of shingles in a dermatomal distribution. She also
has postherpetic neuralgia, a disabling condition of chronic pain that is
A Aging difficult to manage. Aging alone does not explain disease, but older
B Diabetes mellitus individuals are more likely to have disease conditions. Diabetes mellitus
C Multiple sclerosis leads to neuropathies that can have loss of sensation or even pain, but
D Somatoform pain disorder there are no associated skin lesions. Multiple sclerosis can involve the
E Varicella-zoster virus infection spinal cord white matter with variable neurologic findings, but there are no
F Vitamin B12 (cobalamin) deficiency skin lesions, and the onset of MS is at a younger age. Pain out of
proportion to the pathologic findings suggests a somatoform pain disorder,
but she does have significant pathologic findings that explain her continued
pain. Cobalamin deficiency with pernicious anemia and subacute
combined degeneration of the spinal cord can lead to paresthesias and
loss of function, but not to skin lesions.

7.A 66-year-old man receiving hemodialysis for chronic renal failure has B. Diabetes mellitus
noted increasing loss of sensation in his legs for the past 4 years. On The most common cause of a predominantly sensory peripheral
physical examination, there is symmetrically decreased sensation over neuropathy is diabetes mellitus. Long-standing diabetes mellitus also
both lower extremities. He has no decrease in strength or abnormality of gives rise to nephropathy with chronic renal failure. Sensorimotor
gait. Which of the following is most likely to produce these findings? disturbances are typically not seen with intracranial mass lesions such as
astrocytomas. Cerebral infarctions could lead to decreased motor activity
A Cerebral astrocytoma and/or sensory loss, although not usually in a symmetric pattern; small
B Diabetes mellitus infarcts culminating in dementia may not produce significant motor or
C Hansen disease sensory loss. Hansen disease (leprosy) produces focal anesthesia. The
D Multi-infarct dementia demyelinating lesions of multiple sclerosis in different white matter sites
E Multiple sclerosis and at different times can produce many signs and symptoms, but
F Uremia symmetric lesions should suggest another disease process. Uremic
neuropathy resembles diabetic neuropathy, but it typically regresses with
dialysis.

8.A 55-year-old man has had a foot ulcer for 2 months that has not healed. D. Segmental demyelination
Physical examination shows a 2-cm shallow, nonhealing ulceration of the

❗USE AT YOUR OWN RISK❗ page 55 of 59


ROBBINS REVIEW

left medial malleolus. There is symmetric decreased sensation in the distal The features described are consistent with peripheral neuropathy
regions of the lower extremities. He has a history of multiple urinary tract associated with diabetes mellitus. Both motor and sensory nerves can be
infections resulting from difficulty in completely emptying the bladder. He involved, and there may be autonomic neuropathy. Histologic examination
is impotent. Which of the following pathologic findings is most likely to be shows an axonal neuropathy with segmental demyelination. Difficulty in
present in the peripheral nerves? emptying the urinary bladder and impotence are results of autonomic
neuropathy. Longer nerves are affected first; this explains the lower leg
A Acute inflammation involvement and accounts for many cases of diabetic foot, with trauma and
B Axonal neuropathy subsequent ulceration. Acute inflammation is not generally seen in
C Onion bulb formation neuropathies. Lymphocytic infiltrates may be seen in Guillain-Barré
D Segmental demyelination syndrome. Onion bulb formation is a feature of hereditary neuropathy
E Wallerian degeneration known as Refsum disease. Wallerian degeneration typically occurs with
traumatic transection of a nerve.

9.A 24-year-old woman has had episodes of numbness and tingling in both C. Entrapment with compression
hands for 5 months. The problem is worse near the end of the day and Carpal tunnel syndrome is a form of compression neuropathy that results
makes it difficult for her to use the computer keyboard. The thumb and first from entrapment of the median nerve beneath the flexor retinaculum at the
two fingers are most affected. There is no pain or swelling, and she does wrist. Women are more commonly affected than men, and the problem is
not recall any trauma to the upper extremities. On physical examination, often bilateral. The role of excessive repetitive use of the wrist in causation
she has a positive Tinel sign and decreased sensation to light touch and has been debated, but lifestyle modifications and ergonomics can be
pinprick over the palmar surface of both hands in the distribution of the first employed as initial conservative therapy. Conditions such as
three digits. Thenar muscle atrophy is present. This neuropathy is most hypothyroidism, amyloidosis, and edema with pregnancy also diminish the
likely due to which of the following underlying causes? space in the carpal tunnel. Acute intermittent porphyria can lead to a
hereditary form of motor and sensory neuropathy. Diabetes mellitus leads
A Acute intermittent porphyria to a more widespread symmetric distal sensorimotor neuropathy. Systemic
B Diabetes mellitus, type 2 lupus erythematosus can produce arthralgias and myalgias, but not
C Entrapment with compression typically focal nerve injury. Infection with varicella-zoster virus produces a
D Systemic lupus erythematosus painful neuropathy in a dermatomal distribution pattern.
E Varicella-zoster virus infection

10. A 41-year-old woman has noted marked pain in the right foot for the C. Entrapment neuropathy
past 2 months. The pain makes it difficult for her to wear high heeled shoes Fashion may have a price. The patient has a Morton neuroma, a form of
and seems to be worse at the end of the day. On physical examination, compressive neuropathy in which a plan- tar nerve is trapped between
she has severe pain on palpation of the interdigital space between the metatarsal heads. Chronic injury leads to growth of a tangled mass of
second and third toes. There is no swelling or erythema of the foot. Motor axons, fibroblasts, and perineural cells. Diabetic neuropathy, which occurs
strength in the lower extremities seems to be normal. What has most likely in either type 1 or type 2 disease, is bilateral. It is characterized by loss of
produced these findings? sensation, which may be a predisposing factor for foot trauma. Toxic
disorders (lead poisoning) and metabolic disorders (beriberi with thiamine
A Beriberi deficiency) are not as focal. Wallerian degeneration is a dying-back
B Type 2 diabetes mellitus neuropathy associated with severing of a nerve; a neuroma may form at
C Entrapment neuropathy the site of injury.
D Lead poisoning
E Wallerian degeneration

11. A 58-year-old man has experienced worsening double vision and A. Acetylcholine receptor antibody positivity Myasthenia gravis leads
eyelid drooping, particularly toward the end of the day, for 1 month. He to muscle weakness from loss of motor end plate function. This disorder is
also has had difficulty chewing his food at dinner. He was diagnosed with caused by an antibody-mediated loss of acetylcholine receptors at
Sjögren syndrome more than a decade ago. On physical examination, he neuromuscular junctions. Edrophonium inhibits cholinesterase to improve
has 5/5 motor strength in his extremities that decreases to 4/5 strength synaptic transmission. The anti–Jo-1 anti- body is present in polymyositis.
with repetitive movement. Administration of edrophonium restores muscle Sjögren syndrome does not cause muscle weakness with use. Myopathic
strength. There is no pain on palpation and no decrease in joint mobility. diseases such as muscular dystrophies are accompanied by increased
Which of the following laboratory findings is most likely to be reported for levels of serum creatine kinase. Generalized muscle weakness with type
this patient? II muscle fiber atrophy occurs in glucocorticoid excess. Parasitic infection
with Trichinella spiralis can lead to
A Acetylcholine receptor antibody positivity eosinophilia.
B Anti–histidyl tRNA synthetase (anti–Jo-1) titer 1:512
C Elevated serum creatine kinase level
D Increased serum cortisol level
E Peripheral blood eosinophilia

12. A 72-year-old man has had a 7-kg weight loss, proximal muscle E. Small cell lung carcinoma Lambert-Eaton myasthenic syndrome is a
weakness, and difficulty with urination for the past 4 months. On physical rare form of paraneoplastic syndrome. Proximal muscles tend to be in-
examination, he has 4/5 muscle strength in his proximal extremities that volved first. Similar to many paraneoplastic syndromes, it is most often
does not diminish with repetitive motion. He has no muscle pain or loss of associated with small cell carcinoma of the lung. The autoantibodies are
mobility. Laboratory studies show that he does not have serum antibodies directed against calcium channels at motor nerve terminal membranes.
to acetylcholine receptors. He was prescribed anticholinesterase agents Patients with chronic viral hepatitis may have generalized malaise and
but shows no improvement. Which of the following underlying conditions weakness that is not related to specific muscle disease. Diabetes mellitus
is most likely to be present in this man? also may produce peripheral neuropathy, but more often involving distal,
not proximal, regions first. Duchenne muscular dystrophy is an X-linked
A Chronic hepatitis C disease that manifests early in childhood. Lead poisoning leads to
B Diabetes mellitus peripheral neuropathy, often with central nervous system findings.
C Duchenne muscular dystrophy
D Lead poisoning
E Small cell lung carcinoma

13. A previously healthy 40-year-old man and his 42-year- old wife have A. Botulism
had increasing blurred vision and weakness for the past day. On

❗USE AT YOUR OWN RISK❗ page 56 of 59


ROBBINS REVIEW

examination they are afebrile. Orthostatic hypotension is present. They They consumed food contaminated with Clostridium botulinum toxin. This
have 3/5 muscle weakness in all extremities and difficulty breathing. They potent neurotoxin disrupts the neuromuscular junction by inhibiting
are treated with intubation and mechanical ventilation. Which of the acetylcholine release. This anaerobic organism grows in canned foods that
following is the most likely cause for their paralysis? lack acidity. Cocaine is a powerful vasoconstrictor but does not affect
nerves directly. Diabetic neuropathy tends to be slowly progressive.
A Botulism Guillain-Barré syndrome produces an ascending paralysis. Thymoma is
B Cocaine ingestion associated with myasthenia gravis that tends to progress more slowly,
C Diabetes mellitus though a superimposed myasthenic crisis with respiratory difficulty can
D Guillain-Barré syndrome occur acutely.
E Thymoma

14. A 42-year-old man has had increasing progressive muscle weakness A. ALS
in both arms and legs along with dysarthria and difficulty in swallowing for Amyotrophic lateral sclerosis is also known as Lou Gehrig disease, named
the past 2 years. He is now wheelchair-bound. Physical examination for the famous New York Yankee first baseman affected with the disorder.
shows 3/5 motor strength in all extremities. He has no muscle pain on Lower and upper motor neurons can be affected, resulting in a denervation
palpation, no deformities or loss of joint mobility, and no tremor. A biopsy type of pattern of muscular atrophy. This occurs because an individual
specimen of the quadriceps muscle is obtained, and microscopic neuron innervates a group of muscle fibers. Bulbar (cranial nerve)
examination shows no inflammation, but only atrophy of the myofibers. involvement denotes a more rapid course. Werdnig-Hoffman disease also
What is the most likely diagnosis? is a neuropathic disease with grouped atrophy, but onset is in infancy. The
other listed options are not associated with denervation.
A Amyotrophic lateral sclerosis
B Becker muscular dystrophy
C Mitochondrial myopathy
D Myasthenia gravis
E Werdnig-Hoffmann disease

15. A 30-year-old woman has had gradually increasing muscle weakness A. Antibody- and complement-mediated injury to the
with myalgia for the past year. She now has difficulty getting up from a microvasculature
chair and climbing stairs. She does not have weakness in her hand Dermatomyositis is an immunologically mediated form of inflammatory
muscles. Physical examination reveals a fine violaceous rash on her face, myopathy. In these patients, antibodies and complement damage small
pre- dominantly palpebral. Dusky, flat, red patches are present on her capillaries, resulting in characteristic perifascicular myofiber atrophy.
elbows, knees, and knuckles. Laboratory studies show serum creatine Expansion of CTG repeats is seen with myotonic dystrophy. Mutations in
kinase of 620 U/L. A deltoid biopsy specimen is obtained, and microscopic ion channel genes give rise to various channelopathies, including
does not have serum antibodies to acetylcholine receptors. He was hypokalemic periodic paralysis and malignant hyperthermia. The CD8+ T
prescribed anticholinesterase agents but shows no improvement. Which cells are believed to be important in the pathogenesis of polymyositis. T
of the following underlying conditions is most likely to be present in this cell–mediated myelin in generalized malaise and weakness that is not
man? related to specific muscle disease. Diabetes mellitus also may produce
peripheral neuropathy, but more often involving distal, not proximal,
A Chronic hepatitis C regions first. Duchenne muscular dystrophy is an X-linked disease that
B Diabetes mellitus manifests early in childhood. Lead poisoning leads to peripheral
C Duchenne muscular dystrophy neuropathy, often with central nervous system findings.
D Lead poisoning
E Small cell lung carcinoma

13. A previously healthy 40-year-old man and his 42-year- old wife have A. Botulism
had increasing blurred vision and weakness for the past day. On They consumed food contaminated with Clostridium botulinum toxin. This
examination they are afebrile. Orthostatic hypotension is present. They potent neurotoxin disrupts the neuromuscular junction by inhibiting
have 3/5 muscle weakness in all extremities and difficulty breathing. They acetylcholine release. This an- aerobic organism grows in canned foods
are treated with intubation and mechanical ventilation. Which of the that lack acidity. Cocaine is a powerful vasoconstrictor but does not affect
following is the most likely cause for their paralysis? nerves directly. Diabetic neuropathy tends to be slowly progressive.
Guillain-Barré syndrome produces an ascending paralysis. Thymoma is
A Botulism associated with myasthenia gravis that tends to progress more slowly,
B Cocaine ingestion though a superimposed myasthenic crisis with respiratory difficulty can
C Diabetes mellitus occur acutely.
D Guillain-Barré syndrome
E Thymoma

14. A 42-year-old man has had increasing progressive muscle weakness A. ALS
in both arms and legs along with dysarthria and difficulty in swallowing for Amyotrophic lateral sclerosis is also known as Lou Gehrig disease, named
the past 2 years. He is now wheelchair-bound. Physical examination for the famous New York Yankee first baseman affected with the disorder.
shows 3/5 motor strength in all extremities. He has no muscle pain on Lower and upper motor neurons can be affected, resulting in a denervation
palpation, no deformities or loss of joint mobility, and no tremor. A biopsy type of pattern of muscular atrophy. This occurs because an individual
specimen of the quadriceps muscle is obtained, and microscopic neuron innervates a group of muscle fibers. Bulbar (cranial nerve)
examination shows no inflammation, but only atrophy of the myofibers. involvement denotes a more rapid course. Werdnig-Hoffman disease also
What is the most likely diagnosis? is a neuropathic disease with grouped atrophy, but onset is in infancy. The
other listed options are not associated with denervation.
A Amyotrophic lateral sclerosis
B Becker muscular dystrophy
C Mitochondrial myopathy
D Myasthenia gravis
E Werdnig-Hoffmann disease

15. A 30-year-old woman has had gradually increasing muscle weakness A. Antibody- and complement-mediated injury to the
with myalgia for the past year. She now has difficulty getting up from a microvasculature
chair and climbing stairs. She does not have weakness in her hand Dermatomyositis is an immunologically mediated form of inflammatory
muscles. Physical examination reveals a fine violaceous rash on her face, myopathy. In these patients, antibodies and complement damage small

❗USE AT YOUR OWN RISK❗ page 57 of 59


ROBBINS REVIEW

pre- dominantly palpebral. Dusky, flat, red patches are present on her capillaries, resulting in characteristic perifascicular myofiber atrophy.
elbows, knees, and knuckles. Laboratory studies show serum creatine Expansion of CTG repeats is seen with myotonic dystrophy. Mutations in
kinase of 620 U/L. A deltoid biopsy specimen is obtained, and on ion channel genes give rise to various channelopathies, including
microscopic examination shows a mononuclear inflammatory cell infiltrate hypokalemic periodic paralysis and malignant hyperthermia. The CD8+ T
around small blood vessels and groups of atrophic myofibers at the cells are believed to be important in the pathogenesis of polymyositis. T
periphery of fascicles. What mechanism is most likely responsible for her cell–mediated myelin in jury is seen with Guillain-Barré syndrome, causing
disease? an acute ascending paralysis.

A Antibody- and complement-mediated injury to the microvasculature


B Expansion of CTG repeat sequences on chromo- some 19q13.2
C Mutation in a gene encoding for voltage gated calcium channels
D Myofiber injury by CD8+ cells directed against muscle antigens
E T cell–mediated nerve injury induced by Myco- plasma pneumoniae
infection

16. A 56-year-old woman has had increasing generalized muscle A. Cushing syndrome
weakness for the past 2 months. On physical examination, she has 4/5 Type II muscle fiber atrophy can occur with glucocorticoid excess and also
motor strength in upper and lower extremities. She has fat redistribution in after prolonged immobilization. Routine light microscopy may not
the upper trunk and rounded facies. Ecchymoses are scattered over the distinguish type II atrophy from denervation atrophy. Histochemical
extremities. She is afebrile, and her blood pressure is 155/90 mm Hg. A staining for ATPase could be done (but a biopsy is not needed, given this
biopsy specimen of the gastrocnemius muscle is obtained, and history). There is a deficiency of myophosphorylase enzyme in McArdle
histochemical staining with ATPase shows type II muscle fiber atrophy. disease, leading to muscle pain and cramping with vigorous exercise.
What is the most likely diagnosis? Duchenne muscular dystrophy is an X-linked condition and is rare in
females. Onset is in early childhood. Antibodies to the acetylcholine
A Cushing syndrome receptor cause muscular weakness in myasthenia gravis. Polymyositis is
B McArdle disease an inflammatory condition affecting all fiber types.
C Duchenne muscular dystrophy
D Myasthenia gravis
E Polymyositis

17. A 71-year-old woman is receiving a drug to lower her serum D. Lovastatin


cholesterol. Over the past week she has developed muscle pain and She has a statin-induced myopathy, with a creatine kinase level more than
weakness unrelated to physical activity. On examination she has diffuse 10 times normal from rhabdomyolysis (myoglobin released from muscle
but mild muscle tenderness. Laboratory studies show her serum creatine can be detected by the urine dipstick). Statins are HMG-CoA reductase
kinase is 2049 U/L and creatinine is 2 mg/dL. Urine dipstick analysis is inhibitors that reduce endogenous cholesterol synthesis in the liver.
positive for blood, without RBCs on urine microscopy. Which of the Cholestyramine binds bile acids in the intestine and disrupts enterohepatic
following drugs is most likely to produce her findings? bile acid circulation to increase conversion of cholesterol to bile acids in
the liver. Clofibrate enhances up- take and oxidation of free fatty acids in
A Cholestyramine muscle. Ezetimibe interferes with intestinal lipid absorption. Nicotinic acid
B Clofibrate inhibits mobilization of peripheral free fatty acids to reduce hepatic
C Ezetimibe triglyceride synthesis and secretion of VLDL
D Lovastatin
E Nicotinic acid

18. A 44-year-old man has had worsening exercise intolerance for the past B. Becker muscular dystrophy
year. On physical examination, he has 4/5 motor strength in the The biopsy specimen shows a reduced amount of dystrophin, but not a
extremities, but has no muscle pain or loss of joint mobility. He has pitting complete absence; this suggests Becker muscular dystrophy. In
edema to the knees. A chest radiograph shows cardiomegaly with Duchenne muscular dystrophy, dystrophin is absent because of gene
pulmonary edema and pleural effusions. A deltoid muscle biopsy deletion. In keeping with the diagnosis of Becker muscular dystrophy, the
specimen is obtained. The figure shows the immunohistochemical staining patient is older and not severely affected. Both dystrophies are X- linked
pattern with antibody to dystrophin (A, normal; B, patient). What is the most conditions. Amyotrophic lateral sclerosis is a denervation atrophy seen in
likely diagnosis? adults, with loss of anterior horn cells in the spinal cord and cranial nerve
nuclei. Myasthenia gravis results from antibodies to acetylcholine
A Amyotrophic lateral sclerosis receptors, and there is minimal structural change to the muscle.
B Becker muscular dystrophy Polymyositis is an autoimmune disease that results from a T cell–mediated
C Myasthenia gravis attack on muscle fibers, causing muscle fiber degeneration with
D Polymyositis inflammation. Werdnig- Hoffmann disease is a form of spinal muscular
E Werdnig-Hoffmann disease atrophy. Onset is at birth, and it results from a genetically determined loss
of anterior horn cells.

19. A 5-year-old boy develops increasing muscle weakness. He is unable B. Immunohistochemical staining for dystrophin
to play with other children because he quickly becomes tired and is unable The onset of muscle weakness in childhood suggests an inherited
to keep up with them. On physical examination, he is afebrile. No muscular dystrophy. The biopsy specimen shows variation in muscle fiber
deformities are noted. He has 4/5 muscle strength in his extremities, with size and increased connective tissue between the fibers. This morphologic
more apparent weakness of the proximal muscles. Laboratory studies finding in a boy strongly suggests X-linked muscular dystrophy.
show a serum creatine kinase level of 689 U/L. A muscle biopsy is done Immunohistochemical staining for dystrophin would show an absence of
and the microscopic appearance shown in the figure. Which of the dystrophin, confirming the diagnosis of Duchenne muscular dystrophy.
following tests would be most appropriate to confirm the diagnosis in this Eosinophilia may be present in allergic or parasitic disorders, including
boy? trichinosis. Expansion of CGG repeats on Xq27.3 is diagnostic of familial
mental retardation.
A Absolute eosinophil count in peripheral blood
B Immunohistochemical staining for dystrophin Oligoclonal immunoglobulin bands in the cerebrospinal fluid are a feature
C PCR to detect expansion of CGG repeats on chromosome Xq27.3 of multiple sclerosis. Antibodies to the acetylcholine receptor are found in
D Presence of oligoclonal bands of immunoglobulin in cerebrospinal fluid myasthenia gravis, which is characterized by weakness in muscles after
E Serum acetylcholinesterase antibody titer repetitive use.

❗USE AT YOUR OWN RISK❗ page 58 of 59


ROBBINS REVIEW

20. A 16-year-old boy has had two episodes of sudden loss of motor function with E. Deficient mitochondrial ATP generation
residual weakness in his right arm and right leg in the past 2 years. He has had ATP generation by oxidative phosphorylation in mitochondria can be affected by
muscle weakness and a seizure disorder since childhood. During the past year, he mitochondrial genes, which are separate from those on chromosomes in the cell
has had difficulty with memory and performing activities of daily living. On physical nucleus. These abnormal genes can lead to mitochondrial myopathies,
examination, he has short stature. He has 4/5 motor strength in all extremities, with encephalopathies, and deafness. In this case, there is mitochondrial encephalopathy
no muscle tenderness. Laboratory studies show glucose, 71 mg/dL; creatinine, 1.1 with lactic acidosis and stroke-like episodes (MELAS). Mitochondrial genes have a
mg/dL; and lactic acid, 9.2 mmol/L. A gastrocnemius muscle biopsy specimen is maternal pattern of inheritance. An abnormal voltage-gated calcium channel is seen
obtained, and microscopic examination shows ragged red fibers. On electron in one of the channelopathies; it causes hypokalemic periodic paralysis.
microscopy, the myofibrils have “parking lot” inclusions. The boy’s mother and
grandmother had similar findings, but his father and grandfather did not. Which of the Channelopathies are typically inherited in an autosomal dominant fashion. Antibodies
following most likely explains the pathogenesis of his disease? to acetylcholine receptor cause myasthenia gravis; its sole manifestation is muscle
weakness. Cytotoxic CD8+ cells mediate the muscle injury in polymyositis.
A Abnormal voltage-gated calcium channel Decreased sarcolemmal dystrophin is present in Becker muscular dystrophy;
B Antibodies to acetylcholine receptor dystrophin is absent in Duchenne muscular dystrophy. The dystrophin gene is
C Cytotoxic CD8+ lymphocytes against myofibers located on the X chromosome. Increased CTG repeats occur in myotonic dystrophy.
D Decreased sarcolemmal dystrophin
E Deficient mitochondrial ATP generation
F Increased CTG repeat sequences at 19q13.2–13.3

21. A 10-year-old girl has exhibited muscular weakness since early childhood that B. Congenital myopathy
has not worsened. She can ambu- late unassisted, but does not participate in Nemaline rod myopathy, one form of congenital myopathy, may present in infancy or
strenuous physical activities. On examination she has 4/5 motor strength in proximal childhood and may be non- progressive or slowly progressive. Congenital
muscles and 5/5 in distal muscles. There is no mus- cle pain on palpation. A biopsy myopathies are often named for their characteristic histologic features.
of deltoid muscle is obtained, and with Gomori trichrome stain, microscopic analysis Channelopathies typically present as periodic paralysis with abnormalities in serum
shows subsarcolemmal aggregates of rod shaped intracyto- plasmic inclusions. potassium. The two glycogen storage diseases most often affecting striated muscle
Laboratory studies show a normal serum creatine kinase. Which of the following is include Pompe disease (type II) and McArdle disease (type V). There are increased
the most likely form of muscle disease she has? glycogen deposits seen with periodic acid–Schiff (PAS) stain. Neuropathies affect
muscle through denervation, leading to groups of atrophic muscle fibers. Polymyositis
A Channelopathy and dermatomyositis are inflammatory myopathies that are accompanied by myalgia
B Congenital myopathy and fiber degeneration with increased serum creatine kinase. Mitochondrial
C Glycogen storage disease myopathies may appear in childhood but are usually progressive, and often other
D Inflammatory myopathy organs such as heart or brain are involved.
E Mitochondrial myopathy

22. An infant born at term exhibits difficulty with movement beginning at 1 month of F. Spinal muscular atrophy
age. By 1 year of age, there is flaccid paralysis. A muscle biopsy is done, and Werdnig-Hoffman disease is a form of spinal muscular
microscopically shows perifascicular atrophy of myofibers with scattered enlarged atrophy resulting from loss of motor neurons in infancy, so the biopsy specimen
myofibers, but no inflammation. The serum creatine kinase is not elevated. What is shows grouped atrophy of myofibers. Death is inevitable by age 3 years. Amyotrophic
the most likely diagnosis? lateral sclerosis is a progressive disease with a neurogenic form of muscle atrophy
resulting from loss of motor neurons. Becker muscular dystrophy has an X-linked
A Amyotrophic lateral sclerosis pattern of inheritance and onset in adulthood, unlike Duchenne muscular dystrophy,
B Becker muscular dystrophy but in both there is a mutation in the dystrophin gene. McArdle disease is an
C McArdle disease autosomal recessive condition resulting from a deficiency in muscle phosphorylase
D Myasthenia gravis and does not produce progressive weakness. Myasthenia gravis results from
E Myotonic dystrophy F Spinal muscular atrophy acetylcholine receptor antibody and leads to progressive weakness. Myotonic
dystrophy is characterized by facial and upper body weakness, cataracts, gonadal
atrophy, cardiomyopathy, and dementia.

23. A 40-year-old man undergoes elective laparoscopic hernia repair. He receives A. Calcium ion channel
anesthesia with halothane and succinylcholine. His blood loss is minimal. Thirty Malignant hyperthermia is a rare but life-threatening
minutes into this surgery, his temperature increases to 39.5° C, and pulse increases disorder seen in only 1 in 20,000 adults, but with millions of surgical procedures under
to 115/min. The anesthesiologist notices muscular spasms with rigidity of the anesthesia being performed, it must be considered when the patient’s temperature
extremities. Laboratory studies show an elevated serum creatine kinase and increases rapidly. In patients with mutations in the ryanodine receptor, there is
myoglobinuria. This man most likely has a mutation in a gene encoding for a protein impaired reuptake of calcium into the sarcoplasmic reticulum with increased
that regulates the function of which of the following? intracellular calcium leading to hypermetabolism. Motor end plates include
acetylcholine and acetylcholine receptors that are involved with myasthenia gravis.
A Calcium ion channel The mitochondrial DNA genes are mainly involved in oxidative phosphorylation, and
B Motor end plate there is a maternal in- heritance pattern, with possible muscular weakness, but not
C Oxidative phosphorylation enzyme hyperthermia. The sarcoglycan complex includes many proteins such as dystrophin
D Sarcoglycan complex that can be involved with mus- cular dystrophies, but not hyperthermia.
E Thick filament Thick filaments in- clude the myosin contractile protein; àmyosin mutations are found
in some cases of hypertrophic cardiomyopathy.

24. A 42-year-old woman has had an increasing number of subcutaneous nodules E. Neurofibromin
developing over the past 20 years. On physical examination the nodules range from She has neurofibromatosis type 1 (NF-1) with multiple subcutaneous neurofibromas,
0.5 to 1.5 cm in size and are firm and nontender. She also has multiple café- au-lait and she has developed a larger malignant peripheral nerve sheath tumor (MPNST).
spots on her skin. She now has pain in her posterior left thigh. MR imaging shows Such MPNSTs may arise from a preexisting neurofibroma. Neurofibromin functions
an 8-cm mass. A mutation encoding for which of the following proteins is most likely as a tumor suppressor.
to be present in this woman? Actin is one of the contractile proteins of muscle and its absence is not compatible
A Actin with life.
B Cardamom Cardamom is an aromatic spice from the ginger family of plants, used in traditional
C Caveolin medicine for oral and intestinal conditions.
D Dystrophin Caveolin is an intracellular vesicle transport protein, and mutations involving this
E Neurofibromin protein may be present in some forms of limb-girdle muscular dystrophy.
F Troponin Dystrophin stabilizes sarcolemmal membranes, and its absence leads to Duchenne
muscular dystrophy.
Troponin is part of the myofiber unit and is used to detect striated muscle injury,
specifically myocardial ischemia and infarction.

❗USE AT YOUR OWN RISK❗ page 59 of 59

You might also like